capítulo 1: números complejos

154
Capítulo 1: Números Complejos. Resolución de ejercicios de fin de capítulo. AVISO IMPORTANTE : Antes de seguir lee este AVISO . Resoluciones completas: 1.- Calcular las siguientes raíces: a) 3 27i Resol. : Trabajaremos en coordenadas polares: 3 3 3 2 2 2 2 6 6 2 3 6 4 3 6 3 27 27 27 3 3 ,3 ,3 k k i 3 6 5 6 9 6 5 5 3 3 3 ,3 ,3 3 cos sin ,3 cos sin ,3 cos sin 6 6 6 6 2 2 i i i 3 1 3 1 3 ,3 , 2 2 2 2 i i 3 i Los demás valores de k dan los mismos tres resultados. (Mientras no se diga nada distinto, en el contexto de este capítulo supondremos k , es decir, que k toma o puede tomar todos los valores enteros: k = 0, ±1, ±2, ±3...) Lo mismo puede hacerse con notación exponencial. Hay que escribir un poco más (ρ·e i en lugar de o incluso ρ θ , sin el angulito) pero potencias y raíces aparecen de forma más natu- ral: 2 2 13 13 2 2 13 2 13 3 3 27 (27 ) 27 27 27 k i i k i i i e e e 2 4 2 2 5 3 6 3 6 3 3 6 6 6 2 3 3 ,3 ,3 3 ,3 ,3 k i i i i i i i e e e e e e e y a partir de aquí con senos y cosenos como antes. Con Mathematica (y de paso vas familiarizándote con cosas que te pueden venir bien): z /. Solve[z^3 == 27I] // ComplexExpand {-3 I, (3 I)/2+(3 Sqrt[3])/2, (3 I)/2-(3 Sqrt[3])/2} En general, para calcular las raíces n-ésimas de un número complejo en forma binómica, una vez calculada una de ellas (por ejemplo la más sencilla) el resto se pueden obtener multipli- cando varias veces por el número complejo 1 ±2π/n escrito también en forma binómica. Esto en ocasiones (no tanto aquí, que es un caso muy sencillo) facilita las operaciones. b) 4 16 Resol. : Con notación polar (o módulo-argumento) por ser ligeramente más breve: 4 4 4 2 2 4 3 4 5 4 7 4 4 4 4 16 16 16 2 2 ,2 ,2 ,2 k k 3 3 5 5 7 7 2 cos sin ,2 cos sin ,2 cos sin ,2 cos sin 4 4 4 4 4 4 4 4 i i i i 2 2 2 2 2 i i 2 1 (4 valores)

Upload: others

Post on 09-Jul-2022

5 views

Category:

Documents


0 download

TRANSCRIPT

Page 1: Capítulo 1: Números Complejos

Capítulo 1: Números Complejos. Resolución de ejercicios de fin de capítulo. AVISO IMPORTANTE: Antes de seguir lee este AVISO.

Resoluciones completas:

1.- Calcular las siguientes raíces:

a) 3 27i

Resol.: Trabajaremos en coordenadas polares:

3 332 2 2 2 6 6 2 3 6 4

3 6 3

27 27 27 3 3 ,3 ,3kk

i

3

6 5 6 9 6

5 5 3 33 ,3 ,3 3 cos sin ,3 cos sin ,3 cos sin

6 6 6 6 2 2i i i

3 1 3 1

3 ,3 ,2 2 2 2

i i

3i

Los demás valores de k dan los mismos tres resultados. (Mientras no se diga nada distinto, en el contexto de este capítulo supondremos k , es decir, que k toma o puede tomar todos los

valores enteros: k = 0, ±1, ±2, ±3...)

Lo mismo puede hacerse con notación exponencial. Hay que escribir un poco más (ρ·ei en lugar de o incluso ρθ, sin el angulito) pero potencias y raíces aparecen de forma más natu-

ral: 2 21 31 3 2 21 3 2 1 33 327 (27 ) 27 27 27

kii kii i e e e

2 42 2 5 36 3 6 33 6 6 6 23 3 ,3 ,3 3 ,3 ,3

k i ii i i i ie e e e e e

e

y a partir de aquí con senos y cosenos como antes.

Con Mathematica (y de paso vas familiarizándote con cosas que te pueden venir bien):

z /. Solve[z^3 == 27I] // ComplexExpand {-3 I, (3 I)/2+(3 Sqrt[3])/2, (3 I)/2-(3 Sqrt[3])/2}

En general, para calcular las raíces n-ésimas de un número complejo en forma binómica, una vez calculada una de ellas (por ejemplo la más sencilla) el resto se pueden obtener multipli-cando varias veces por el número complejo 1±2π/n escrito también en forma binómica. Esto en ocasiones (no tanto aquí, que es un caso muy sencillo) facilita las operaciones.

b) 4 16

Resol.: Con notación polar (o módulo-argumento) por ser ligeramente más breve:

4 44 22 4 3 4 5 4 7 4

44 4

16 16 16 2 2 , 2 , 2 , 2k

k

3 3 5 5 7 7

2 cos sin ,2 cos sin ,2 cos sin ,2 cos sin4 4 4 4 4 4 4 4

i i i i

2 22

2 2i i

2 1 (4 valores)

Page 2: Capítulo 1: Números Complejos

Con Mathematica:

z/.Solve[z^4==-16]//ComplexExpand {(-1-I) Sqrt[2], (1+I) Sqrt[2], (1-I) Sqrt[2], (-1+I) Sqrt[2]}

La figura 1 muestra las posiciones en el plano complejo de z = 27 i y sus tres raíces cúbicas (izquierda) y de z = −16 y sus cuatro raíces cuartas (derecha). En el primer caso, obsérvese cómo el cubo de 3 (módulo de las raíces) es 27 (módulo de z), mientras que el triple de cada uno de los argumentos de las raíces lleva al argumento de z (con separaciones de múltiplos de 2π). En el segundo caso obsérvese cómo la cuarta potencia de 2 (módulo de las raíces) es 16 (módulo de z), mientras que el cuádruple de los argumentos de las raíces también lleva al de z (con separaciones de múltiplos de 2π).

z = 27i

3/6

35/6

33/2

z = -16

2/4

= 21/2(1+i)23/4

= 21/2(-1+i)

25/4

= 21/2(-1-i) 27/4

= 21/2(1-i)

Figura 1. Posiciones de z y de sus raíces en el plano complejo: casos a) y b).

2.- Resolver las siguientes ecuaciones:

a) z4 + 16 = 0

Resol.: Restando 16 a ambos lados: z4 = −16 Se trata de encontrar los números complejos que, elevados a la cuarta, dan −16. Esto es, por definición, encontrar las cuatro raíces cuartas de −16, y por tanto, como en el ejercicio 1b):

2 1z i (cuatro valores)

b) z3 – 1 = 0

Resol.: Por igual motivo se trata de calcular las tres raíces cúbicas de 1. Podemos hacerlo como en ejercicios anteriores, pero para variar vamos a resolver de una manera algo diferente. Una de las raíces es evidentemente 1. Podemos dividir p3(z) = z3 – 1 entre (z – 1) para obtener un polinomio de grado ≤ 2, p2(z), cuyas raíces serán las dos que nos faltan.

Una manera de dividir es “a pelo”:

2

Page 3: Capítulo 1: Números Complejos

3

3 2 2

2

2

1 1

1

1

1

0

z z

z z z z

z

z z

z

comprobándose que, como tiene que ser,

(z – 1) (z2 + z + 1) = z3 + z2 + z − z2 − z – 1 = z3 – 1 = p(z)

Otra manera es dividir mediante una tabla de Ruffini:

1 0 0 1

1 1 1

1 1 1 0

1

con lo que (sabiendo leer la última fila) obtenemos, como antes, que p2(z) = z2 + z + 1.

Finalmente obtenemos las dos raíces de p2 mediante la solución general de la ecuación cua-drática, que funciona en el campo complejo aún mejor que en el real (pues ningún número se escapa del campo complejo como algunas raíces lo hacen del real):

2

2 1 1 4 1 1 1 3 1 31 0

2 2

iz z z

2

Juntando las tres: 1,2,3

1 3 1 31, ,

2 2 2 2z i i

pudiéndose comprobar fácilmente que p(z) = (z – z1) (z – z2) (z – z3) (porque el coeficiente di-rector o principal de p(z), es decir, el coeficiente de z3, es 1).

Comprobamos (y aprendemos) con Mathematica:

z /. Solve[z^3 - 1 == 0] // ComplexExpand {1, -(1/2)-(I Sqrt[3])/2, -(1/2)+(I Sqrt[3])/2}

c) (1+i) z3 – 2i = 0

Resol.: Es una ecuación cúbica, pero no completa. Gracias a ello es fácil despejar z3 para, a continuación, calcular la raíz cúbica pertinente:

342 2

2 1 2 21 2

1 1 1 1

i i iz i

i i

1 3 1 3 6 6 69 171,2,3 4 2 8

12 12 123 12

2 , 2 , 22 2k kz

6 6 63 3 17 172 cos sin , 2 cos sin , 2 cos sin

12 12 4 4 12 12i i i

Eso basta para calcular valores numéricos (con una calculadora); pero calculemos los valores exactos (y aprovechemos para repasar algo de trigonometría).

El segundo valor, z2, es inmediato, pues cos(3π/4) = −21/2/2 y sin(3π/4) = 21/2/2.

3

Page 4: Capítulo 1: Números Complejos

Para la primera raíz, z1, podemos usar las fórmulas de seno y coseno de una diferencia, dado que π/12 = 3π/4 − 2π/3. De esto nos damos cuenta porque ya vamos con la idea de que las raíces cúbicas están separadas por múltiplos de 2π/3, ángulo cuyos seno y coseno conocemos:

3 2 3 2 3 2

cos cos cos cos sin sin12 4 3 4 3 4 3

2 1 2 3 21 3

2 2 2 2 4

3 2 3 2 3 2

sin sin sin cos cos sin12 4 3 4 3 4 3

2 1 2 3 21 3

2 2 2 2 4

Otra opción es usar las fórmulas de seno y coseno del ángulo mitad (con la desventaja de que, aunque fáciles de deducir, no son fórmulas que todo el mundo sepa de memoria):

6 1 cos( 6) 1 3 2 2 3 2

cos cos12 2 2 2 4 2

3

6 1 cos( 6) 1 3 2 2 3 2 3

sin sin12 2 2 2 4 2

Aquí ambos valores se han elegido con signo positivo porque π/12 está en el primer cuadran-te, con lo que tanto su seno como su coseno son positivos. Estos valores pueden parecer dis-tintos de los anteriores, pero son iguales, como se ve por ejemplo elevando al cuadrado:

22

2 2 4 2 3 2 3 21 3 1 3 2 3

4 16 8 4 2

3

22

2 2 4 2 3 2 3 21 3 1 3 2 3

4 16 8 4

3

2

No existe consenso universal para decidir cuál es la forma más sencilla de una expresión ma-temática cualquiera, pero en este caso preferimos las primeras por no contener raíces de raí-ces.

Finalmente, para z3 usaremos el hecho de que 17π/12 = 3π/4 + 2π/3, de nuevo porque sabemos que las tres raíces están simétricamente situadas alrededor del origen del plano complejo:

17 3 2 3 2 3 2

cos cos cos cos sin sin12 4 3 4 3 4 3

2 1 2 3 21 3

2 2 2 2 4

17 3 2 3 2 3 2

sin sin sin cos cos sin12 4 3 4 3 4 3

2 1 2 3 21 3

2 2 2 2 4

Mathematica corrobora estos valores trigonométricos:

4

Page 5: Capítulo 1: Números Complejos

{Cos[Pi/12], Sin[Pi/12], Cos[17Pi/12], Sin[17Pi/12]} {(1 + Sqrt[3]) / (2 Sqrt[2]), (-1 + Sqrt[3]) / (2 Sqrt[2]), -((-1 + Sqrt[3]) / (2 Sqrt[2])), -((1 + Sqrt[3]) / (2 Sqrt[2]))}

Sustituyendo los senos y cosenos:

2 3

61

2 2 22 1 3 1 3 1 3 1 3

4 4 4z i i

1 6 1 2 2 3

62

2 2 2 2 22 1

2 2 2 2z i i

1 i

2 3

63

2 2 22 1 3 1 3 1 3 1 3

4 4 4z i i

Los correspondientes valores numéricos son, usando Mathematica:

z /. Solve[(1 + I) z^3 - 2I == 0] // N {1.08422 + 0.290515 I, -0.290515 - 1.08422 I, -0.793701 + 0.793701 I}

aunque aquí salen en el orden z1, z3, z2.

Esto ha estado bien para repasar trigonometría, pero quizá lo más rápido para obtener los tres valores z1, z2, z3 era partir del más sencillo, que en este caso era z2:

2 3

6 62 3

4

2 2 22 2 ( 1

2 2 2z i

)i

y como sabemos que las otras dos tienen igual módulo y argumentos separados en múltiplos de un tercio de vuelta, multiplicar z2 por 1±2π/3:

2 2 3 2 3 2 33

1 2

2 2 3 2 3 2 33

3 2

2 1 3 2 2( 1 ) ( 1 )( 1 3 ) 1 3 ( 1 3)

2 2 2 4 4

2 1 3 2 2( 1 ) ( 1 )( 1 3 ) 1 3 ( 1 3)

2 2 2 4 4

i

i

z z e i i i i i

z z e i i i i i

saliendo obviamente lo mismo que antes pero usando menos trigonometría.

Finalmente cabe preguntarse qué podríamos hacer si la ecuación cúbica a resolver, en lugar de ser (1+i) z3 – 2i = 0, a la que le faltan los términos en z2 y en z, fuera completa. Ya sabe-mos que la ecuación cuadrática completa, a x2 + b x + c = 0 tiene solución explícita “cerra-da”. Con Mathematica:

También la ecuación cúbica completa, a x3 + b x2 + c x + d = 0, la tiene, solo que más larga (y desdoblándose, lógicamente en 3 valores en lugar de en 2):

5

Page 6: Capítulo 1: Números Complejos

Por tanto, aunque las operaciones fueran más largas, también podríamos aplicar esta fórmula general, dentro del campo de los números complejos, para resolver el problema originalmente planteado, incluso si se tratara de una ecuación cúbica completa1.

d) z3 + z = 0 Resol.: Obviamente una raíz es z1

= 0, con lo que z (z2 + 1) = 0, y las otras dos raíces se sacan de z2 + 1 = 0 z2 = −1 z2,3 = ± i. Por tanto:

1,2,3 0, ,z i i

e) z3 − z = 0

Resol.: Aún más fácil: z3 − z = z (z2 − 1) = 0 1 2,3 0, 1 1z z .

3.- Hallar los números complejos z tales que 2z z .

Resol.: Puedo trabajar en forma binómica casi sin pensar. Llamando z = a + b i:

2 2 2 2 2 2( ) ( ) 2 2z a bi a bi abi a b abi

z a bi

Igualando partes reales e imaginarias: 2 2

2

a b a

ab b

y resuelvo en a, b (reales). De la segunda ecuación, una solución es b = 0 (con cualquier a), y sustituyendo en la primera a2 = a, con lo que o bien a = 0 o bien a = 1. Esto nos da las prime-ras soluciones: z1 = 0, z2 = 1.

1 Bueno es saber que también la ecuación cuártica general a x4 + b x3 + c x2 + d x + e = 0 tiene solución exacta en forma cerrada, pero no así de la quíntica en adelante.

6

Page 7: Capítulo 1: Números Complejos

Si b ≠ 0, la segunda ecuación solo se cumple si 2a = −1, es decir, si a = −1/2. Sustituyendo en la primera, (−1/2)2 – b2 = −1/2 b2 = 1/4 + 1/2 = 3/4 b = ±31/2/2, lo que da las soluciones z3,4 = −1/2 ± 31/2/2 i. Juntando todas:

1,2,3,4

1 3 1 30,1, ,

2 2 2 2z i i

Es fácil comprobar que las cuatro soluciones efectivamente cumplen 2z z (o z*; notar z* al conjugado de z, además de ser bastante común, tiene la ventaja para los estudiantes de esta asignatura de que así es como se hace en Mathematica.)

Otra manera es trabajar en forma polar. Elevar z al cuadrado es elevar al cuadrado el módulo y doblar el argumento, mientras que conjugar es mantener el módulo y cambiar el signo al argumento: 2 2

2( )z 2

z

Para que ρ22θ = ρ−θ, o bien ρ = 0 (en cuyo caso θ da igual, porque solo tenemos la solución

z1 = 0), o bien, si ρ > 0:

2

2 2k

porque para que dos números complejos sean iguales es condición necesaria y suficiente que sus módulos lo sean y que sus argumentos difieran en algún múltiplo de 2π.

De la primera ecuación, con ρ > 0 (el caso ρ = 0 lo hemos tratado antes) necesariamente ρ = 1 (puntos sobre la circunferencia de centro el origen y radio 1). Y de la segunda ecuación:

0 2 4 6

3 2 , , , ,3 3 3 3

k

De estos valores de θ, solo tres dan valores diferentes de z; por ejemplo:

2,3,4 0, 2 3

1

2 2 1 31 cos sin

3 3 2 2

2 2 1cos sin

3 3 2

z i

i i

3

2

i

Todo ello coincide con las cuatro soluciones anteriores (como no podía ser de otra manara).

Expresadas en forma polar: 1,2,3,4 2 3 2 30,1,1 ,1z

4.- Pasar a forma binómica 1

3 4i.

Resol.: Se puede hacer de más de una manera. Por ejemplo, trabajando en forma binómica:

2 2

1 3 4 3 4 3 4 13 4

3 4 3 4 3 4 25 5

i i ii

i i

Para la última raíz cuadrada busco a + b i (con a, b reales) cuyo cuadrado sea 3 + 4 i :

2 2

2 2 2 3( ) 2 3 4

2 4

a ba bi a b abi i

ab

7

Page 8: Capítulo 1: Números Complejos

Para que se cumpla la última ecuación tanto a como b deben ser no-cero, y por tanto puedo despejar b = 4 / (2a) = 2/a y sustituir en la primera:

2

2 423 3a a a

a

2 4 0

Esta es una ecuación bicuadrada, es decir, que si consideramos la incógnita intermedia a2 (en vez de a) la podemos obtener como la solución de una ecuación cuadrática:

2 2 2 2 43 9 4 4 3 5( ) 3 4 0

12 2a a a

Como a es real, las únicas soluciones válidas son a = ± 2, y por tanto b = 2/a = ±1 (el signo positivo con el positivo y el negativo con el negativo). Sustituyendo:

1,2

1 1 1 1 23 4 ( ) 2 1

3 4 5 5 5 5

iz i a b i i

i

También se puede resolver trabajando en forma polar. El módulo de 3 – 4i es ρ = (32 + 42)1/2 = 251/2 = 5; y llamando θ = atan(4/3) = acos(3/5) (el situado en el 1er cua-drante), su argumento es – θ. Entonces:

0

0 2

11 1 1 1 1cos sin

3 4 5 5 2 25 5i

i

Para el coseno y el seno de θ/2 usamos las fórmulas de los del ángulo mitad, que aunque quizá no son para tener permanentemente memorizadas, sí son fórmulas fáciles de deducir:

1 cos 1 3 5 4 2cos

2 2 2 5 5

1 cos 1 3 5 1 1sin

2 2 2 5 5

Aquí hemos tomado la raíz cuadrada positiva en ambos casos porque θ está en el primer cua-

drante. Sustituyendo: 1 1 2 1 12

3 4 55 5 5i

i

i igual que antes.

Comprobamos con Mathematica:

Sqrt[1/(3-4I)] (* Sqrt is a function, so single-valued, so no +- *) 2/5 + I/5

z /. Solve[z^2 == 1/(3-4I)] {-(2/5)-I/5, 2/5+I/5}

Y ya, para terminar de destripar este ejercicio, aclaremos esa pregunta que te viene reco(nco)miendo por dentro. Arriba hemos desechado la ecuación a2 = −1 porque desde el principio habíamos decidido que a sería real. Utilizar ahora esta ecuación sería incorrecto porque, para empezar, la identificación de partes reales y de partes imaginarias con la que comenzábamos, y que luego llevaría a a2 = −1, se basa en suponer a, b reales. Pero ¿qué pa-sará si, contra toda lógica, utilizamos dicha ecuación? Desde luego, si obtenemos alguna “so-lución” adicional al problema usando a2 = −1, será una solución errónea, porque las que hemos hallado son todas las que existen—la raíz cuadrada de cualquier número distinto de cero siempre tiene exactamente dos soluciones distintas en el plano complejo. Aun así cabe

8

Page 9: Capítulo 1: Números Complejos

preguntarse si usar a2 = −1 llevará, en este caso concreto, a algún resultado erróneo o a las mismas soluciones que ya hemos calculado bien. Para salir de dudas hagámoslo:

2 2 2 21 ;

1

i ia a i b

a i i2i

2 2( 1) 2 (2a bi i ii i i i )

Así que haciéndolo mal se obtiene exactamente lo mismo que haciéndolo bien! Ignoraré las implicaciones éticas que pueda tener esto.

5.- Hallar 8 6 i dando la respuesta en forma binómica.

Resol.: En el ejercicio anterior hemos visto cómo trabajar en forma polar, así que ahora lo haremos solo en forma binómica, que es más sencilla (sin trigonometría) y en la que se pide la solución. Buscamos un número z = a + b i con a, b reales cuyo cuadrado sea −8 − 6 i:

2 2

2 2 2 8( ) 2 8 6

2 6 3

a ba bi a b abi i

ab b a

2 2 4 2 2 18 64 36( 3 ) 8 8 9 0

92a a a a a

1 3 3 (1a a b a z 3 )i

6.- Hallar y representar las raíces sextas de la unidad.

Resol.: 6 66 20 2 0 2 3 4 56

3 3 3 3 3

1 1 1 1 ,1 ,1 ,1 ,1 ,1kk

(y los otros valores de k repiten los mismos valores). Insertando los cosenos y senos:

0 2

3 3

3 4 5

3 3 3

1 3 2 2 1 31 1 ; 1 cos sin ; 1 cos sin

3 3 2 2 3 3 2 2

4 4 1 3 5 5 11 1 ; 1 cos sin ; 1 cos sin

3 3 2 2 3 3 2

i i i i

i i i

3

2i

Y la representación gráfica:

z = 1

1/3

12/3

1

14/3

15/3

Figura 2. Posiciones de z = 1 y de sus seis raíces sextas.

7.- Expresar en forma binómica los siguientes números complejos:

a) 10 10

1 1

2 2

i iA

Resol.: El módulo de ambos sumandos es obviamente 1 (tanto antes como después de elevar a 10), y sus argumentos (antes de elevar a 10) son π/4 el del primero y –π/4 el del segundo:

9

Page 10: Capítulo 1: Números Complejos

10 10 10 104 4 5 5

10 104 4 2 2

1 1 1 1 1 1A i 0i

b) 2

2

1

1

i

i

eB

e

Resol.: Aplicando la fórmula de Euler y multiplicando y dividiendo por el conjugado del

denominador: 2 2 2

2 2 2

1 1 1 1 1 2 1 1 2

1 1 1 1 1 1 2

i

i

e i i i i i iB i

e i i i

8.- Comprobar que si iz e , se cumple que iz e .

Resol.: Aplicando fórmula de Euler, definición de conjugado e identidades trigonométricas elementales: cos sin cos sin cos( ) sin( )i iz e i z i i e .

9.- Demostrar que 11 3 1 3 2 cos 3n n

ni i n .

Resol.: Llamemos 1 3z i

El módulo es: 1 3 2

con lo que el argumento es: 3

Arcsin2 3

Por tanto: 3 3 31 3 1 3 2 2 2 2n n n n n n

n ni i 3

2 cos sin 2 cos sin3 3 3 3

n nn n n ni i

12 cos sin 2 cos sin 2 2 cos 2 cos3 3 3 3 3

n n nn n n n ni i

3n n

como queríamos demostrar.

¿Cómo se nos ocurre resolver el problema así? Habiendo imaginando la posición de ambos puntos, 1 ± i 31/2, en el plano complejo. Aunque aquí no los dibujemos (haz tú el dibujo si quieres), una representación gráfica o geométrica de las cosas suele ser de gran ayuda, tanto para el matemático como para el ingeniero. Además cuando hay que hacer muchas multipli-caciones, como al elevar a n, la forma polar suele ser más conveniente que la binómica.

10.- Hallar las ecuaciones de los lugares geométricos de los afijos de los números comple-jos z tales que:

a) | z | = 1.

Resol.: 2 2 1z x y

El módulo de un número complejo es su distancia al origen. Los puntos de módulo 1 forman la circunferencia de centro el origen y radio 1.

b) | z | < 1.

Resol.: 2 2 1z x y

Los puntos de módulo estrictamente menor que 1 forman el círculo abierto (es decir, sin su circunferencia frontera) de centro el origen y radio 1. Es abierto porque de todo punto de di-cho círculo existe un entorno (suficientemente pequeño) totalmente incluido en él.

10

Page 11: Capítulo 1: Números Complejos

c) | z | ≤ 1.

Resol.: 2 2 1z x y

Los puntos de módulo menor o igual a 1 forman el círculo cerrado (es decir, con su circunfe-rencia frontera) de centro el origen y radio 1. Es cerrado porque su complementario es abierto: de todo punto fuera del círculo_con_circunferencia existe un entorno (suficientemente peque-ño) totalmente fuera del círculo_con_circunferencia.

d) 1z z .

Resol.: ( ) ( ) 2 1 1z z x iy x iy x x 2

Estos puntos forman la recta vertical de abscisa (parte real) 1/2.

e) z z i .

Resol.: ( ) ( ) 2 1z z x iy x iy iy i y 2

Estos puntos forman la recta horizontal de ordenada (parte imaginaria) 1/2.

f) 2

z z z .

Resol.: 2 2 2 2 2( ) ( ) 2 2z z x iy x iy x z x y x y x 0

Como veo que esto es una circunferencia, voy a completar cuadrados en la expresión anterior hasta ponerla en la forma (x – x0)2 + (y – y0)2 = r2, y así sabré cuál es el centro (x0, y0) y cuál es el radio r de dicha circunferencia:

2 2 2 2 2 22 ( 1) 1 0 ( 1) ( 0) 1x y x x y x y

(Si no fuera tan sencillo, siempre puedo desarrollar los cuadrados de (x – x0)2 + (y – y0)2 = r2 e identificar coeficientes con x2 + y2 – 2x = 0.)

Por tanto el lugar geométrico definido es la circunferencia de centro el punto (1, 0) y radio 1.

g) 1z z .

Resol.: Por la pinta que tiene, seguramente esto salga más elegantemente trabajando en for-ma polar; pero voy a empezar operado en forma binómica “sin pensar”:

1 22 2

11

x iy x iyz x iy z x y

x iy x iy x y 2

es decir, la circunferencia de centro el origen y radio 1.

Pues más fácil es difícil2… pero aun así hagámoslo también en polares:

1 1z z

Ambos números complejos tienen el mismo argumento (−θ), de modo que serán iguales si y solo si (“ssi”) tienen el mismo módulo, es decir, ssi ρ = 1/ρ (con ρ > 0). Por tanto ρ2 = 1 y ρ = 1 es la única condición. Al igual que antes, el lugar geométrico de los puntos de mó-dulo 1 es la circunferencia de centro el origen y radio 1.

11.- Hallar la ecuación del lugar geométrico que describe el afijo del número complejo z cuando verifica la ecuación: Arg(z2) = −π/4.

Resol.: La siguiente figura, basada en que arg(z) = arg(z2)/2 + k (k = 0, ±1, ±2…), muestra de forma rápida y clara la solución a este ejercicio:

2 En el sentido de que es difícil hacerlo más fácil.

11

Page 12: Capítulo 1: Números Complejos

Arg(z2) = −/4

Arg(z) = −/8

Arg(z) = −/8 +x

y

Figura 3. Solución basada en que arg(z) = arg(z2)/2 + k

El lugar geométrico pedido es la recta que pasa por el origen y tiene por pendiente tan(−/8). Para calcular dicha pendiente podemos usar la fórmula de la tangente del ángulo mitad:

4 1 cos( 4) 1 2 2 2 1

tan tan 1 2 (1 2)8 2 sin( 4) 12 2

yy x

x

De esta recta, hilando fino, habría que excluir el punto z = 0, cuyo argumento es indetermina-do (aunque con módulo 0 cualquier argumento vale, incluyendo –π/4).

Con Mathematica este valor de la tangente se puede confirmar así:

Tan[-Pi/8] // RootReduce 1 - Sqrt[2]

Aun así, estarás pensando que la fórmula de la tangente del ángulo mitad ni te la sabes de memoria ni, aunque sea fácil hacerlo, querrás dedicar tiempo a deducirla el día del examen. A mí me pasaría lo mismo. Veamos otra manera de resolver este ejercicio que no requiere usar dicha fórmula. Suponiendo z ≠ 0 (el caso z = 0 lo hemos comentado antes):

2 22 2 2 2

2tan 1

4Arg( ) Arg ( ) Arg 2 4

2 0

xy

x yz x iy x y i xy

xy

Obsérvese que aquí solo necesitamos el valor de tan(−/4) = −1, no de tan(−/8); y que la últi-ma condición (2xy < 0) se impone porque z2 debe estar en el 4º cuadrante, no en el 2º (con argumento 3/4 la tangente también sería −1). Desarrollando le penúltima condición:

2

2 2 2 4 42 0 2 1 0 1

2

y y yy x xy

x x x

2

De estas dos soluciones a la ecuación cuadrática, la primera no es una solución válida al pro-blema, porque incumpliría la condición 2xy < 0 (x e y tendrían el mismo signo). La única so-lución válida es la misma de antes, y = (1 − 21/2) x. Obsérvese que el valor de tan(−/8) ha apa-recido al resolver la ecuación cuadrática en y/x.

12.- Expresar analíticamente el lugar geométrico de los afijos de los números complejos ta-

les que z i

kz i

sea un número real.

Resol.: Para empezar, z no puede ser −i porque el denominador sería cero y k no estaría de-finido (o sería el punto del infinito del plano complejo extendido, que no es un número real). Dicho lo cual, llamando z = x + i y y multiplicando y dividiendo por el conjugado del de-

12

Page 13: Capítulo 1: Números Complejos

nominador:

2 2 2 2

2 2 2 2

( 1) ( 1) 1 ( ) 1 ( 2 )

( 1) ( 1) ( 1) ( 1)

x iy i x i y x i y x y i xy x xy x x y i xk

x iy i x i y x i y x y x y

Para que este número sea real es condición necesaria y suficiente que −2x = 0, o sea, x = 0 (y que el denominador no sea cero, lo que vuelve a dar x = 0, y = −1, es decir, z = −i). Por tanto el lugar geométrico pedido es el eje imaginario excluido su punto z = −i = (0, −1):

0 salvo el punto (0,-1)x

13.- Hallar el lugar geométrico del afijo del número complejo z sabiendo que los de los nú-meros 1, z y 1 + z2 están alineados.

Resol.: Llamando a los tres puntos z1, z2, z3, y llamando z = x + i y, los puntos del plano complejo correspondientes a esos tres números tienen coordenadas:

1

2

2 2 2 2 2 23

1 (1,0)

( , )

1 1 ( ) 1 2 (1 ,2

z

z z x y

z z x iy x y xyi x y x

)y

)

Aquí hemos hecho abuso de notación, lo cual no está mal porque la notación está para ser usada y abusada. Concretamente hemos escrito como “iguales” (signo =) los números com-plejos y sus correspondientes coordenadas en el plano complejo. Esto lo sugiere el isomorfis-mo existente entre números complejos, puntos del plano complejo, vectores en dicho plano, y conjunto 2 de sus correspondientes coordenadas.

Los tres números estarán alineados si las diferencias entre dos cualesquiera de ellos son vecto-res proporcionales. A simple vista parece que las cuentas saldrán más fáciles si usamos las diferencias z2

− z1 y z3 − z1:

2 22 1 3 1( 1, ) ; ( , 2z z x y z z x y xy

Condición de proporcionalidad:

2 2

2 2 2 222 ( 1) 2 0

1

x y xyx y x x x y x

x y

De esta ecuación no hay que excluir ningún punto (a pesar de que hemos manejado los deno-minadores x − 1 e y, que pueden valer cero) porque para todo punto z los tres puntos z1, z2, z2 están bien definidos y pueden estar alineados (o no). Por otra parte, se ve que la ecuación ob-tenida es la de una circunferencia, y estaría bien saber cuáles son su centro y su radio. Para ello se puede desarrollar (x − x0)2 + (y − y0)2 = r2 e identificar coeficientes para obtener el centro (x0, y0) y el radio r; pero, con un poco de “ojo”, es más rápido completar el cuadrado incompleto que aparece en la ecuación:

2 2 2 2 2 22 ( 1) 1 0 ( 1) ( 0) 1x y x x y x y

con lo que se ve que el lugar geométrico pedido es la circunferencia de centro (1, 0) y radio 1.

14.- Hallar un número complejo z sabiendo que 43 2 ie es una de las soluciones de 3 z . Hallar y representar las soluciones restantes.

13

Page 14: Capítulo 1: Números Complejos

Resol.: Que z1 = 43 2 ie sea una de las raíces cúbicas de z solo significa que z13 = z:

3 33 4 4

3

4

2 2 2i i

z e e z

Queda calcular sus otras raíces cúbicas, que tendrán igual módulo 21/3 que z1 y argumentos separados del de z1 por múltiplos de 2/3:

3 3 32 52,3 2,34 3 12 12

2 2 ,z z 112

Representando los 4 puntos:

z = 2

-3/4

z1 = 21/3

-/4

z2 = 21/3

5/12

z3 = 21/3

-11/12

Figura 4. Raíces cúbicas z1, z2, z3 de z.

Con esto habríamos terminado, porque no se pide nada en forma binómica. Aun así, veamos. En forma binómica z es:

3

4

3 3 2 22 2 cos sin 2 2( 1

4 4 2 2z i i

)i

La raíz z1 proporcionada en forma binómica es:

1 1 1

13 3 3 3 2 6

1 4

2 22 2 cos sin 2 2 1 2 (1

4 4 2 2z i i i

)i

Multiplicando por 1±2π/3 obtenemos las otras dos:

1 1

6 62 1 2 3 1 2

3 3

1 3 1 31 2 (1 ) ; 1 2 (1 )

2 2 2 2z z i i z z i i

y habría que desarrollar y simplificar (el resultado, al final del ejercicio).

Finalmente usando trigonometría:

3 3 3 3 32 5 112,3 4 3 12 12

5 5 11 112 2 , 2 2 cos sin , 2 cos sin

12 12 12 12z i i

Calculamos estos senos y cosenos como los de sumas o diferencias, inspirados por el hecho de que sabemos que z1 tiene argumento de seno y coseno sencillos, y los argumentos de z2, z3 difieren en múltiplos de 2/3, de seno y coseno también sencillos:

5 2 2 2 2 1 2 3 2cos cos cos cos sin sin 1 3

12 4 3 4 3 4 3 2 2 2 2 4

5 2 2 2 2 1 2 3 2sin sin sin cos cos sin 1 3

12 4 3 4 3 4 3 2 2 2 2 4

14

Page 15: Capítulo 1: Números Complejos

y ahora −11/3 quizá más fácil pensando en la simetría sobre la circunferencia goniométrica:

11 5 2 11 5 2cos sin 1 3 ; sin cos 1 3

12 12 4 12 12 4

Todo ello sustituyendo y simplificando da (Mathematica):

z2 = 2^(1/3) (Cos[5Pi/12] + I Sin[5Pi/12]) 2^(1/3) ((-1+Sqrt[3])/(2 Sqrt[2])+(I (1+Sqrt[3]))/(2 Sqrt[2]))

z3 = 2^(1/3) (Cos[-11Pi/12] + I Sin[-11Pi/12]) 2^(1/3) (-((I (-1+Sqrt[3]))/(2 Sqrt[2]))-(1+Sqrt[3])/(2 Sqrt[2]))

15.- Calcular las raíces de la ecuación p(z) = z3 – 17i z2 + (4i – 91) z + (171i + 36) = 0 sabiendo que la de mayor componente imaginaria está situada en la parte positiva del eje ima-ginario y su módulo es tres veces el de las raíces de la ecuación z3 = 27.

Resol.: Las raíces de z3 = 27 tienen módulo 271/3 = 3. Con el triple de ese módulo, y sobre la parte positiva del eje imaginario, la primera raíz de p(x) debe ser z1 = 9i. Para comprobarlo (pues no me fío) construiré una tabla de Ruffini que aquí me vale para dos cosas: para com-probar que p(9i) = 0, y para obtener p(z) / (z – 9i), que, si 9i es efectivamente raíz de p, será un polinomio de grado 2 cuyas dos raíces se podrán calcular sistemáticamente.

Nótese que el motivo por el que puedo hacer una tabla de Ruffini con números complejos exactamente igual que con números reales es que todas las propiedades de suma y producto se cumplen igual con los números complejos que con los reales. Por eso el cuerpo de los núme-ros reales es un subcuerpo del de los complejos.

1 17 4 91 171 36

9 9 72 171 36

1 8 4 19

i i i

i i i

i i 0

Efectivamente p(9i) = 0 (esquina inferior derecha) y el polinomio cociente es (sabiendo leer la última fila) q(z) = z2 – 8i z + (4i – 19). Si no te fías de Ruffini (porque no lo entiendes) es-tudia el algoritmo de Hörner y entiéndelo. Y si no te fías de las operaciones, desarrollando comprueba que efectivamente:

q(z) (z – 9i) = (z2 – 8i z + 4i – 19) (z – 9i) = z3 – 17i z2 + (4i – 91) z + (171i + 36) = p(z)

Mathematica también lo confirma:

(z^2 – 8I z + 4I-19) (z - 9I) // Expand (36 + 171 I) - (91 - 4 I) z - 17 I z^2 + z^3

Las otras dos raíces de p serán las de q:

2 8 64 4(4 19) 18 (4 19) 0 4 12 16 4 3 4

2 2

i iz iz i z i i i

i

22 2

2 2 23 23 4 2 3 4 3

2 4

a bi a bi a b abi i a

aab

2 2 2 2 43 9 4 4 3 5 2( ) 3 4 0 2 , 1

12 2 2aa a a a b

1,2,3

2 33 4 2 (2 ) 4 (2 ) 9 ,2 3 , 2 5

2 5

ii i i z i i z i i

i

i

15

Page 16: Capítulo 1: Números Complejos

16.- La suma de dos números complejos es 3 + 2i, y la parte real de uno de ellos es 2. Hallar dichos números sabiendo que su cociente es un número imaginario puro.

Resol.: Sean dichos números z1 = x1 + i y1, z2 = x2

+ i y2

Con x1 = 2; x1 + x2 = 3; y1

+ y2 = 2 x2 = 1, y2 = 2 − y1

sustituyendo queda z1 = 2 + i y1, z2 = 1 + i (2 − y1)

Que su cociente sea imaginario puro significa que la parte real de dicho cociente (da igual z1

/ z2 o z2 / z1) sea cero; por ejemplo:

2

2 1 1 1 1 12

1 1 1 1 1

1 (2 ) 1 (2 ) 2 2 2 (...)

2 2 2 4

z i y i y iy y y i

z iy iy iy y

La parte imaginaria del numerador no la he calculado porque no la necesito. Para que esto sea imaginario puro:

21 1 1

2 4 4 2 122 2 0 1 1

2 2y y y

3

Sustituyendo se obtienen dos soluciones:

1 1

2 2

2 1 3 2 1 3o bien

1 1 3 1 1 3

z i z i

z i z i

Compruébese que ambas soluciones efectivamente cumplen todas las condiciones del enun-ciado (lo cual es buena costumbre, en general, para detectar posibles errores en la resolución).

La interpretación geométrica de este ejercicio, aunque no se pide, es interesante, así que la haremos. Se trata de encontrar dos números, uno sobre la recta vertical x = 1 y otro sobre x = 2, cuyas ordenadas sumen 2 y de forma que sus radio-vectores sean perpendiculares (para que su cociente sea imaginario puro). Obsérvese en la figura cómo se cumplen dichas condiciones.

(2, 1+31/2)

(1, 1-31/2) (2, 1-31/2)

(1, 1+31/2)

0 1 2

Figura 5. Interpretación geométrica de las dos soluciones del ejercicio.

17.- Dada la ecuación z2 − 8iz − (19 − 4i) = 0 cuyas raíces son z1 y z2, hallar los números complejos z3 tales que los afijos de z1, z2 y z3 formen un triángulo rectángulo isósceles (el vértice del ángulo recto debe ser el afijo de la raíz de mayor componente imaginaria).

16

Page 17: Capítulo 1: Números Complejos

Resol.: Los z1 y z2 de este ejercicio son los z2 y z3 del ejercicio 15, y cuyos valores calculá-bamos ahí: z1 = 2 + 3i, z2 = −2 + 5i

El de mayor parte imaginaria es z2, luego ahí debe estar el ángulo recto. Si el triángulo es

isósceles un cateto será 1 2z z , y el otro de igual longitud (módulo) y perpendicular a él (ver

figura).

-6 -4 -2 0 2 4 60

2

4

6

8

z1 = (2, 3)

z2 = (-2, 5)

z3 = (0, 9)

z3 = (-4, 1)

Figura 6. Representación geométrica del ejercicio.

Esto se consigue multiplicando (z1 – z2) por i:

(z1 – z2) i = [2 + 3i − (−2 + 5i)] i = (4 – 2i) i = 2 + 4i

Por tanto habrá dos posibles puntos z3, que serán:

3 2

9(2 4 ) 2 5 (2 4 )

4

iz z i i i

i

18.- Se consideran los números complejos z1, z2 raíces de la ecuación:

z2 – (6 + 8i) z + (1 + 30i) = 0

Determinar otros dos números complejos z3, z4 cuyos afijos en el plano complejo sean vérti-ces opuestos de un cuadrado, siendo sus otros dos vértices los afijos de z1 y de z2.

Resol.: Lo primero será calcular z1, z2:

2

21,2

6 8 (6 8 ) 4(1 30 )(6 8 ) (1 30 ) 0

2

i i iz i z i z

36 64 96 4 120 32 24

3 4 3 4 3 4 8 62 2

i i ii i

i i

22 2

2 28 3 3( ) 8 6

2 6

a ba bi i b a

a aab

8

2 2 2 2 18 64 4 9( ) 8 9 0 1, 3

92 aa a a a b

28 6 (1 3 ) ; Compruebo: (1 3 ) 1 9 6 8 6 oki i i i i

Esta raíz cuadrada también la puedo calcular en polares, aunque haciendo uso de la fórmula del seno y del coseno del ángulo mitad. Calcularé la raíz de −8 − 6i perteneciente al cuarto cuadrante, y la otra será su opuesto. El módulo de −8 − 6i es (82 + 62)1/2 = 1001/2 = 10, con lo que el coseno de su argumento es un número sencillo (cos θ = −8/10 = −4/5):

17

Page 18: Capítulo 1: Números Complejos

4º cuadrante2

1 cos 1 cos8 6 10 10 10 cos sin 10

2 2 2 2

ai i

i

1 4 5 1 4 5 1 9

10 10 1 32 2 10 10

i i

i

como antes.

Sustituyendo: 1,2 3 4 8 6 3 4 (1 3 ) 4 ,2 7z i i i i i i

Con Mathematica:

z /. Solve[z^2 - (6 + 8I) z + (1 + 30I) == 0] {2 + 7 I, 4 + I}

Ya podría representar “en su sitio” los puntos z1, z2, la diagonal que los une, su punto me-dio m, y el cuadrado con sus otros dos vértices (pedidos). Hazlo tú si quieres. Yo puedo conti-nuar pensando en un cuadrado “abstracto” sin tener que dibujarlo, tanto menos “a escala”.

El punto medio del cuadrado será la semisuma de sus vértices opuestos z1 y z2:

1 2 4 2 7 6 83 4

2 2 2

z z i i im i

Una “semidiagonal centrífuga” será s1 = z1 − m (también valdría z2

− m). Multiplicándola por i conseguiremos su semidiagonal perpendicular s2 de igual módulo, que sumada o restada a m nos dará los otros dos vértices buscados:

1 1 2 14 (3 4 ) 1 3 ; (1 3 ) 3s z m i i i s s i i i

i

3,4 2 3 4 (3 ) 6 5 , 3z m s i i i i

18

Page 19: Capítulo 1: Números Complejos

Capítulo 1: Números Complejos. Resolución de ejercicios de pasados exámenes. AVISO IMPORTANTE: Antes de seguir lee este AVISO.

Resoluciones completas:

Ejercicio 1 (de examen de diciembre de 2015):

a) Determinar los números complejos tales que 4 0z z .

Resol.: Parece que va a ser más fácil trabajar en coordenadas polares. Llamando z = ρθ:

4 4 440z z z z

Para que dos números complejos sean iguales deben ser iguales sus módulos y tener argumen-tos que difieran en múltiplos de 2π (salvo que el módulo sea cero, en cuyo caso los argumen-tos dan igual).

Respecto a los módulos, las únicas soluciones reales no-negativas de ρ4 = ρ son ρ = 0 (lo que directamente nos da una solución, z = 0, sin necesidad de mirar al argumento) y ρ = 1. Con ρ = 1 debe además cumplirse:

2 2

4 2 ( 0, 1, 2...) 5 2 0, ,5 5

kk k k

4

5

resultando los demás valores de k en valores repetidos de z. Por tanto:

2 4

5 5

0,1,1 ,1z

(6 soluciones)

Como no se pide expresar z en forma binómica, no hace falta calcular los cosenos y los senos de ±2π/5.

b) Las dos raíces de la ecuación 2 (3 3 ) 2 3 0z i z i son uno de los vértices de un triángulo equilátero (la de mayor componente real) y el centro de la circunferencia cir-cunscrita a dicho triángulo. Hallar los otros dos vértices.

Resol.: Calculemos primero las raíces z1, z2 de esa ecuación:

2

2 3 3 (3 3 ) 4(2 3 )( ) (3 3 ) 2 3 0

2

i ip z z i z i z

i

3 3 9 3 6 3 8 4 3 3 3 2 2 3

2 2

i i i i

i

2 2

2 2 22 3

( ) 2 2 2 32 2 3

a ba bi a b abi i b

aab

2

2 2 2 2 2 13 22 ( ) 2 3 0

32 2a a a a

a

4 4 3 2 4

1,2

3 3 (1 3 ) 2 31 3

2 1

i i ia a b z

Page 20: Capítulo 1: Números Complejos

Hay personas de “mirada penetrante” que detectan a golpe de vista que una de las raíces de p(x) era z = 1. (Yo lo veo fácil solo a posteriori.) Si tú eres una de esas, te habrías podido aho-rrar unas operaciones calculando q(z) = p(z)/(z − 1) mediante una tabla de Ruffini:

1 3 3 2 3

1 1 2

1 2 3 0

i i

i

i

3

con lo que ( ) 2 3 0 2 3q z z i z i da la otra raíz.

La raíz de mayor componente real es el primer vértice del triángulo, v1 = 2 + 31/2 i, siendo la otra raíz, c = 1 su circuncentro (o incentro, o baricentro, pues el triángulo es equilátero). El vector1 centrífugo que va desde el centro c hasta el vértice ya conocido v1 es vc1 = v1

− c = (2 + 31/2 i) − 1 = 1 + 31/2 i. Una manera de obtener los otros dos vértices es rotar vc1 un tercio de vuelta a derecha y a izquierda (±2π/3 radianes) y sumar ambos vectores al centro c. Las rotaciones se consiguen multiplicando por los números complejos de módulo 1 y argumento ±2π/3:

2

3

2 2 11 cos sin

3 3 2i i

3

2

2 1

1 3 1 3 1 3 3 31 (1 3 ) 1 1

2 2 2 2 2 2 2v c vc i i i i

3 1

1 3 1 3 1 3 2 31 (1 3 ) 1 2 3

2 2 2 2 2 2 2v c vc i i i i i

Aunque he podido hacer todos los cálculos apodícticamente pensando solo en un triángulo y una circunferencia “abstractos” (sin representación gráfica, y mucho menos en verdadera po-sición), y aunque no se pide en el ejercicio, aquí está dicha representación gráfica.

-2 0 2 4

-2

0

2

c = 1

v1 = 2 + 31/2i

v2 = -1

v3 = 2 - 31/2i

Figura 1. Triángulo equilátero y circunferencia circunscrita.

c) Hallar, en forma binómica, las raíces de la ecuación z7 − 9 z4 + 8 z = 0.

Resol.: Una raíz es obviamente z = 0 (hasta ahí ya llega mi penetrante mirada). Las de-más se obtendrán resolviendo z6 − 9 z3 + 8 = 0. Tomando como incógnita intermedia z3

1 Por el isomorfismo existente entre números complejos y puntos y vectores del plano (y elementos de 2), en

adelante me permitiré decir casi indistintamente “vector”, “número (complejo)” o “punto”.

2

Page 21: Capítulo 1: Números Complejos

(ecuación bicúbica):

3 2 3 3 89 81 4 8 9 7( ) 9 8 0

12 2z z z

Añadiendo a z1 = 0 las tres raíces cúbicas de 8 y las tres raíces cúbicas de 1:

1,2,3,4,5,6,7 2 3 2 3 2 3 2 30, 2, 2 , 2 ,1,1 ,1z

y como se piden en forma binómica:

2 2 1 3

cos sin3 3 2 2

i i

1,2,3,4,5,6,7

1 3 1 3 1 3 1 30, 2, 2 , 2 ,1,1 ,1

2 2 2 2 2 2 2 2z i i i

i

1 3 1 3

0, 2, 1 3, 1 3,1, ,2 2 2 2

i i i i

Con Mathematica:

z /. Solve[z^7 - 9z^4 + 8z == 0] // ComplexExpand {0, 1, 2, -(1/2)-(I Sqrt[3])/2, -(1/2)+(I Sqrt[3])/2, -1 - I Sqrt[3], -1 + I Sqrt[3]}

Ejercicio 2 (de examen de enero de 2016):

a) ¿Pueden ser z1 = 2 + i, z2 = −2 + i, z3 = −1 − 2 i, z4 = 1 − 2 i las cuatro raíces cuartas de un número complejo? Razonar la respuesta.

Resol.: La directa sería elevar esos números a la cuarta y ver si sale siempre lo mismo. Pero nos vamos a poder ahorrar incluso esas sencillas operaciones.

Una representación en el plano complejo, aun muy aproximada, de las cuatro supuestas raíces basta para ver que, aunque de igual módulo (como debe ser), sus argumentos no difieren en múltiplos de 2π/4 (como debería ser):

-3 -2 -1 0 1 2 3-3

-2

-1

0

1

2

2 + i -2 + i

1 - 2i-1 - 2i

Figura 2. Las cuatro raíces impostoras.

Si queremos una justificación numérica, el método más bruto consistiría en calcular los argu-mentos de z1, z2, z3 y z4 y comprobar que no difieren en múltiplos de π/2. Mucho trabajo tri-gonométrico innecesario. Más fácil es comprobar que no hay dos números que sean uno el opuesto del otro: por ejemplo −z1 = −(2 + i) = −2 − i no es una de las otras tres supuestas raí-ces. Además de lo dicho al principio (calcular zi

4).

Por todo ello no, no pueden ser las cuatro raíces cuartas de un número (salvo que ese número vaya cambiando ;-).

3

Page 22: Capítulo 1: Números Complejos

b) Resolver la ecuación 4

3z iz

.

Resol.: Suele convenir quitarse pronto de en medio los casos singulares (y aún no sé si aquí será importante): z = 0 no puede ser solución de 4z z 3i , porque 4/z no está definido.

Ahora trabajemos en forma binómica (porque no solo hay multiplicaciones y divisiones, que sí suelen ser más sencillas en polares, sino también sumas y restas). Llamando z = x + y i:

2 2

2 2

2 2 2 2

2 2

4 4 4 4

40

4 43

43

4x iy x iyz x iy x iy x iy

z x iy x iy x iy x y

xx

x yx yx i y i

yx y x yy

x y

La primera ecuación se cumple si x = 0 para cualquier y ≠ 0 (y si y = 0 ya vimos que z = 0 no sirve). En el caso de que x ≠ 0, la primera ecuación se cumple solo si x2 + y2 = 4. Pero sustituyendo esto en la segunda quedaría −y + y = 3, obviamente incompatible. Por tanto, si existe alguna solución, será con x = 0. Sustituyendo en la segunda (de nuevo, con y ≠ 0):

22

14 4 3 9 163 3 4 0

42

yy y y y y

y y

Por tanto las soluciones serán 1 2, 4z i z i

Vamos a comprobar que cumplen la ecuación original. Esto es buena costumbre siempre, pero además en este caso las ecuaciones anteriores ha habido que tratarlas con cierto cuidado y me he podido equivocar en algo, razón de más para hacer la comprobación:

1 21 2

4 4 4 44 3 ok; 4 4 3 ok

4z i i i i z i i i i

z i z i

Y finalmente Mathematica también nos da la razón:

z /. Solve[Conjugate[z] - 4/z == 3I] {I, -4 I}

c) Hallar dos números complejos cuya suma sea 3 + i, la parte real del primero sea 2, y su producto sea un número real.

Resol.: Sean dichos números z1 = x1 + i y1

; z2 = x2 + i y2. Planteemos las ecuaciones:

1

1 21 2 1 2 1 2

1 2

1 1 2 2 1 2 1 2 1 2 1 2

2

3( ) 3

1

( )( ) [...] ( ) 0

x

x xz z x x i y y i

y y

x i y x i y i y x x y y x x y

De las dos primeras x1 = 2, x2 = 1. Sustituyendo en la última, junto con la penúltima:

1 22 1

1 2

11, 2

2 0

y yy y

y y

Por tanto: 1 22 2 ; 1z i z i

4

Page 23: Capítulo 1: Números Complejos

No olvides ahora comprobar estos valores de z1, z2 cumplen todas las condiciones originales.

Ejercicio 3 (de examen de junio de 2016):

a) El número complejo z1 = 2 + 4 i es el centro de una circunferencia de radio 2. Sea ABC un triángulo equilátero circunscrito a la circunferencia anterior. Uno de los puntos de tangencia es el número complejo z2 = 4 i. Hallar el número complejo correspondiente al vértice del triángulo de mayor componente imaginaria.

Resol.: Circunscrito significa “tocando por fuera”, e inscrito “tocando por dentro”. El trián-gulo está circunscrito a la circunferencia, y la circunferencia está inscrita en el triángulo.

Los puntos z1, z2 tienen igual parte imaginaria, luego en el plano complejo están en la misma horizontal, y la tangencia a la circunferencia será vertical (ver figura). La distancia de cada punto de tangencia al centro de la circunferencia debe ser 2 (su radio), así que primero com-pruebo que la distancia entre z1 = 2 + 4i y z2 = 4i es 2, dato redundante del problema (y sí, lo es; si no lo fuera, ya habría terminado el ejercicio diciendo que está mal planteado).

z1

z2

C = z

3

A

B

Figura 3. Circunferencia con triángulo equilátero circunscrito.

Por ser el triángulo equilátero, el ángulo 2 3ACB z z B es π/3 (= 60º); y por simetría (tangen-

tes a la circunferencia desde un punto exterior a ella) el ángulo 2 3 1z z z es la mitad de ACB , es

decir, π/6 (= 30º). Por tanto:

1 22 3 1 2 3

2 3 2 3

sin( 6) 1 2 1 2tan tan 2 3

6 cos( 6) 3 2 3

z zz z z z z

z z z z

3 2 32 3 4 2 3 4 2 3z z i i i z i

Esto ha sido muy fácil porque la tangente en z2 es vertical. En otros triángulos equiláteros “más inclinados” puede venir bien saber que la distancia del centro del triángulo al centro de cualquier lado es la mitad de la distancia a cualquiera de los vértices, o cómo girar un vector en el plano complejo 30º ó 60º.

b) Sean z1 y z2 las raíces de la ecuación z4 + z2 + 1 = 0 situadas en el semiplano supe-rior. Hallar el valor de z1

3 + z23.

Resol.: Calculo las cuatro raíces de esta ecuación bicuadrada y veo cuáles tienen parte ima-ginaria positiva:

2 32 2 2 2

2 3

11 1 4 1 3( ) 1 0

12 2 2z z z i

De estas últimas formas polares me doy cuenta porque nada más ver 1/2 y 3 2 me tienen que recordar al coseno y al seno de π/3!

5

Page 24: Capítulo 1: Números Complejos

Los valores de z serán por tanto: 3 3 3 3

1 ,1 ,1 ,1

Los de parte imaginaria positiva: 21,2

3 3

1 ,1z

La suma de sus cubos: 3 3 3 31 2 3 3 6 31 1 1 1z z 0

Ejercicio 4 (de examen de diciembre de 2014):

Hallar los afijos de los 4 vértices z1, z2, z3, z4 de un rombo sabiendo que: i) z1 es imaginario puro negativo con | z1

| = 3; z2 está situado en el 2º cuadrante; z1 y z2 son vértices opuestos del rombo;

ii) 2 2

1 1

5 ; Re 1z z

z z

iii) 3 1 2 1

5

2z z z z

Resol.: Si z1 es imaginario puro negativo con | z1 | = 3 solo puede ser:

z1 = −3i

Las otras condiciones de i) las usaré más adelante.

Por ii): 2 22 1 2 25 3 5z z x y

2 2 2 22

1

Re Re 1 33 3

z x i y yy

z i

2 2 22 2(3 5) 45 3 6x y

pero, por i), z2 está en el 2º cuadrante: z2 = −6 + 3i

Por iii):

2 23 1 2 1

5 5 5 5 56 3 3 6 6 6 6 6 2 3 10

2 2 2 2 2z z z z i i i

Con esto ya puedo hacer una figura que se parezca bastante a la definitiva (a falta de que el rombo sea más ancho o más estrecho). La Fig. 4 muestra la definitiva.

z1 = - 3i

z2 = - 6 + 3i

m = - 3

z3 = 3 + 6i

z4 = - 9 - 6i

x

y

3·101/26·21/2

3·21/2

Figura 4. Rombo del ejercicio.

6

Page 25: Capítulo 1: Números Complejos

Ahora podría calcular la ecuación de la mediatriz del segmento z1-z2, así como la de la circun-ferencia de centro z1 (o z2) y radio 3·101/2. Sus puntos de intersección me darían los puntos z3 y z4. Pero habría que resolver una ecuación cuadrática. Quizá sea más fácil, a la par que ins-tructivo, calcular el punto medio m del rombo, aplicar el Teorema de Pitágoras para obtener la distancia de z1 a z3 (o a z4) (aunque esto también sea resolver una ecuación cuadrática senci-lla) y usar giros y sumas de vectores para llegar desde m hasta z3 y hasta z4. Así lo haremos.

Punto medio del rombo: 1 2 3 ( 6 3 )3

2 2

z z i im

Distancia de m a z1: 1 1 3 3 9 9 3 2mz m z i

Distancia de m a z3:

2 2 2 2

3 1 3 1 (3 10) (3 2) 90 18 72 36 2 6mz z z mz 2

Veo que es el doble que la distancia de m a z1, con lo que puedo girar el vector z1−m un ángu-lo recto hacia la izquierda, multiplicar por 2, y sumar a m para obtener z3 (y análogamente z4):

Semidiagonal menor: 1 3 ( 3) 3 3z m i i (3 3 ) 3 3i i i

Giro de + /2 a la izqda.: 2(3 3 ) 3 6 6 3z m i i

2(3 3 ) 3 6 6i i

Puntos que faltan:

3 6

4 9 6z m i

i

En resumen: 1,2,3,4 3 , 6 3 , 3 6 , 9 6z i i i i

Ejercicio 5 (de examen de junio de 2014):

a) Calcular, dando el resultado en forma binómica, el valor de la siguiente expresión:

8

cos sin12 12

A i

Resol.: 8 812 8 12 2 3

2 2 11 1 1 cos sin

3 3 2A i

3

2i

b) Sea 1

3 3 33

2 2z

i un vértice de un triángulo equilátero inscrito en una cir-

cunferencia de centro 3 2C i . Hallar el vértice de mayor componente imaginaria de dicho triángulo.

Resol.: Vector “centrífugo” v1 desde el centro C hasta el vértice z1:

1 1

3 3 3 3 3 33 3 2

2 2 2 2v z C i i i

2

Ahora lo giraría ±2 /3 (sendos tercios de vuelta) para obtener los otros dos vectores centrífu-gos, v2 de C a z2 y v3 de C a z3. Sumándolos a C obtendría ambos vértices, y miraría cuál tie-ne mayor componente imaginaria.

Pero no quiero hacer más operaciones de las necesarias (soy vago natural), así que voy a in-tentar saber cuál tendrá mayor parte imaginaria sin hacer operaciones. El vector v1 tiene parte real negativa, luego girándolo −π/3 siempre quedará mayor parte imaginaria que girándolo

7

Page 26: Capítulo 1: Números Complejos

+π/3, con lo que sumado a C también dará mayor parte imaginaria. Llamando v2 a ese vector, y z2 al correspondiente vértice (pedido):

2 1 2

3

3 3 3 1 3 3 9 3 3 3 31 2 3 1

2 2 2 2 4 4 4 4v v i i i

3 3 i

2 2 23 2 3 3 3 3z C v i i z i

Nótese cómo no he dibujado nada, pero he pensado en términos geométricos apodícticos. A veces un dibujo más o menos “realista” ayuda; otras veces, como ésta, es innecesario.

Ejercicio 6 (de examen de enero de 2014):

i) Obtener y representar gráficamente el lugar geométrico de los números complejos

z = x + i y tales que 2

1z

z .

Resol.: Suele convenir quitarse cuanto antes de en medio los casos singulares. Aquí z = 0 no vale porque α no estaría definido (división por cero). Por lo demás:

2 2

2 2 2 2 2 2 2 2

1 1 1 1 2

( ) 2 2 2

z x iy x iy x iy x y i xy

z x iy x y i xy x y i xy x y i xy

Como vago natural, voy a hacer las menos operaciones posibles. Que α sea real significa que su parte imaginaria sea cero. El producto de denominadores es real (para eso hemos multipli-cado y dividido por ese conjugado), de modo que la condición es que la parte imaginaria del numerador sea cero:

2 2 2 3 2

2 2

0( ) 2 ( 1) 2 2 0

2 0

yy x y xy x yx y x y xy

x y x

La recta y = 0 es el eje real (y obviamente (z − 1) / z2 es real para todo z real distinto de cero). La otra ecuación representa una circunferencia cuyos centro y radio obtenemos fácilmente:

2 2 2 2( 1) 1 0 ( 1) ( 0)x y x y 1

Por tanto se trata de la circunferencia de centro 1 = (0, 1) y radio 1 (excluido z = 0).

En total el lugar geométrico pedido es: 2 2

0salvo 0

( 1) ( 0) 1

yx y

x y

Representación gráfica:

Figura 5. El lugar geométrico pedido.

ii) De los infinitos puntos que pertenecen al lugar geométrico anterior, dar dos puntos α1 y α2 tales que β = α1

− α2 sea un número imaginario puro.

8

Page 27: Capítulo 1: Números Complejos

Resol.: Habrá infinitos pares. Siempre que α1 y α2 tengan la misma parte real, es decir, sean puntos situados sobre la misma vertical, su diferencia será un número imaginario puro. Por tanto podemos cortar el lugar geométrico del apartado anterior por cualquier recta vertical (distinta de x = 0) y cualesquiera dos de los puntos de corte (uno o tres) pueden ser α1 y α2. Como solo se pide un par, quizá el más sencillo no trivial sea:

1 21 ; 1i i

Trivialmente también vale cualquier número real α1 = α2

0, pues su diferencia, siendo cero, tiene parte real cero, y por tanto se puede considerar imaginaria pura. Y por supuesto cual-quier número sobre la circunferencia α1

0 y su parte real α2 = Re{α1} cumplirían lo pedido.

Lo anterior es tan sencillo que no merece representación geométrica. Pero se me ocurre una pregunta más interesante: ¿qué pares de puntos α1, α2 situados sobre la circunferencia del apartado anterior son tales que su cociente α1

/ α2 sea imaginario puro? El cociente de dos números complejos es imaginario si y solo si sus vectores de posición (“radiovectores”) for-man ángulo recto (porque, al dividir, los argumentos se restan). Por virtud del “arco capaz”,2 esto sucede para cualquier par de puntos α1, α2 diametralmente opuestos sobre la circunferen-cia (ver Fig. 6).

Figura 6. El cociente α1

/ α2 es imaginario puro.

También en este caso el par de puntos más sencillo parece ser α1 = 1 + i, α2 = 1 − i.

iii) Sean 1

3 1

2 2z i , z2 = z1

21, y z3 la raíz situada en el 2º cuadrante de la ecuación

z3 − 1 = 0. Expresar z2 y z3 de forma binómica.

Resol.: 21

21 212 6 21 18 3 2

36 6 6 2

3 11 1 1 1

2 2z i z

i

Respecto a z3: 1 33 3 1 330 0 2

3 3

1 0 1 1 1 1 1kk

z z z 2

con k = 0, ±1, ±2…. Solo una de estas raíces, la correspondiente a, p.ej., k = 1, está en el se-

gundo cuadrante: 3 2 3 3

2 2 11 cos sin

3 3 2z i z

3

2i

Ejercicio 7 (de examen de noviembre de 2013):

i) Calcular

4

1 3

1

iw

i

, dando el resultado en forma binómica.

2 ¿Todavía estudiáis esto en Dibujo Técnico? Me dice una de vosotras que sí.

9

Page 28: Capítulo 1: Números Complejos

Resol.: No me apetece hacer trigonometría, así que haré todo en forma binómica—elevar al cuadrado y otra vez al cuadrado no parece mucho trabajo:

2

22

(1 3 ) 1 3 2 3 2 2 33 ; ( 3 ) 3 1 2 3 2 2 3

(1 ) 1 1 2 2

i i ii i i w

i i i

i

ii) Hallar el vértice A de menor componente imaginaria del triángulo equilátero circunscri-

to a la circunferencia de centro 2(1 3 )C i y tangente al triángulo en el punto que vie-ne dado por el número complejo 3 + 2i.

Resol.: Llamemos z1 al punto de tangencia dado, z1 = 3 + 2i.

La Fig. 7 muestra la circunferencia, el triángulo y su vértice A, cuyas coordenadas se piden.

C = 2 + i 2·31/2

z1 = 3 + 2 i

A Figura 7. Circunferencia, triángulo circunscrito, y vértice pedido A.

Este es el plan—se puede hacer de otras maneras:

1º Calcular el vector centrífugo v1 = z1 − C.

2º Girarlo 90º hacia la derecha (multiplicándolo por −i) y multiplicar su módulo por la co-

tangente del ángulo , que, siendo el triángulo equilátero, sé que vale 30º. 1z AC

3º Ese vector, sumado a z1, me dará A.

Así: 1 1 (3 2 ) (2 2 3 ) 1 2(1 3)v z C i i i

1 1

2(1 3) 2 2 3( ) 2(1 3) ; 2 3 6 3

tan( 6) 1 3

i iv i i A z i

1 2 3 6 3 3 2 2 3 6 3 3 2 3 2 3A z i i i A i

Ejercicio 8 (de examen de junio de 2013):

1- Expresar en forma binómica los siguientes números complejos:

i) z1 = (1 + i)25 ii) z2 = (1 − i)26 iii) z3 = (−1 + i)27

Resol.: i) 2525 25 2 12 1 2 12 12

3(2 )1 25 244 4

4 4 4

(1 ) 2 2 2 2 2 2 2z i

4

12 12 121

2 22 2 cos sin 2 2 2 1

4 4 2 2i i z

i

10

Page 29: Capítulo 1: Números Complejos

ii) 2626 26 2 13 13 13 13

2 26 24 24 3(2 )4 4 4 2 2

(1 ) 2 2 2 2 2 1 2z i z

2 i

iii) 2727 27 2 13 1 2 13 133

10(2 )3 81 10 84 4

4 4 4

( 1 ) 2 2 2 2 2 2 2z i

4

13 13 133

2 22 2 cos sin 2 2 2 1

4 4 2 2i i z

i

2- Hallar el lugar geométrico de los números complejos 1( 3 )z i z y representarlo

gráficamente en los siguientes casos:

i) 1

11 1

1

1( ) con

[0, 3]z

ii)

1

11 1

1

2( ) con

[ 6, 2 3]z

Resol.: Como vamos a multiplicar, primero convendrá expresar 3 i en forma polar. Su

módulo es 3 1 2 . La tangente de su argumento es 1/ 3 , y el coseno y el seno de su ar-

gumento son respectivamente 1/2 y 3 / 2 . Obviamente:

63 2i

i) Operando: 1 11 6 6( 3 ) 2 1 2z i z

donde 1 [0, /3] y por tanto /6+1

[ /6, /2].

Ecuación y figura: 2 con [ 6, 2]z

Figura 8. Lugar geométrico pedido.

ii) Operando: 1 11 6 6( 3 ) 2 2 4z i z

donde 1 [ /6, 2 /3] y por tanto /6+1

[ /3, 5 /6].

Ecuación y figura: 4 con [ 3, 5 6]z

Figura 9. Lugar geométrico pedido.

Ejercicio 9 (de examen de enero de 2013):

Dada la ecuación z2 + (−6 + 2i) z + (11 − 2i) = 0, se pide:

i) Hallar sus raíces z1, z2. Considerar z2 la situada en el primer cuadrante.

11

Page 30: Capítulo 1: Números Complejos

ii) Sean z1, z2 los números complejos cuyos afijos son los vértices de un lado de un hexá-gono regular. Hallar el número complejo z3 cuyo afijo es el otro vértice, el de mayor compo-nente imaginaria, contiguo a z2.

Resol.: i) 26 2 ( 6 2 ) 4(11 2 ) 36 4 44 ( 24 8)

32 2

i i i iz i

12 16

3 32

ii i

3 4i

2

2 2 2 2 2 12 33 ( ) 3 4 0

42a a a a

a

9 16

Tú puedes comprobar que sustituyendo estos dos valores en la ecuación original esta se cum-ple. Yo prefiero recordarte cómo comprobarlo con Mathematica:

4 31 2 3 (1 2 )

2

ia a b z i i

i

z /. Solve[z^2 + (-6 + 2I) z + 11 - 2I == 0] {2 + I, 4 - 3 I}

Siendo z2 la raíz del 1er cuadrante: 1 24 3 ; 2z i z i

ii) Comenzamos dibujando los puntos z1 y z2 y los posibles hexágonos en sus posiciones aproximadas (si lo hago a mano alzada) de acuerdo con los valores z1, z2 obtenidos.

-4 -2 0 2 4 6 8 10

-6

-4

-2

0

2

4

z1

z2

z3

/3

Figura 10. Puntos z1, z2 en su posición, y posibles hexágonos regulares.

Hay dos hexágonos regulares con un lado en el segmento z1-z2: uno a su derecha / hacia arri-ba y otro a su izquierda / hacia abajo. El vértice contiguo a z2 de mayor componente imagina-ria será el del hexágono que está a la derecha / arriba, según se muestra en la figura.

Podemos resolver esto de varias maneras. Viene bien saber que un hexágono regular se puede formar con seis triángulos equiláteros unidos por un vértice común central, y conocer los án-gulos en un triángulo equilátero. Pero el razonamiento que voy a seguir es más general, por-que servirá para otros polígonos regulares.

12

Page 31: Capítulo 1: Números Complejos

Considérese el vector3 z2 − z1, es decir, el segmento orientado que va desde z1 hasta z2. ¿Qué

ángulo habrá que girarlo hacia la derecha para llegar a la dirección del siguiente lado, la del segmento z3

− z2? Un sexto de vuelta completa. Porque si repetimos ese mismo giro a lo largo de lados sucesivos, tras dar la vuelta completa al hexágono volveremos a llegar al lado inicial z2

− z1. Eso son 2 /6 = /3 radianes por giro, como se muestra en la figura. (De análoga ma-nera en un heptágono regular, por ejemplo, siete giros de 2 /7 radianes darán la vuelta com-pleta, con lo que de un lado al siguiente/anterior habrá que girar un ángulo de ±2 /7.)

A partir de aquí es sencillo:

2 1 (2 ) (4 3 ) 2 4z z i i i

3 2 2 1 3

1 3( ) 1 2 4 1 2 3 2

2 2z z z z i i i

3

3 2 3 2 3( ) 2 1 2 3 (2 3) 1 2 3 3 3z z z z i i z i

Ejercicio 10 (de examen de noviembre de 2012):

i) Hallar las raíces z1, z2 de la ecuación z2 − (3 − 4i) = 0, con z1 la situada en el cuarto cuadrante.

Resol.: Buscamos los valores de z que cumplan z2 = 3 − 4i con lo que por definición estamos buscando las dos raíces cuadradas de 3 − 4i. Esta vez hagá-moslo en polares para variar (aunque haya que usar las fórmulas del seno y el coseno del án-gulo mitad). El módulo de z2 = 3 − 4i es 5 (triángulo pitagórico). Llamemos a su argumento = −atan(4/3) = −acos(3/5). O sea, llamemos z2 = 5

. Entonces (siempre en 4º cuadrante):

12

2 2 1 cos 1 cos5 5 5 cos sin 5

2 2 2 2z i

i

1 3 5 1 3 5 8 2

5 5 42 2 10 10

i i

1 2i i

comprobándose que efectivamente z12 = (2 − i)2 = 4 − 1 − 4i = 3 − 4i y además z1 está en el 4º

cuadrante. La otra raíz será z2 = −z1, con lo que:

1 22 ; 2z i z i

ii) Hallar los números complejos y sabiendo que 1

arg( ) 2

, , y el

cociente

Re( ) 3

Im( ) 0

1(3 2 )

13i

.

Resol.: En realidad arg() tendría que ser un conjunto de infinitos valores separados por múltiplos de 2, pero nos entendemos. Con ese argumento, es un número real positivo, y con módulo 1 solo puede ser: = 1 Por otra parte: = 3 + iy

3 Los números complejos, además de estructura de cuerpo, tienen estructura de espacio vectorial, con lo que no es necesario decir “vector asociado al número complejo z2

− z1”: son vectores en sentido estricto.

13

Page 32: Capítulo 1: Números Complejos

Sustituyendo: 1 1

(3 2 ) 13 (3 )(3 2 ) 9 2 (3 6)3 13

i iy i y iiy

y

9 2 13 2

3 6 0 2

y y

y y

Los datos del problema eran bastante redundantes (hemos obtenido y = 2 dos veces, y además ha salido mayor que cero, que era otro dato innecesario), pero al menos eran compatibles.

La solución es: 1; 3 2i

iii) Sean z3 = z1 + y z4 = z2

+ los afijos de los vértices de la diagonal menor de un rombo. Hallar los afijos de los otros dos vértices, sabiendo que la longitud de la diagonal mayor es el doble de la de la diagonal menor.

Resol.: Si he hecho bien los ejercicios anteriores:

3 1 4 22 1 3 ; 2 3 2 1 3z z i i z z i i i

Esto yo lo voy a poder pensando en un rombo “abstracto” cuyos vértices opuestos z3, z4 no tengo por qué dibujar, y mucho menos en su sitio. Generar la figura con el ordenador a mí me lleva un rato, pero tú, con un lápiz y un papel, puedes a mano alzada hacer en 10 segundos (literalmente!) un esquema que te oriente bastante.

Punto medio del rombo: 3 4 3 1 32

2 2

z z i im i

i

i

Una semidiagonal centrífuga: 4 4 1 3 (2 ) 1 2s z m i i

Semidiagonales perpendiculares y de longitud doble:

5 4 6 5( ) 2 2 ( 1 2 ) 4 2 ; 4 2s i s i i i s s

Vértices que faltan: 5 5 6 62 4 2 2 ; 2 4 2 6z m s i i i z m s i i 3i

Todo junto: 3 4 5 63 ; 1 3 ; 2 ; 6 3z i z i z i z i

Ejercicio 11 (de examen de julio de 2012):

i) Hallar dos números complejos y sabiendo que es la raíz imaginaria pura de la

ecuación z3 – 8i = 0 y que 13

.

Resol.: 3 3 2 232 6 5 6 9 6 6 5 6

38 8 8 2 ,2 ,2 2 ,2 , 2 ; 2kz i i

i

2

0

; 2 ( 2)2

xx iy x iy i x i y x i

y

2 213 4 ; 13 4 9 3 3 2 ; 2x x i i

ii) Hallar otro número complejo γ tal que Re( ) 0

14

Page 33: Capítulo 1: Números Complejos

Debido a la última condición, α, β, γ son los vértices de un triángulo equilátero en el plano complejo. Si haces un dibujo rápido a mano alzada con las posiciones de α y β obteni-das en el apartado anterior, verás de inmediato cuál de los dos posibles triángulos con un lado sobre α-β tiene el otro vértice γ en el semiplano complejo derecho.

Vector desde α hasta β: 2 ( 3 2 ) 3 4i i i

Lo giro +π /3: 3

3(3 4 ) 1 2 3

2i

1 3 3(3 4 ) 2 3

2 2 2i i i

Lo sumo a α: 3 3 3 4 3

3 2 2 3 2 32 2 2

i i i

3 3

2

Ejercicio 12 (de examen de enero de 2012):

i) La ecuación z3 − (4 + 4i) z2 − (2 − 11i) z + (7 − i) = 0 tiene tres raíces complejas: z0, z1 y z2. Una de ellas, z0, es imaginaria pura y también es raíz de la ecuación z3 + i = 0. Hallar las otras dos raíces, z1 y z2.

Resol.: 3 33 2 23 72 03 6 6

1 , ,10 1 1 k iz i z i z

; i

(y efectivamente i3 + i = 0). Para dividir el polinomio original, que voy a llamar p(z), por (z − z0), uso una tabla de Ruffini:

1 4 4 2 11 7

3 4 7

1 4 3 1 7

i i

i i i

i i 0

i

i

Efectivamente z0 = i es raíz de p(z), pues en la esquina inferior derecha sale 0 (donde siempre

aparece el polinomio evaluado en el número de la izquierda de la tabla, tanto si su valor es 0 como si no). Obtengo q(z) = p(z) / (z − i) de la última fila de la tabla y escribo:

p(z) = z3 − (4 + 4i) z2 − (2 − 11i) z + (7 − i) = [z2 − (4 + 3i) z + (1 + 7i)] (z − i)

Las otras dos raíces son las del polinomio entre corchetes, que es q(z):

24 3 (4 3 ) 4(1 7 ) 4 3 16 9 24 4 28 4 3 3 4

2 2

i i i i i i iz

2

i

2 2

2 2 2 33 4 ( ) 2

2 4 2

a bi a bi a b i ab

ab b a

2

2 2 2 2 2 42 33 ( ) 3 4 0

12a a a a

a

9 16

2 1 3 4 (a a b i 2 )i

Sustituyendo: 1,2

4 3 (2 )3 ,1 2

2

i iz z

i i

ii) Sean z1 = 3 + i y z2 = 1 + 2i dos números complejos. Hallar otro número comple-

jo z3 tal que 3 2 3 1 2 1

5

2z z z z z z y Im(z3) > Im(z2).

15

Page 34: Capítulo 1: Números Complejos

Resol.: Los puntos z1, z2 y z3 formarán un triángulo isósceles cuyos lados tendrán estas lon-gitudes:

2 22 1 3 2 3 1 2 1

5 5(1 3) (2 1) 5 ; 5

2 2z z z z z z z z

5

2

Con estos datos podemos dibujar (si lo hacemos a mano alzada, aproximadamente) z1, z2 y las posibles posiciones de z3, de las cuales la válida es la de arriba (la marcada en la figura) por-que nos dicen que Im(z3) > Im(z2).

0 1 2 3 4-1

0

1

2

3

4

z1

z2

z3

m51/2/2

5/2

51/2

Figura 11. Puntos z1, z2 en su posición, y posibles triángulos.

El punto medio m de z1-z2 es: 1 2 3 1 2 32

2 2

z z i im i

2

Vamos a fijarnos en el triángulo rectángulo de arriba a la izquierda. Su cateto corto tiene por longitud la mitad de | z2

− z1 |, es decir, 51/2 / 2, lo cual marcamos en la figura. Su hipotenusa

tiene por longitud | z3 − z2

|, es decir, 5/2, lo cual también marcamos en la figura. Por el Teo-rema de Pitágoras, su otro cateto tendrá por longitud:

22

3

5 5 25 5 205

2 2 4 4 4z m

lo que marcamos también en la figura.

Una manera sencilla de llegar al punto z3 es coger el vector z1 − z2, girarlo un ángulo recto

hacia la izquierda (multiplicándolo por i), multiplicar por el cociente de módulos | z3

− m | / | z1 − z2

| (que casualmente es 1), y sumar ese vector al punto medio m:

Giro y escala:

1 2 3 (1 2 ) 2z z i i i

3

1 2

5(2 ) (1 2 ) 1 2

5

z mi i i i

z z

Finalmente: 3 3 3

3 7( ) 2 1 2 3

2 2z m z m i i z i

Este modo de resolver ha tenido la ventaja de que el único giro es de /2. Por ejemplo el án-

gulo es atan(2), algo mayor que π/3; se puede usar sin saber su valor en radianes o gra-

dos, solo su seno y su coseno, pero manejar ángulos rectos es algo más sencillo.

1 2 3z z z

16

Page 35: Capítulo 1: Números Complejos

Ejercicio 13 (de examen de noviembre de 2011):

Sea 3 1

2 2z i . Se pide:

1) Obtener en forma binómica: i) z1 = z5 ii) z2 = z9

2) Hallar el número complejo z3 (el de mayor componente imaginaria) tal que los afijos de z1, z2 y z3 formen un triángulo rectángulo isósceles, sabiendo que los vértices de la hipotenusa son los afijos de z1 y z2.

Resol.:

1-i) 5

5

71 35 36

66 6 6 6

3 1 31 1 1 1 cos sin2 2 6 6 2 2

iz i i z

1

1-ii) 997

2 7 21 20 296

6 2 2 2 2

1 1 1 1 1z z

i

2) Con las coordenadas de z1 y z2 ya podemos dibujar algo muy parecido al triángulo final:

-0.25 0 0.25 0.5 0.75 1 1.25 1.5-0.75

-0.5

-0.25

0

0.25

0.5

0.75

1

1.25

z1

z2

z3

m

Figura 12. Puntos z1, z2 en su posición, y triángulo rectángulo isósceles.

Voy a calcular el punto medio m del segmento z1-z2, girar el vector z1 − m un ángulo recto

hacia la izquierda, y sumar ese vector a m para obtener z3:

Punto medio: 1 2 1 3 3

2 2 2 2 4

z z i im i

4

Vector z1 − m y giro: 1

3 3 3 3 3

2 2 4 4 4 4 4 4

i i iz m i i i i

3

Punto z3: 3 3

3 3 3 3 3 3 3 1

4 4 4 4 4 4 4 4

iz m i i z i

3

17

Page 36: Capítulo 1: Números Complejos

Ejercicio 14 (de examen de mayo de 2011):

Se consideran los números complejos 5

2

iA

i

y

7

2 22

2 2B i

.

Y sea M = A∙B el vértice de un triángulo equilátero MNP inscrito en una circunferencia cen-trada en el origen. Hallar los afijos de los tres vértices del triángulo.

Resol.: 2 2

5 2 5 101 2

2 2 2 1

i i iA i

i i

773

2154

4 4 4

2 212 2 1 2 1 2 ( 1) 1 22 2

1B i i

(1 2 )( 1 ) 1 2 ( 2 1) 1 3M AB i i i M i

Siendo el triángulo equilátero y centrado en z = 0 (circuncentro, o baricentro o incentro), los otros dos vértices serán M girado ±2 /3:

2

3

2 2 1 3 1 3 3 31 (1 3 ) cos sin (1 3 )

3 3 2 2 2 2M i i i i N i

3

2

3

2 2 1 3 1 3 3 31 (1 3 ) cos sin (1 3 )

3 3 2 2 2 2M i i i i P i

3

No voy a hacer la figura porque el razonamiento anterior no la necesita.

Ejercicio 15 (de examen de enero de 2011):

Sea 1 3z i una raíz de la ecuación z3 − z0 = 0. Se pide:

i) Hallar las otras dos raíces z2 y z3 de la ecuación anterior.

ii) Si los afijos de z2 y z3 son los vértices de la diagonal menor de un rombo, hallar los afi-jos de los otros dos vértices de dicho rombo, sabiendo que la relación entre la diagonal menor y la diagonal mayor es 1/2.

Resol.: i) z1, z2, z3 son las tres raíces cúbicas de algo (concretamente de z0), así que me basta girar ±1/3 de vuelta la raíz proporcionada para obtener las otras dos sin tener siquiera que calcular z0:

2 1 2

3

1 3 1 31 3 0

2 2 2 2z z i i i

2i

3 2 2

3

1 31 2 3

2 2z z i i

i

Si no me doy cuenta de lo anterior también es fácil calcular z0 y sus tres raíces cúbicas:

3 3 3 30 0 5 6 15 6 2 3 6 23 0 3 2 2 8 8i z z i 8i

(o si no operando en forma binómica también sale fácil). Por tanto la ecuación es z3 − 8i = 0, y

18

Page 37: Capítulo 1: Números Complejos

z1, z2, z3 son las tres raíces cúbicas de 8i:

3

3 3 2 232 5 3 1

3 , ,6 6 6

2

3

8 8 8 2 3

2

k

z i

i z

z i

i

Ahora sin dibujar nada puedo pensar en un rombo “abstracto” con diagonal menor z3 − z2. Su

punto medio será: 2 3 2 3 3

2 2 2

z z i im i

1

2

Una semidiagonal menor: 2 2

3 1 3 32

2 2 2s z m i i i

2

Llamo semidiagonal mayor s4 a la semidiagonal menor s2 girada π /2, es decir, multiplicada por i, y el doble de larga, es decir, multiplicada por 2:

4 2

3 32 2 3 3

2 2s s i i i i

Sumándola y restándola al centro del rombo obtengo los dos vértices pedidos:

4

4,5 4

5

3 13 3

2 23 13 3

2 2 3 13 3

2 2

z i

z m s i i

z i

Ejercicio 16 (de examen de enero de 2011):

Hallar analítica y gráficamente el lugar geométrico de los números complejos α en los si-guientes casos:

a) z

z b)

2

z

z c)

3z

z

cuando z recorre la semirrecta 3

( 33

y x x 2) .

a) Prefiero empezar gráficamente, porque es más intuitivo.

La figura muestra el lugar geométrico de puntos z (semirrecta de pendiente 31/2/3, o ángulo π /6 radianes con la horizontal, a partir del punto 31/2/2 + i /2). El conjugado de cualquiera de esos puntos z, marcado z* en la figura, tendrá igual módulo y argumento opuesto. Al dividir, el cociente de módulos iguales dará 1, mientras que la resta de argumentos dará −π /3. Por tanto, el lugar geométrico de α es un solo punto.

19

Page 38: Capítulo 1: Números Complejos

x

y

/6

31/2/2

1/2

z

z*

-/3

1

1

Figura 13. Lugar geométrico de α en el apartado a).

Analíticamente, llamando z = ρπ/6 (con ρ ≥ 1, pero esto aquí da igual):

*

6

6 36 6

11

2 3

zi

z

3 (un punto)

b) El caso α = z* / z2 es parecido, pero en lugar de dividir por z, dividimos por z2. Esto, por una parte, hace que el argumento de α siempre sea −π /6 − 2(π /6) = −π /2, con lo cual esta-remos dentro del semieje imaginario negativo; pero además el módulo de α será ρ / ρ2 = 1 / ρ; cuando ρ = 1, también 1 / ρ = 1; pero a medida que ρ crece por la semirrecta, tendiendo a +∞, 1 / ρ decrece, tendiendo a cero, valor que nunca se alcanzará (salvo que consideremos el plano complejo extendido, cosa que no haremos). Ver figura.

x

y

/631/2/2

1/2

z

z*

1

- i

Figura 14. Lugar geométrico de α en el apartado b).

Analíticamente: *

6

2 2 22 6 6 2 6 2

1con [1, )

z

z

Es decir: 2 con (0,1]

con [ 1,0)

r r

yi y

(intervalo del eje imaginario)

c) El caso α = z3 / z* es parecido. El argumento de α será el cuádruple del de z, o sea, 4π/6 = 2π/3; y su módulo, el cuadrado del de z. Cuando éste va de 1 a +∞, aquel también lo hace. Por tanto el lugar geométrico de α será la semirrecta de argumento 2π/3 y módulos des-de 1 (incluido) a +∞. Ver figura.

20

Page 39: Capítulo 1: Números Complejos

Figura 15. Lugar geométrico de α en el apartado c).

Llamando z = ρθ: 33

3 6 24*

6 6

con [1, )z

z

y por tanto: 2 3 con [1, )

3 con 1 2

r r

y x x

(semirrecta del plano complejo)

Un lugar geométrico es el conjunto de todos los puntos que cumplen ciertas condiciones ge-ométricas. En el apartado a) es importante dejar claro que ese lugar geométrico es un punto (y cuál); en el b) que es un intervalo (y cuál); y en el c) que es una semirrecta (y cuál).

Ejercicio 17 (de examen de septiembre de 2010):

i) Hallar las raíces z1, z2 y z3 de la ecuación 3(1 ) 8 2 0i z i siendo z1 la raíz si-tuada en el primer cuadrante.

ii) Dibujar el cuadrado circunscrito a la circunferencia que pasa por z1, z2 y z3 y es tangen-te en z1 a dicha circunferencia.

iii) Hallar en forma binómica el afijo del vértice de mayor componente imaginaria del cua-drado anterior.

Resol.: i) 3 34

8 2 1 8 2(1 )(1 ) 8 2 0 4 2(1 ) 8

1 1 2

i i ii z i z i

i i

1 12

3 4 234 8 9 7 2

3 , ,12 12 12 12

3 7 1

2

8 8 2 2 2

2

kk

z

z z

z

3 4

2

Como no me las piden en forma binómica, no voy a perder tiempo en calcular senos y cose-nos de π/12 y de −7π/12, que el tiempo no me sobra. Ya los calcularé si los necesito.

ii) A mano alzada (y no se pide más) puedo ya dibujar algo muy parecido a esto sin hacer un solo cálculo:

21

Page 40: Capítulo 1: Números Complejos

Figura 16. Dibujo pedido en el apartado ii).

iii) El vértice v1 tiene argumento 12 4 3

(el de z1 más 1/8 de vuelta completa, por si-

metría del cuadrado) y módulo 2 2 (aplicando el Teorema de Pitágoras al triángulo 1 10z v ,

que es rectángulo en z1): 1 12

1 32 2 2 2 2 6

2 2v i v

i

Otra manera de obtener v1 podría ser calcular las coordenadas binómicas de z1 (para lo cual el seno y el coseno de π/12 los obtenemos por ejemplo como los de 3π/4 − 2π/3, tal como sugiere la figura), multiplicar z1 por i para girar su vector de posición un ángulo recto hacia la iz-quierda, y sumar este último vector a z1.

Ejercicio 18 (de examen de enero de 2010):

Una raíz de la ecuación z4 − z0 = 0 es 1 3 1 3 1z i . Se pide:

i. Módulo y argumento de z1. Dato: 3 1

tan12 3 1

.

ii. Hallar z0 en forma binómica.

iii. Hallar las demás raíces de la ecuación.

iv. Si los afijos de las raíces situadas en el tercer y cuarto cuadrantes son los vértices A y B del lado desigual de un triángulo isósceles, hallar el otro vértice C (el de menor componente imaginaria del triángulo) sabiendo que la longitud de la altura correspondiente al vértice C es igual a la longitud del lado AB.

Resol.: i.) 2 21 ( 3 1) ( 3 1) 3 1 2 3 3 1 2 3 8 2 2z

1 1(dato)

3 1Arg( ) ArcTan ; 2 2 ; Arg( )

12 123 1z z 1z

ii.) 4 44

0 1 41212 3

1 32 2 2 2 64 64 cos sin 64

3 3 2 2z z i i

0 32 32 3z i

iii.) Las raíces de z4 − z0 = 0 son las cuatro raíces cuartas de z0. Una la conocemos (es z1, dato) y las otras tres tendrán el mismo módulo y argumentos separados por múltiplos de

22

Page 41: Capítulo 1: Números Complejos

2π/4 = π/2, es decir, z0 multiplicada por i, −1 y −i:

2 1 2

3 1 3

4 2 4

( 3 1) ( 3 1) 3 1 ( 3 1)

3 1 ( 3 1)

3 1 ( 3 1)

z z i i i z i

z z z i

z z z i

iv.) A mano alzada puedo ya dibujar algo muy parecido a esto sin hacer un solo cálculo:

Figura 17. Triángulo del apartado iv.

Bastará calcular el punto medio m del lado AB, girar el vector BA = z3 − z4 un ángulo recto

hacia la derecha (multiplicándolo por i) y sumárselo a m:

3 4 3 1 ( 3 1) 3 1 ( 3 1) 2 2 31 3

2 2 2

z z i i im i

3 4 3 1 ( 3 1) 3 1 ( 3 1) 2 3 2BA z z i i i

( ) ( 2 3 2 ) 1 3 2 2 3 3 3 3C m C m m i i i i C i

Ejercicio 19 (de examen de septiembre de 2009):

Sean z1 y z2 dos números complejos tales que su cociente está situado en el eje imaginario positivo, su suma es igual a 10 + 10i, y el módulo de uno de ellos es el triple que el del otro. Se pide:

i) Hallar z1 y z2.

ii) Si z1 y z2 son los vértices opuestos de un cuadrado, hallar los otros dos vértices z3 y z4.

Resol.: No nos dicen si es el cociente z1 / z2 o el cociente z2

/ z1 el que está situado sobre el eje imaginario positivo. Usaré el primero, es decir, Arg(z1

/ z2) = π/2, sin pérdida de gene-ralidad, pues solo es elegir los nombres de las incógnitas. Si tú eliges que sea z2

/ z1 el que esté en el eje imaginario positivo, puedes resolver el ejercicio igual que yo pero llamando z1 a lo que yo llamo z2 y viceversa.

La 1ª condición la escribo así: 1z k i z2 con k > 0

pues multiplicar por un imaginario puro positivo añade π/2 al argumento. (Recuérdese que simplemente escribir k > 0 ya indica que k es real, porque el conjunto de los números comple-jos no es ordenado. Y respecto al caso k = 0, puede descartarse porque, aunque podría aceptar-se que z1

= 0 tiene cualquier argumento, entonces z2 sería 10 + 10i, y su módulo no sería el triple del de z1, que es 0.)

Para la segunda condición, z1 + z2 = 10 + 10i, da igual cuál sea z1 y cuál sea z2, porque la

suma es conmutativa.

23

Page 42: Capítulo 1: Números Complejos

Y para la tercera condición tampoco nos dicen si z1 debe tener triple módulo que z2 o al revés. Además ahora ya no puedo elegir arbitrariamente, porque ya he decidido cuál es para mí z1 y cuál es z2 (haciendo que Arg(z1

/ z2) = π/2). Por tanto ya no estoy simplemente eligiendo nom-bres de incógnitas: usar | z1

| / | z2 | = 3 me dará una solución o conjunto de soluciones, y usar

| z2 | / | z1

| = 3 me dará otro. (Otra cosa es si luego ambos resultan ser iguales o uno o ambos resultan ser vacíos—sistema incompatible—, aunque no lo creo.)

Llamaré 1 1 1 2 2;z x iy z x iy 2

1 2 2

2 2 2

3 3z kiz ki z

kz z z

Caso | z1

| / | z2 | = 3:

1 2 2 2 2 23 3 ( ) 3 3z kiz iz i x iy y i x 2

2 21 2 2 2 2 2

2 2

3 103 3 ( ) 10 10

3 10

x yz z y i x x iy i

x y

22 2 2 2 2

2

4( 3 ) 3(3 ) 10 10 30

10 12 2

xx y x y x

y

1 2 1 23 3 (4 2 ) 6 12 ; 4 2z iz i i z i z i 1ª solución:

Compruebo que se cumplen todas las condiciones (ante todo las buenas costumbres!): efecti-vamente z1 = 3i z2, lo que garantiza el cumplimiento de las condiciones sobre el cociente de números y de módulos. Y que suman 10 + 10 i es de comprobación inmediata.

Caso | z2 | / | z1

| = 3: 2 2 2

1 2 2

1 13 3

3

z z zk

z kiz ki z k

1 2 2 2 1 1 1 1

13 3 ( ) 3 3

3z kiz iz z iz i x iy y i x 1

1 11 2 1 1 1 1

1 1

3 10(3 3 ) 10 10

3 10

x yz z x iy y i x i

x y

11 1 1 1 1

1

43( 3 ) ( 3 ) 10 30 10 ;

10 12 2

yx y x y y

x

2 1 1 23 3 ( 2 4 ) 2 4 ; 12z iz i i z i z 6i 2ª solución:

De nuevo comprobamos que (1/3) i z2 = (1/3) i (12 + 6i) = −2 + 4i = z1, con lo que se cum-plen las condiciones sobre el cociente de números y de módulos, y además 10 + 10 i.

Las representaciones geométricas de la figura muestran que en ambas soluciones z1 / z2 es

imaginario puro positivo (por el ángulo recto en el origen), sus módulos son triple uno del otro, y el punto medio de z1 y z2 es 5 + 5i (lo que equivale a que sumen 10 + 10i).

24

Page 43: Capítulo 1: Números Complejos

Figura 18. Las dos soluciones al apartado i).

ii) Para ambas soluciones del apartado anterior lo más sencillo es probablemente calcular una semidiagonal centrífuga v = z1

− m, multiplicarla por ±i para girarla ±π/2 y obtener la semidiagonal centrífuga perpendicular vp, y sumársela a m:

1ª solución:

1 (6 12 ) (5 5 ) 1 7 ; (1 7 ) 7pv z m i i i v i i i

3 4(5 5 ) ( 7 ) 2 6 ; (5 5 ) ( 7 ) 12 4p pm v i i z i m v i i z i

2ª solución:

1 ( 2 4 ) (5 5 ) 7 ; ( 7 ) 1 7pv z m i i i v i i i

3 4(5 5 ) (1 7 ) 6 2 ; (5 5 ) (1 7 ) 4 12p pm v i i z i m v i i z i

La figura muestra las correspondientes representaciones gráficas (no pedidas).

Figura 19. Las dos soluciones al apartado ii).

Ejercicio 20 (de examen de junio de 2009):

Sea z1 la raíz de la ecuación z2 − i z − 1 − i = 0 de menor componente imaginaria, y sea z2 la otra raíz. Sea C la circunferencia de centro z1 que pasa por z2, y sea T el triángulo equilátero circunscrito a C que es tangente a C en z2. Hallar el vértice de mayor componente imaginaria de T.

Resol.: 1 4(1 )

2

i iz

Antes de seguir a la manera habitual, y espoleado por la pereza, me acabo de dar cuenta de que hay una raíz muy sencilla, que es −1, así que voy a obtener la otra con una tabla de Ruffi-

25

Page 44: Capítulo 1: Números Complejos

ni:

1 1

1 1

1 1

i i

i

i

1

0

Por tanto (z + 1) (z − 1 − i) = 0 y 1 21; 1z z i

Ya puedo hacer a mano un dibujo muy parecido al de la figura para guiar las operaciones.

Figura 20. Circunferencia y triángulo circunscrito tangente en z2.

Para obtener el vértice superior z3 conviene saber que en un triángulo equilátero el triángulo

rectángulo 1 2 3z z z tiene ángulos de 30º y 60º (es como un cartabón). Por tanto:

3 23 2 2 1 3

2 1

3 3( ) 1 3(2 ) 1 3 (1 2 3)z z

z z i z z i i i z iz z

tan

3

También puedo usar que la circunferencia inscrita C corta al segmento z1-z3 en su punto me-dio, y que el ángulo desde el segmento z1-z2 hasta z2-z3 es 2 /6 = /3:

32 1 2 1

1 32( ) 1 2(2 ) 1 2 3 1 2 3

2 2

iz z z z e i i i

con el mismo resultado.

Ejercicio 21 (de examen de junio de 2006):

Sea z = x + i y un número complejo no nulo, y sea z z . Se pide:

i) Expresar el módulo y el argumento de en función de z y de x e y.

ii) Indicar gráficamente el lugar geométrico que describe el afijo de cuando el afijo de z recorre el arco, situado en el primer cuadrante, de una circunferencia de radio R = 1 y centro el origen de coordenadas.

iii) Si el radio de la circunferencia del apartado anterior fuera R = 3, indicar razonadamente cuál sería la respuesta en este caso.

26

Page 45: Capítulo 1: Números Complejos

Resol.: i) 1z zz

z z z

1arg( ) arg arg arg( ) arg( ) arg( ) 2arg( ) ;

arg( ) 2arg( )

zz z z z z

z z

arg( ) 2arg( ) 2arctan ; 1; arg( ) 2arctany y

zx x

ii) El módulo de es 1, luego se encontrará en una circunferencia centrada en el origen y de radio 1. Y su argumento será menos el doble del de z, que va de 0 a /2, y por tanto irá desde 0 hasta −. Por tanto el lugar geométrico descrito por será la semicircunferencia de centro el origen y radio 1 situada en el semiplano complejo inferior, como muestra la figura.

xy

r = 1

Figura 21. Lugar geométrico descrito por (apartados ii e iii).

iii) Sería lo mismo porque ni el módulo ni el argumento de dependen del módulo de z.

Ejercicio 22 (de examen de febrero de 2006):

Los afijos de las raíces z1 y z2 de la ecuación z2 − (3 + 4i) z + (−1 + 7i) = 0 son el centro y el punto medio del lado AB de un hexágono regular, respectivamente. Hallar los afijos de los vértices A y B sabiendo que 1 2Im Imz z .

Resol.:

23 4 (3 4 ) 4( 1 7 ) 3 4 9 16 24 4 28 3 4 3 4

2 2

i i i i i i iz

2

i

2 2

2 2 2 33 4 ( ) 2

2 4 2

a bi a bi a b i ab

ab b a

2

2 2 2 2 2 12 33 ( ) 3 4 0

42a a a a

a

9 16

1 2 3 4 (1a a b i 2 )i

Sustituyendo: 1

2

23 4 (1 2 )

1 32

z ii iz

z i

Como no me fío de mis operaciones, lo compruebo con Mathematica:

Solve[z^2 − (3 + 4I) z + (−1 + 7I) == 0] {{z -> 1 + 3 I}, {z -> 2 + I}}

Y si estuviera en un examen lo comprobaría muy fácilmente con una rápida tabla de Ruffini:

27

Page 46: Capítulo 1: Números Complejos

1 3 4 1 7

2 2 2 3 ( 1

1 1 3 0

i i

i i i

i

6)

donde se ve (sabiendo leer) que ambas raíces son correctas.

El centro y el punto medio del lado AB son la misma cosa… no, se debe referir al centro del hexágono y al punto medio de su lado. Con las coordenadas de z1 y de z2 ya puedo dibujar a mano alzada algo muy parecido a la Fig. 22, para guiar las operaciones.

Figura 22. Hexágono del ejercicio.

En un polígono regular se llama radio r a la distancia del centro a cualquier vértice (o radio de la circunferencia circunscrita), y apotema a a la distancia del centro a cualquier lado (o radio de la circunferencia inscrita). En un hexágono regular el radio y el lado son de igual longitud (el hexágono se puede formar con seis triángulos equiláteros unidos en el centro del hexágo-no), y la apotema y el radio cumplen a = r cos( /6) (ver figura). Por tanto para obtener A y B bastará sumar o restar a z2, respectivamente, la apotema a = z2

− z1 dividida por cos( /6) (pa-ra obtener la longitud de un lado), multiplicada por −i (para girarla /2 a la derecha), y divi-dida por 2 (medio lado):

2 1 1 3 (2 ) 1 2 3( ) (2 )

cos( 6) 2 2 33 2 3

z z i i i i ii i

2

3 3 2 3(2 ) 1 3 (2 ) 1 3

3 3 3A z i i i A i

3

3

2

3 3 2 3(2 ) 1 3 (2 ) 1 3

3 3 3B z i i i B i

3

3

Hay otras maneras de hacer lo mismo.

Ejercicio 23 (de examen de febrero de 2005):

Hallar tres números complejos z1, z2 y z3 cuyos afijos equidisten entre sí, y del afijo de otro número complejo z0, sabiendo que z0 y z1 son respectivamente las soluciones de las ecuacio-

nes z3 − 8i = 0 y 2 (2 3 ) (2 2 3 ) 0z i z i situadas en el primer cuadrante.

28

Page 47: Capítulo 1: Números Complejos

Resol.: 3 3 2 230 2 6, , 0 0

3

3 18 8 8 2 2 3

2 2kz i z i z

i

Para la 2ª ecuación me puedo ahorrar bastantes operaciones si encuentro una raíz sencilla usando una tabla de Ruffini (de lo contrario tendría que hacerlo como en problemas anterio-res). Probando con ±1 inmediatamente se ve que no son raíces. Probando con i:

1 2 3 2 2 3

2 3

1 2 3 0

i i

i i

i

con lo que i no es raíz, pero parecemos acercarnos bastante. Probando con −i:

1 2 3 2 2 3

2 2 3

1 2 3 2 0

i i

i i

i

i

Por tanto ( ) (2 3 2 )z i z i 0

(y desarrollando se comprueba fácilmente que está bien). De las dos raíces, la del primer cua-

drante es la segunda: 1 2 3 2z i

Leyendo el enunciado interpreto que z1, z2 y z3 son los vértices de un triángulo equilátero (“equidistan entre sí”) cuyo centro es z0 (“los tres equidistan de z0”). Por tanto, con las coor-denadas de z0 y z1, ya puedo dibujar a mano alzada algo muy parecido al triángulo pedido en su verdadera posición. Sin embargo la geometría es tan sencilla que no lo necesito; me basta razonar sobre un triángulo equilátero “abstracto”. Voy a calcular el vector centrífugo v1 = z1

− z0 y, girándolo ±2 /3 y sumándolos al centro z0, obtener los otros dos vértices pedi-

dos: 1 1 0 2 3 2 ( 3 ) 3v z z i i i

2

32 1 2 0 2 2

2

33 1 3 0 3 3

1 33 3 ; 3 3

2 2

1 33 0 2 ; 3 2

2 2

i

i

v v e i i i z z v i i z i

v v e i i i z z v i i z i

2

3

Ejercicio 24 (de examen de enero de 2004):

Se consideran la circunferencia de radio R, cuyo centro C es el afijo de la raíz de la ecuación 2 2(1 2 2 ) 0z i situada en el primer cuadrante, y el cuadrado de vértices A, B, D, E

inscrito en ella, siendo A el de mayor componente imaginaria, B el de mayor componente real y M = 3 + 2 i el punto medio del lado AB.

Hallar el número complejo cuyo afijo es el vértice de mayor componente imaginaria del cua-drado de lados paralelos al anterior e inscrito en una circunferencia concéntrica a la anterior y

de radio 2 2 R .

29

Page 48: Capítulo 1: Números Complejos

Resol.: 2 2

2 2 2 22

( ) 2 2(1 2 2 )2 4 2 2 2

a bz a bi a b i ab i

ab b a

2

2 2 2 2 2 42 2 2 4 322 ( ) 2 8 0

22a a a a

a

2 2 (2 2 ) 2a a b z i C 2 i

y compruebo que 2(2 2 ) 4 2 4 2 2 4 2i i i como tiene que ser.

Con estos datos ya puedo dibujar a mano alzada algo muy parecido a la Fig. 23. En realidad ni siquiera tengo que dibujar el cuadrado exterior; basta dibujar el segmento CA y otro CA' que sea 2·21/2 veces más largo.

Figura 23. Construcción geométrica del ejercicio.

A la vista de la figura lo siguiente es claro:

(3 2 ) (2 2 ) 1 (2 2)M C i i i

( ) (3 2 ) 1 (2 2) 1 2A M i M C i i i i 3

2 2( ) 2 2 1 2 3 (2 2 )A C A C C i i

2 2 2 2 1 2 (3 2) 2 2 4 2 2 (6 2 4)i i i i

6 2 2 (7 2 4)A i

Ejercicio 25 (de examen de enero de 2003):

Calcular el vértice de mayor componente imaginaria del triángulo circunscrito a la circunfe-

rencia que pasa por las raíces de la ecuación 3 2 20

2 2z i siendo los lados del triángulo

tangentes a la circunferencia en las raíces anteriores.

Resol.:

1 4

33 33 4 3 4 2 2 11 12

3

3 5 1

12 2

1 1 12 2

1

k

z

z i z

z

2

Esto me basta, sin calcular componentes binómicas, para poder hacer ya a mano alzada un dibujo muy aproximado (y luego hacer solo los cálculos que necesite).

30

Page 49: Capítulo 1: Números Complejos

Figura 24. Construcción geométrica del ejercicio.

Conociendo la geometría de un triángulo equilátero sé que V = −2z3, así que calcularé solo la forma binómica de este:

3 5 2

12 4 3

2 21 1 cos sin

4 3 4 3z i

2 2 2

cos cos sin sin sin cos cos sin4 3 4 3 4 3 4 3

i

2

2 1 2 3 2 1 2 3 6 2 6 2

2 2 2 2 2 2 2 2 4 4i i

3

2 6 6 22

2 2z V i

31

Page 50: Capítulo 1: Números Complejos

Capítulo 2: Transformada de Laplace. Resolución de ejercicios de fin de capítulo. AVISO IMPORTANTE: Antes de seguir lee este AVISO.

Resoluciones completas:

1.- Calcular la transformada de Laplace de las siguientes funciones:

a) ( ) cosh( )atf t e bt=

Resol.: Veo una función de transformada “inmediata” multiplicada por una exponencial. Esto pide aplicar la propiedad de desplazamiento (o traslación) en s:

[ ] 2 2 2 2cosh( ) cosh( )( )

ats s a

s s a

s s ae bt bt s a bs b s a b→ −

→ −

− = = = ∀ > + − − −L L

La validez es ∀s > a + |b| porque ese es el orden exponencial de f: el de eat es obviamente a, que se suma al del cosh, que es |b| porque es combinación lineal de dos exponenciales, una de orden b y otra −b, el mayor de los cuales es |b|.

Comprobación con Mathematica (que en adelante pondré sin más preámbulos): LaplaceTransform[Exp[a t] Cosh[b t], t, s]

(-a + s) / (-b^2 + (a - s)^2)

b) ( ) sinh( )atf t e bt=

Resol.: Casi igual que antes:

[ ] 2 2 2 2sinh( ) sinh( )( )

ats s a

s s a

b be bt bt s a bs b s a b→ −

→ −

= = = ∀ > + − − −L L

LaplaceTransform[Exp[a t] Sinh[b t], t, s] b / (-b^2 + (a - s)^2)

c) ( ) atf t e t=

Resol.: Usando la propiedad de traslación en s:

[ ] 2 2

1! 1( )

ats s a

s s a

e t t s as s a→ −

→ −

= = = ∀ > −L L

válido ∀s > a porque eat es de orden exponencial a (evidentemente); y t, como cualquier poli-nomio, es de orden b donde b puede ser un número estrictamente positivo tan pequeño como queramos (porque toda exponencial creciente, por despacio que empiece creciendo, termina creciendo más rápido que cualquier polinomio). De modo que si por ejemplo s = a + 0.001, como la función eat t es de orden exponencial a + 0.0005 (por ejemplo), la transformada es válida en ese valor de s. O sea: un polinomio no es de orden exponencial 0 (porque no está acotado) pero sí es de orden exponencial positivo tan pequeño como queramos; y multiplica-do por eat no es de orden exponencial a, pero si de orden exponencial mayor que a en tan poco como queramos.

También podemos usar la propiedad de derivada de la transformada, con igual resultado:

1 12

1 1( 1)( )

at at atd de t t e e s ads ds s a s a

− = = − = − = − ∀ > − − L L L

LaplaceTransform[Exp[a t] t, t, s] 1 / (a - s)^2

Page 51: Capítulo 1: Números Complejos

2

d) ( ) sin( )f t t at=

Resol.: La transformada será válida ∀s > 0, porque t es de orden exponencial positivo pero tan pequeño como queramos, mientras que sin(at) es de orden exponencial 0 (acotada en el campo real). Resolveré aplicando la propiedad de derivada de la transformada:

[ ] [ ]1 12 2sin( ) sin( ) ( 1) sin( )d d at at t at at

ds ds s a = = − = − = +

L L L

2 2 2 2 2 2

2 2 0( ) ( )

a s as ss a s a−

= − = ∀ >+ +

LaplaceTransform[t Sin[a t], t, s] (2 a s) / (a^2 + s^2)^2

e) ( ) cosh( )f t t at=

Resol.: La transformada será válida ∀s > |a| porque t es un polinomio (orden exponencial casi 0) y cosh(at) es de orden exponencial |a|. Aplicando derivada de la transformada:

[ ] [ ]12 2cosh( ) ( 1) cosh( )d d st at at

ds ds s a = − = − = −

L L

2 2 2 2

2 2 2 2 2 2

2( ) ( )

s a s s s a s as a s a− − +

= − = ∀ >− −

LaplaceTransform[t Cosh[a t], t, s] (a^2 + s^2) / (-a^2 + s^2)^2

f) f (t) = eat tn

Resol.: La transformada será válida ∀s > a porque eat es de orden exponencial a (de hecho es una exponencial de orden a) y tn es un polinomio.

Como la transformada de eat es “inmediata”, podemos aplicar la propiedad de derivada enési-ma de la transformada; y como la de tn también lo es, podemos aplicar la propiedad de trasla-ción en s. Empecemos haciéndolo de esta última manera, que es más sencilla:

1 1

! !( )

at n nn ns s a

s s a

n ne t t s as s a+ +→ −

→ −

= = = ∀ > −L L

Ahora hagámoslo aplicando derivadas de la transformada:

1( 1) ( 1)

n nn at n at n

n n

d dt e eds ds s a

= − = − − L L

Calculemos estas derivadas:

( )1 2( ) ( )d s a s ads

− −− = − −

( )2

1 3 2 32 ( ) 2( ) ( 1) 2( )d s a s a s a

ds− − −− = − − − = − −

( )3

1 2 4 3 43 ( ) ( 1) ( 3)2( ) ( 1) 3!( )d s a s a s a

ds− − −− = − − − = − −

( )1 1( ) ( 1) !( )n

n nn

d s a n s ads

− − −− = − −

Page 52: Capítulo 1: Números Complejos

3

La demostración formal de esta última expresión se puede hacer por inducción, pero cuando una ingeniera ve el patrón apodícticamente no pierde el tiempo en tonterías. Sustituyendo:

11

!( 1) ( 1) !( )( )

n at n n nn

nt e n s as a

− −+

= − − − = −L

igual que antes, como no podía ser de otra manera. Además vemos que particularizando para n = 1 se obtiene el resultado del apartado c). LaplaceTransform[Exp[a t] t^n, t, s] /. Gamma[n + 1] -> Factorial[n]

(-a + s)^(-1 - n) n!

2.- Utilizando la transformada de Laplace, hallar el valor de las siguientes integrales:

3 21 20 0

sin(2 ) ; sin( )t tI t e dt I t t e dt∞ ∞− −= =∫ ∫

Resol.: Se trata de transformadas de Laplace evaluadas en valores concretos de s.

La primera: [ ]31 2 230

3

2 2 2sin(2 ) sin(2 )2 9 4 13

ts

s

I t e dt ts

∞ −→

= = = = =+ +∫ L

lo cual es válido porque s = 3 > 0 = orden exponencial de sin(3t); de lo contrario, la integral divergiría.1

La segunda: [ ]22 20

sin( ) sin( )ts

I t t e dt t t∞ −

→= =∫ L

Por el apartado d) del ejercicio anterior:

2 2 2 22

2 4 4( 1) (4 1) 25s

sIs →

= = =+ +

lo cual es válido porque s = 2 > 0 = orden exponencial de sin(t); de lo contrario, la integral divergiría. Integrate[Sin[2 t] Exp[-3 t], {t, 0, Infinity}] (* sin usar Laplace *)

2 / 13 Integrate[t Sin[t] Exp[-2 t], {t, 0, Infinity}] (* sin usar Laplace *)

4 / 25

3.- Encontrar la transformada inversa de Laplace de las siguientes funciones:

a) 2

3( )4

F ss

=+

Resol.: Esto se parece tanto a la transformada de un seno que podemos considerarla una transformada inversa “inmediata”:

1 12 2 2

3 3 2 3( ) sin(2 ) ( 0)4 2 2 2

f t t ss s

− − = = = > + + L L

válido para s > 0 porque sin(2t) es acotada en ; de lo contrario su transformada de Laplace sería una integral divergente. InverseLaplaceTransform[3 / (s^2 + 4), s, t]

3/2 Sin[2 t]

1 Divergir (no diverger): verbo irregular que se conjuga como dirigir. Convergir (conjugado como los anteriores) y converger son ambos correctos, pero el segundo es más habitual. Converger es irregular y se conjuga como coger.

Page 53: Capítulo 1: Números Complejos

4

b) 3

4( )( 1)

F ss

=−

Resol.: Esto recuerda mucho a un desplazamiento en s, y lo que queda es casi la transfor-mada de tn con n = 2:

1 1 1 1 1 2 23 3 3 3

1

4 4 4 4 2!( ) 2 2 ( 1)( 1) 2!

t t t t

s s

f t e e e t e t ss s s s

− − − −

→ −

= = = = = = > − L L L L

válido para s > 1 porque f (t) “casi” es de orden exponencial 1. InverseLaplaceTransform[4 / (s - 1)^3, s, t]

2 E^t t^2

c) 2

2( )3 4

F ss s

=+ −

Resol.: Una posible manera es completar en el denominador el cuadrado perfecto de un bi-nomio para poder aplicar traslación en s y luego quede una transformada inversa casi inme-diata. El cuadrado del denominador se busca mirando primero al término en s2 y al doble pro-ducto:

3

1 1 1 122 2 2 2

3 2

2 2 2 23 4 ( 3 2) 25 4 25 4 25 4

t

s s

es s s s s

−− − − −

→ −−

= = = = + − + − − − L L L L

( )5 5

3 3 3 2 21 42 2 2

2

2 5 2 4 5 4 2sinh ( 1)5 2 25 4 5 2 5 2 5

t tt t t t te ee e t e e e s

s

−− − −

− − − = = = = − > − L

válido (y ya voy a dejar de explicar esto cada vez) para s > 1 porque f (t) es de orden exponen-cial 1.

De las dos expresiones recuadradas, ambas válidas y sencillas, creo que Mathematica por de-fecto dará la última. Veamos: InverseLaplaceTransform[2 / (s^2 + 3 s - 4), s, t]

2/5 E^(-4 t) (-1 + E^(5 t))

Pues no, nunca se sabe. (El “pensamiento” de Mathematica y, en general, de los sistemas de Computación Simbólica o Álgebra Computacional, es un poco misterioso, porque utilizan sus propios códigos de representación de las expresiones matemáticas.)

Otra manera es descomponer F(s) en fracciones simples y calcular la transformada inversa de cada una:

2 13 9 16 23 4 0 ; ( )42 ( 1)( 4) 1 4

A Bs s s F ss s s s

− ± ++ − = ⇒ = = = = + =− − + − +

2

0 2 54 2 5 2 5( )4 2 2 53 4 1 4A B AAs A Bs B F s

A B Bs s s s+ = = + + −

= ⇒ ⇒ ⇒ = − − = = −+ − − +

Voy a comprobar esto con Mathematica (no porque no me fíe de mis cálculos, cuyo resultado se puede comprobar fácilmente simplificando F(s), sino para recordarte cómo se hace): 2 / (s^2 + 3 s - 4) // Apart

2 / (5 (-1 + s)) - 2 / (5 (4 + s))

Lo que queda, aplicando la propiedad de linealidad, son transformadas inversas inmediatas; o también se puede aplicar traslación en s:

Page 54: Capítulo 1: Números Complejos

5

1 1 1 1 1

1 4

2 5 2 5 2 1 2 1 2 1 2 11 4 5 1 5 4 5 5s s s ss s s s s s

− − − − −

→ − → +

− = − = − = − + − + L L L L L

( )1 4 1 4 42 1 2 1 2 2 21 1 ( 1)5 5 5 5 5

t t t t t te e e e e e ss s

− − − − − = − = ⋅ − ⋅ = − > L L

igual que antes, como debe ser.

No cuál de las dos maneras mostradas te gusta más. La primera parece requerir menos opera-ciones (en este caso), pero la segunda es más sistemática.

d) 2

3( )6

sF ss s

=− −

Resol.: Comencemos con el método de completar un cuadrado perfecto en el denominador (y luego poner todo en términos de ese binomio):

1 1 12 2 2

3 3 3( 1 2) 3 2( )6 ( 1 2) 25 4 ( 1 2) 25 4

s s sf ts s s s

− − − − + = = = = − − − − − − L L L

1 1

1 1 2 22 2

1 2 1 2

3 2 5 2 5 3 53 3 cosh sinh25 4 2 5 25 4 2 5 2

t t

s s s s

s e t e ts s

− −

→ − → −

= + = + = − − L L

5 5 5 51 12 2 2 2

3 2 3 2 3 22 23 3 3 3 3 9 63 ( 3)2 5 2 2 2 10 10 5 5

t t t tt t t t t t t te e e ee e e e e e e e s

− −

− − −+ −= + = + + − = + >

InverseLaplaceTransform[3s / (s^2 – s - 6), s, t] // TrigToExp (6 E^(-2 t)) / 5 + (9 E^(3 t)) / 5

Y ahora por el método de descomponer en fracciones parciales:

2 31 1 24 36 0 ; ( )22 ( 3)( 2) 3 2

s A Bs s s F ss s s s

± +− − = ⇒ = = = = + =− − + − +

2

3 5 62 3 9 5 6 5( )2 3 0 3 26 3 2A B BAs A Bs B F s

A B A Bs s s s+ = = + + −

= ⇒ ⇒ ⇒ = + − = =− − − +

3s / (s^2 - s - 6) // Apart 9 / (5 (-3 + s)) + 6 / (5 (2 + s))

1 1 1 3 29 5 6 5 9 1 6 1 9 6 ( 3)3 2 5 3 5 2 5 5

t te e ss s s s

− − − − + = + = + > − + − + L L L

como antes.

e) 2

2 2( )2 5

sF ss s

+=

+ +

Resol.: Completando un cuadrado perfecto en el denominador:

1 1 1 12 2 2 2

1

2 2 2( 1)( ) 2 22 5 ( 1) 4 4 4

t

s s

s s s sf t es s s s s

− − − − −

→ +

+ + = = = = = + + + + + + L L L L

2 cos(2 ) ( 1)te t s−= > −

InverseLaplaceTransform[(2s + 2) / (s^2 + 2s + 5), s, t] // FullSimplify 2 E^-t Cos[2 t]

Page 55: Capítulo 1: Números Complejos

6

En este caso la descomposición en fracciones simples dejaría F(s) tal cual está, porque las raíces del denominador son complejas:

2 1 22 4 20 2 42 5 01 22 2

iis s si

− +− ± − − ±+ + = ⇒ = = = − −

Obsérvese que incluso Mathematica se niega (aunque supongo que habrá manera de obligar-la) a descomponer en fracciones simples con denominadores lineales complejos: (2s + 2) / (s^2 + 2s + 5) // Apart

(2 (1 + s)) / (5 + 2 s + s^2)

Sin embargo por esta vez, y sin que sirva de precedente, nosotros lo vamos a hacer. Desconpongamos F(s) utilizando las raíces complejas, para mostrar que esto también se pue-de hacer (aunque no sea lo habitual por ser más… complejo):

2 2

(1 2 ) (1 2 ) 2 2( )1 2 1 2 ( 1) 4 2 5A B As A i Bs B i sF s

s i s i s s s+ + + + − +

= + = = ⇒+ − + + + − − + +

2

1(1 2 ) (1 2 ) 2 ( ) 2 2 ( ) 2

A BA B

A i B i A B i A B i A B+ =

⇒ = = + + − = + + − = + − =

1 1 1 12

2 2 1 1 1 12 5 1 2 1 2 1 2 1 2

ss s s i s i s i s i

− − − −+ = + = + = + + + − + + + − + + L L L L

( )2 2

(1 2 ) (1 2 ) 2 2 2 2cos(2 )2

it iti t i t t it it t te ee e e e e e e t

−− − − + − − − −+

= + = + = =

igual que antes.

f) 2

2 3( )4

sF ss−

=−

Resol.: Aplicando linealidad y transformadas inversas inmediatas:

1 1 12 2 2

2 3 3 2 3( ) 2 2cosh(2 ) sinh(2 )4 4 2 4 2

s sf t t ts s s

− − −− = = − = − = − − − L L L

2 2 2 2

2 23 1 72 ( 2)2 2 2 4 4

t t t tt te e e e e e s

− −−+ −

= − = + >

Descomponiendo en fracciones parciales:

2 2

2 1 42 3 2 2( )2 2 3 7 44 2 2 4

A B Bs A B As A Bs BF sA B As s s s+ = = − − + +

= = + = ⇒ ⇒ − + = − =− + − −

(2 s - 3) / (s^2 - 4) // Apart 1 / (4 (-2 + s)) + 7 / (4 (2 + s))

1 1 1 12

2 2

2 3 7 4 1 4 7 1 1 14 2 2 4 4s s s s

ss s s s s

− − − −

→ + → −

− = + = + = − + − L L L L

2 1 2 1 2 2 2 27 1 1 1 7 1 7 11 14 4 4 4 4 4

t t t t t te e e e e es s

− − − − − = + = ⋅ + ⋅ = + L L

como antes. InverseLaplaceTransform[(2 s - 3) / (s^2 - 4), s, t] // TrigToExp

(7 E^(-2 t)) / 4 + E^(2 t) / 4

Page 56: Capítulo 1: Números Complejos

7

g) 2

2 1( )2 2sF s

s s+

=− +

Resol.: Completando el cuadrado de un binomio (fijándose en s2 y en el doble producto):

1 1 12 2 2

2 1 2 1 2( 1) 3( )2 2 ( 1) 1 ( 1) 1s s sf t

s s s s− − − + + − + = = = = − + − + − +

L L L

1 1 1 12 2 2 2

1 1

1 1 12 3 2 3( 1) 1 ( 1) 1 1 1s s s s

s ss s s s

− − − −

→ − → −

−= + = + = − + − + + +

L L L L

( )1 12 2

12 3 2cos 3sin ( 1)1 1

t t tse e e t t ss s

− − = + = + > + + L L

Usando fracciones parciales: 2 2 4 82 2 02

s s s ± −− + = ⇒ = ∉

y no voy a usar las raíces complejas del denominador porque ya lo hice antes y dije que no serviría de precedente. El camino a seguir es el de antes. (2 s + 1) / (s^2 – 2 s + 2) // Apart (* ni Mathematica da complejos *)

(1 + 2 s) / (2 - 2 s + s^2) InverseLaplaceTransform[(2 s + 1) / (s^2 – 2 s + 2), s, t] // FullSimplify

E^t (2 Cos[t] + 3 Sin[t])

h) 2

2

8 4 12( )( 4)

s sF ss s− +

=+

Resol.: Descompondré en fracciones parciales. El término (s2 + 4) tiene obviamente raíces complejas, así que la manera de hacerlo es esta:

2 2 2

2 2 2 2

88 4 12 4 3 5 44 ( )

( 4) 4 ( 4) 44 12

A Bs s A Bs C As A Bs Cs sC F ss s s s s s s s

A

+ =− + + + + + −= + = ⇒ = − → = ++ + + + =

Por tu cuenta comprobarías que operando esta última expresión se obtiene la función F(s) original. Con Mathematica: (8 s^2 - 4 s + 12) / (s (s^2 + 4)) // Apart

3 / s + (-4 + 5 s) / (4 + s^2)

Por tanto: 1 1 1 12 2 2

3 5 4 1 2( ) 3 5 24 4 4

s sf ts s s s s

− − − −− = + = + − = + + + L L L L

3 5cos(2 ) 2sin(2 ) ( 0)t t s= + − >

InverseLaplaceTransform[(8 s^2 - 4 s + 12) / (s (s^2 + 4)), s, t] 3 + 5 Cos[2 t] - 2 Sin[2 t]

i) 2

2 1( )4 5sF s

s s− +

=+ +

Resol.: 1 1 12 2 2

2 1 2 1 2( 2) 5( )4 5 ( 2) 1 ( 2) 1s s sf t

s s s s− − − − + − + − + + = = = = + + + + + +

L L L

1 1 2 1 2 12 2 2 2

2 2

1 12 5 2 51 1 1 1

t t

s s s s

s se es s s s

− − − − − −

→ + → +

= − + = − + = + + + + L L L L

Page 57: Capítulo 1: Números Complejos

8

( )2 2 22 cos 5 sin 2cos 5sin ( 2)t t te t e t e t t s− − −= − + = − + > −

InverseLaplaceTransform[(-2s + 1) / (s^2 + 4s + 5), s, t] // FullSimplify E^(-2 t) (-2 Cos[t] + 5 Sin[t])

Como el discriminante del denominador es b2 − 4ac = 16 − 20 < 0, las raíces del denomi-nador son complejas, la descomposición en fracciones parciales dejaría F(s) como está, y se haría lo de arriba.

j) 2

2 3( )2 10sF s

s s−

=+ +

Resol.: 1 1 12 2 2

2 3 2 3 2( 1) 5( )2 10 ( 1) 9 ( 1) 9s s sf t

s s s s− − − − − + − = = = = + + + + + +

L L L

1 1 1 12 2 2 2

1 1

5 3 5 32 29 3 9 9 3 9

t t

s s s s

s se es s s s

− − − − − −

→ + → +

= − = − = + + + + L L L L

5 52 cos(3 ) sin(3 ) 2cos(3 ) sin(3 ) ( 1)3 3

t t te t e t e t t s− − − = − = − > −

InverseLaplaceTransform[(2s - 3) / (s^2 + 2s + 10), s, t] // FullSimplify 1/3 E^-t (6 Cos[3 t] - 5 Sin[3 t])

Como el discriminante del denominador es b2 − 4ac = 4 − 40 < 0, las raíces del denomina-dor son complejas, la descomposición en fracciones simples dejaría F(s) como está, y se haría lo de arriba.

4.- Definir las funciones de los apartados a) y b) en términos de la función escalón. Encon-trar la transformada de Laplace de cada una de las funciones siguientes:

a) 2

0 2( )

( 2) 2t

f tt t

<=

− ≥

Resol.: Basta multiplicar (t − 2)2 por la función escalón desplazada 2 unidades hacia la dere-cha, pues así hacemos 0 todos los valores de f (t) en t < 2, y el resto los dejamos igual: 2( ) ( 2)( 2)f t H t t= − −

Aplicando la propiedad de desplazamiento en el tiempo (nótese que t0 = 2 > 0, es decir, el desplazamiento es hacia la derecha; de lo contrario no podríamos aplicarla, al menos así):

2

2 2 2 23 3

2! 2( ) ( 2)( 2) 0s

s s eF s H t t e t e ss s

−− − = − − = = = ∀ > L L

La condición s > 0 garantiza que la integral converge, pues estamos transformando un “poli-nomio capado”, que es una función de grado exponencial “casi 0”.

Por esta vez vamos también a resolver mediante la definición de transformada de Laplace:

2

0 2( ) ( ) ( 2)st stF s f t e dt t e dt

∞ ∞− −= = −∫ ∫

Esta integral la podemos hacer de varias maneras. Una es desarrollar e integrar por partes:

2 2

2 2 2 2( ) ( 4 4) 4 4st st st stF s t t e dt t e dt t e dt e dt

∞ ∞ ∞ ∞− − − −= − + = − +∫ ∫ ∫ ∫

Page 58: Capítulo 1: Números Complejos

9

Hacer eso me da una pereza igual al límite superior de esas integrales, pero con Mathematica: i1 = Integrate[t^2 Exp[-s t], {t, 2, Infinity}, Assumptions -> s > 0]

(E^(-2 s) (2 + 4 s (1 + s))) / s^3 i2 = Integrate[-4 t Exp[-s t], {t, 2, Infinity}, Assumptions -> s > 0]

-((4 E^(-2 s) (1 + 2 s)) / s^2) i3 = Integrate[4 Exp[-s t], {t, 2, Infinity}, Assumptions -> s > 0]

(4 E^(-2 s)) / s i1 + i2 + i3 // Simplify

(2 E^(-2 s)) / s^3

Por tanto la respuesta es 2 e−2s / s3, igual que antes.

La integral inicial también se puede resolver mediante un cambio de variable, lo cual es más parecido a lo que hacíamos para demostrar la propiedad de desplazamiento en el tiempo:

2 2 ( 2) 2 2

2 0 02( ) ( 2) ... (por partes)st s s st

tdt d

F s t e dt e d e t e dtτ

ττ

τ τ∞ ∞ ∞− − + − −

− ==

= − = = =∫ ∫ ∫

Exp[-2 s] Integrate[t^2 Exp[-s t], {t, 0, Infinity}, Assumptions -> s > 0] (2 E^(-2 s)) / s^3

Finalmente comprobemos también el resultado de esta otra manera: LaplaceTransform[UnitStep[t - 2] (t - 2)^2, t, s] // Simplify

(2 E^(-2 s)) / s^3 LaplaceTransform[HeavisideTheta[t - 2] (t - 2)^2, t, s] // Simplify

(2 E^(-2 s)) / s^3

b) 0

( ) 20 2

tf t t t

t

ππ π π

π

<= − ≤ < ≥

Resol.: Para igualar a cero a la izquierda de t = π podemos multiplicar por H(t − π); y para igualar a cero a la derecha de t = 2π podemos multiplicar por H(2π − t):

( ) ( ) (2 )( )f t H t H t tπ π π= − − − Comprobemos dibujando que sale lo que queremos: f0 = UnitStep[t - Pi] UnitStep[2 Pi - t] (t - Pi) Plot[f0, {t, 0, 3 Pi}, AspectRatio -> Automatic, PlotStyle -> Thick]

2 4 6 8

0.51.01.52.02.53.0

Figura 1. Función f (t).

La anterior es quizá la expresión más breve, pero, para calcular F(s) de forma sencilla apli-cando propiedades sin necesidad de calcular integrales, es mejor expresar f (t) así:

[ ]( ) ( ) ( 2 ) ( ) ( )( ) ( 2 )( )f t H t H t t H t t H t tπ π π π π π π= − − − − = − − − − −

A la función [H(t −π) − H(t −2π)] la podemos llamar “ventana” o “caja” (unitaria) entre π y 2π , donde vale 1 (y 0 en el resto de la recta real). Para esto Mathematica dispone de funcio-nes como UnitBox o HeavisidePi, pero por ahora vamos a ceñirnos al uso de UnitStep (o HeavisideTheta). En la expresión final, el primer sumando se “queda”

Page 59: Capítulo 1: Números Complejos

10

con (t − π) desde π hacia la derecha; y restando el segundo, que es lo mismo desde 2π hacia la derecha, queda cero. La representación gráfica confirma que ambas cosas son iguales (salvo quizá en algún punto de discontinuidad aislado, irrelevante para el cálculo de su transformada de Laplace): f = (UnitStep[t - Pi] - UnitStep[t - 2Pi]) (t - Pi) Plot[f, {t, 0, 3 Pi}, AspectRatio -> Automatic, PlotStyle -> Thick]

2 4 6 8

0.51.01.52.02.53.0

Figura 2. Función f (t) definida de otra manera.

Por supuesto sus transformadas de Laplace también deben coincidir: LaplaceTransform[f0, t, s] // Simplify LaplaceTransform[f, t, s] // Simplify

(E^(-2 Pi s) (-1 + E^(Pi s) - Pi s)) / s^2 (E^(-2 Pi s) (-1 + E^(Pi s) - Pi s)) / s^2

Calculemos F(s). Obsérvense bien las propiedades aplicadas:

[ ] [ ]( ) ( ) ( )( ) ( 2 )( )F s f t H t t H t tπ π π π= = − − − − − =L L linealidad:

[ ] [ ]( )( ) ( 2 )( )H t t H t tπ π π π= − − − − − =L L despl. en t (t0 > 0):

[ ] [ ] [ ]2 22

1!( 2 )s s s se t e t e e ts

π π π ππ π π− − − −= − + − = − + =L L L linealidad:

[ ] [ ]( )2 22 2 2

1 0s s

s se ee t e ss s s s

π ππ π ππ

− −− − = − + = − + ∀ >

L L

válido para s > 0 porque f (t) es acotada.

Comprobemos también que la aplicación de la definición también da el mismo resultado: Integrate[(t - Pi) Exp[-s t], {t, Pi, 2 Pi}]

(E^(-2 Pi s) (-1 + E^(Pi s) - Pi s)) / s^2

También partiendo de la expresión inicial de f (t), más breve, se puede obtener F(s) aplicando propiedades, pero quizá no tan fácilmente. Por ejemplo así:

[ ] ( )( )( ) ( ) (2 )( ) 2 ( ) ( )sF s H t H t t e H t tππ π π π π π π−= − − − = − + + − = L L

[ ] [ ] [ ]1( ) ( 1) ( ) 1 ( )s s sd de H t t e H t e H tds ds

π π ππ π π− − −= − = − − = − − − =L L L

[ ] [ ]( ) 11 ( )s

s sd d ee H t eds ds s s

ππ ππ

−− −

= − − − = − − =

L L

22 2 2 2 2 2

1 ( ) 1 1s s s s ss s ss e e se e e se e e

s s s s s s

π π π π ππ π ππ π π− − − − −

− − − − − − + += − − = − = −

que es lo mismo que antes.

c) ( ) ( 1) 2 ( 3) 6 ( 4)f t H t H t H t= − + − − −

Resol.: Como ejercicio previo, representa f (t) y mira si has dibujado lo que sale al ejecutar esto en Mathematica:

Page 60: Capítulo 1: Números Complejos

11

f = UnitStep[t - 1] + 2 UnitStep[t - 3] - 6 UnitStep[t - 4] Plot[f, {t, -1, 6}]

En el dibujo se ve que para calcular F(s) aplicando la definición habría que calcular tres inte-grales (entre 1 y 3, entre 3 y 4, y entre 4 e ∞). Mucho mejor es hacerlo aplicando propiedades (básicamente linealidad y traslación en el tiempo):

[ ] [ ] [ ] [ ]( ) ( 1) 2 ( 3) 6 ( 4) ( 1) 1 2 ( 3) 1 6 ( 4) 1F s H t H t H t H t H t H t= − + − − − = − ⋅ + − ⋅ − − ⋅ =L L L

[ ] [ ] [ ]3 4 3 41 3 4

1 2 1 6 1 1 2 1 6 1s s s s s st t t t t t

e e e e e e− − − − − −→ + → + → +

= + − = + − = L L L L L L

( ) [ ] ( )3 4 3 42 6 1 2 6 0s s s s s se e e e e e s s− − − − − −= + − = + − ∀ >L

LaplaceTransform[f, t, s] -((6 E^(-4 s)) / s) + (2 E^(-3 s)) / s + E^-s / s

d) ( ) ( 3) ( 2) ( 2) ( 3)f t t H t t H t= − − − − −

Resol.: Como ejercicio previo representa f (t) y mira si te sale esto: f = (t - 3) UnitStep[t - 2] - (t - 2) UnitStep[t - 3] Plot[f, {t, 0, 5}, AspectRatio -> Automatic, PlotStyle -> Thick]

1 2 3 4 5

1.00.80.60.40.2

Figura 3. Función f (t) del ejercicio.

Solución usando la definición de transformada (peor manera de hacerlo): Integrate[f Exp[-s t], {t, 2, Infinity}, Assumptions -> s > 0]

-((E^(-3 s) (1 + E^s (-1 + s) + s)) / s^2)

Para hacer eso “a mano” hay que dividir en dos subintervalos, y en uno de ellos integrar por partes. Mejor aplicar propiedades (por ejemplo linealidad y desplazamiento en el tiempo):

[ ] [ ] [ ]( ) ( 3) ( 2) ( 2) ( 3) ( 3) ( 2) ( 2) ( 3)F s t H t t H t t H t t H t= − − − − − = − − − − − =L L L

[ ] [ ] [ ] [ ]2 3 2 3( 2) 3 ( 3) 2 1 1s s s se t e t e t e t− − − −= + − − + − = − − + =L L L L

[ ] [ ]( ) [ ] [ ]( )2 3 2 32 2

1 1 1 11 1 0s s s se t e t e e ss s s s

− − − − = − − + = − − + ∀ >

L L L L

LaplaceTransform[f, t, s] // Simplify -((E^(-3 s) (1 + E^s (-1 + s) + s)) / s^2)

5.- Encontrar la transformada inversa de Laplace de las siguientes funciones:

a) 2

2

2( 1)( )2 2

ss eF ss s

−−=

− +

Resol.: Aplicando la propiedad de desplazamiento en el tiempo (con t0 = 2 > 0, con lo que se puede hacer así):

2

1 12 2

2

2( 1) 2( 1)( ) ( 2)2 2 2 2

s

t t

s e sf t H ts s s s

−− −

→ −

− − = = − − + − + L L

Page 61: Capítulo 1: Números Complejos

12

Esta última no es una de las de ejercicios anteriores, así que tendré que hacerla. El discrimi-nante del denominador es b2 − 4ac = 4 − 8 < 0, con lo que sus raíces con complejas y pro-seguimos simplemente completando el cuadrado de un binomio:

1 1 1 12 2 2 2

1

2( 1) 2( 1) 2 2 2 cos2 2 ( 1) 1 1 1

t t

s s

s s s se e ts s s s s

− − − −

→ −

− − = = = = − + − + + + L L L L

Sustituyendo arriba: 2

2( ) ( 2) 2 cos 2 ( 2) cos( 2) ( 1)t t

t tf t H t e t H t e t s−

→ − = − = − − >

f = 2 (s - 1) Exp[-2 s] / (s^2 - 2 s + 2) InverseLaplaceTransform[f, s, t] // FullSimplify

2 E^(-2 + t) Cos[2 - t] HeavisideTheta[-2 + t]

b) 2

2

2( )4

seF ss

=−

Resol.: Propiedad de traslación o desplazamiento en el tiempo (con t0 = 2 > 0):

[ ]2

1 12 2 2

2

2 2( ) ( 2) ( 2) sinh(2 )4 4

s

t tt t

ef t H t H t ts s

−− −

→ −→ −

= = − = − = − − L L

( )2( 2) 2( 2)

( 2)sinh 2( 2) ( 2) ( 2)2

t te eH t t H t s− − −−

= − − = − >

InverseLaplaceTransform[2 Exp[-2 s] / (s^2 - 4), s, t] // Simplify 1/2 E^(4 - 2 t) (-1 + E^(-8 + 4 t)) HeavisideTheta[-2 + t]

InverseLaplaceTransform[2 Exp[-2 s] / (s^2 - 4), s, t] // FullSimplify -HeavisideTheta[-2 + t] Sinh[4 - 2 t]

c) 2

2( )2

seF ss s

=+ −

Resol.: Más de lo mismo:

2

1 12 2

2

1( ) ( 2)2 2

s

t t

ef t H ts s s s

−− −

→ −

= = − + − + − L L

1 1 12 2 2

1 2

1 1 2 3 22 ( 1 2) 9 4 3 9 4 s ss s s s

− − −

→ +

= = = + − + − − L L L

3 2 3 2

1 22 2 22

2 3 2 2 3 2 1sinh ( )3 9 4 3 2 3 2 3

t t t t tt te ee e t e e e

s

− − − −− − − = = = = − −

L

Sustituyendo: ( )2 2 2( 2)

2

1 ( 2)( ) ( 2) ( ) ( 1)3 3

t t t t

t t

H tf t H t e e e e s− − − −

→ −

− = − − = − >

La otra manera de obtener la transformada inversa intermedia, la de 1 / (s2 + s − 2), es des-componiendo en fracciones parciales (con idéntico resultado y operaciones quizá un poco más largas pero quizá un poco más sencillas):

2 11 1 82 022

s s s − ± ++ − = ⇒ = = −

Page 62: Capítulo 1: Números Complejos

13

2 2

0 1 31 22 1 1 32 1 2 2A B AA B As A Bs B

A B Bs s s s s s+ = = + + −

= + = ⇒ ⇒ − = = −+ − − + + −

1 1 22

1 1 3 1 3 1 12 1 2 3 3

t te es s s s

− − −− = + = − + − − + L L

InverseLaplaceTransform[Exp[-2 s] / (s^2 + s - 2), s, t] // Simplify 1/3 E^(4 - 2 t) (-1 + E^(-6 + 3 t)) HeavisideTheta[-2 + t]

d) 2 3 4

( )s s s se e e eF s

s

− − − −+ − −=

Resol.: La transformada inversa de la función escalón desplazada se puede considerar inme-diata, pero es fácil aplicar de nuevo linealidad y desplazamientos en t (nótese que todas las exponenciales son negativas):

2 3 4

1( )s s s se e e ef t

s

− − − −− + − −

= =

L

1 1 1 1

1 2 3 4

1 1 1 1( 1) ( 2) ( 3) ( 4)t t t t t t t t

H t H t H t H ts s s s

− − − −

→ − → − → − → −

= − + − − − − − = L L L L

1 2 3 4( 1)1 ( 2)1 ( 3)1 ( 4)1t t t t t t t tH t H t H t H t→ − → − → − → −= − + − − − − − =

( 1) ( 2) ( 3) ( 4) ( 0)H t H t H t H t s= − + − − − − − >

F = (Exp[-s] + Exp[-2 s] - Exp[-3 s] - Exp[-4 s]) / s InverseLaplaceTransform[F, s, t]

-HeavisideTheta[-4 + t] - HeavisideTheta[-3 + t] + HeavisideTheta[-2 + t] + HeavisideTheta[-1 + t]

6.- Sea f (t) una función periódica de período T: f (t + T) = f (t) ∀t ≥ 0. Demostrar que

[ ] 0( )

( ) 01

T st

sT

f t e dtf t s

e

−= ∀ >−

∫L

Resol.: Además de periódica, tendremos que suponer que es de orden exponencial cero. Condición suficiente para ello es que sea continua, porque cualquier período [t, t+T] es un intervalo cerrado, y toda función continua en un intervalo cerrado está acotada en él (y luego los valores se repiten en el resto de los períodos). También basta que sea continua a trozos (es decir, con un número finito de discontinuidades de salto finito en cada período). Pero por su-puesto existen muchas funciones periódicas que no son de orden exponencial cero: por ejem-plo, la función tangente.

En cualquier caso, si f (t) es de orden exponencial cero, el teorema de existencia garantiza que su transformada de Laplace L[f (t)] existe (converge) ∀s > 0 como pide el enunciado.

Respecto a su valor, para la integral que nos interesa vamos a hacer un cambio de variable lineal de cualquier intervalo de la forma [nT, (n + 1)T] al intervalo [0, T]:

( 1) ( )

0 0 0( ) ( ) ( ) ( )

n T T T Tst s nT snT st nTs st

nT t nTd dt

f t e dt f nT e d e f t nT e dt e f t e dtτ

ττ

τ τ+ − − + − − − −

= −=

= + = + =∫ ∫ ∫ ∫

Ahora dividiremos el intervalo [0, ∞) de la integral de L[f (t)] en infinitos subintervalos de ancho igual a un período T de f (t) cada uno:

Page 63: Capítulo 1: Números Complejos

14

[ ] ( )2 ( 1)

0 0( ) ( ) lim ( ) ( ) ( )

T T n Tst st st st

T nTnf t f t e dt f t e dt f t e dt f t e dt

∞ +− − − −

→∞= = + + + =∫ ∫ ∫ ∫L

( )0 1 2

0 0 0 0lim ( ) ( ) ( ) ( )

T T T TTs st Ts st Ts st nTs st

ne f t e dt e f t e dt e f t e dt e f t e dt− − − − − − − −

→∞= + + + + =∫ ∫ ∫ ∫

( )0 1 2

0( )

TTs Ts Ts nTs ste e e e f t e dt− − − − −= + + + + + ∫

La serie infinita (pero de suma finita) entre paréntesis es una serie geométrica de 1er término e−0Ts = e0 = 1 y razón r = e−Ts = 1 / eTs siendo T, s > 0. Por tanto 0 ≤ r ≤ 1, y la serie infinita converge a su suma S = 1 / (1 − r). Si en un momento dado no te acuerdas de esta fórmula, puedes deducirla muy rápidamente (sin muchas consideraciones de rigor) así:

Definición de suma parcial finita: Sn = 1 + r + r2 + r3 + … + rn

Multiplicando por r: r Sn = r + r2 + r3 + r4 + … + rn+1

Restando: (1 − r) Sn = 1 + r − r + r2 − r2 + … + rn − rn − rn+1 = 1 − rn+1

Despejando: 11

1

n

nrS

r

+−=

Esto es cierto para cualquier n finito y cualquier r. En el límite cuando n → ∞, el numerador de Sn tiende a 1 siempre que rn+1 tienda a cero, es decir, si |r| < 1, con lo que el límite de Sn cuando n → ∞, que llamaremos S, es S = 1 / (1 − r) como queríamos recordar.

En el caso que nos ocupa:

[ ] 00

( )1( ) ( ) 01 1

T stT st

Ts

f t e dtf t f t e dt s

r e

−−= = ∀ >

− −∫

∫L

como queríamos demostrar.

7.- Utilizando el resultado del ejercicio anterior, encontrar la transformada de Laplace de las siguientes funciones:

a) 1 0 1

( ) ; ( 2) ( )0 1 2

tf t f t f t

t≤ <

= + = ≤ <

Resol.: En este caso: [ ]2

02

( )2 ( )

1

st

s

f t e dtT f t

e

−= ⇒ =−

∫L

Entre t = 1 y t = 2, f (t) = 0, con lo que solo queda esto:

[ ] ( )1 1

002 2 2

0

1 1 1( )1 1 (1 )

st tsts

s s st

e dt ef t e ee e s s e

− =−−

− − −=

⋅ = = = − = − − − − −

∫L

2

1 1 1 0(1 ) (1 )(1 ) (1 )

s s

s s s s

e e ss e s e e s e

− −

− − − −

− −= = = ∀ >

− + − +

Tengo curiosidad por ver si Mathematica es lo suficientemente inteligente para obtener este resultado sin mucha ayuda:

Page 64: Capítulo 1: Números Complejos

15

fT[t_] = Piecewise[{{1, 0 <= t < 1}, {0, 1 <= t < 2}}]; (* def en [0,T) *) f = fT[Mod[t, 2]]; (* define f(t) periódica para todo t real *) Plot[f, {t, -4, 6}, AspectRatio -> Automatic, PlotStyle -> Thick]

4 2 2 4 6

0.20.40.60.81.0

Figura 4. Función periódica f (t) del apartado a). LaplaceTransform[f, t, s] // FullSimplify

Pues no. Qué desilusión.

b) ( ) sin( ) 0 ; ( ) ( )f t t t t f t f tπ π= ∀ ≤ < + =

Resol.: En este caso [ ] 0 0( ) sin( )

( )1 1

st st

s s

f t e dt t e dtT f t

e e

π π

π ππ− −

− −= ⇒ = =− −

∫ ∫L

La integral del numerador se puede hacer por partes, pero prefiero pasar a forma exponencial, en que las primitivas son inmediatas:

( )( ) ( )

0 0 0 0

1sin( )2 2

it itst st i s t i s te et e dt e dt e dt e dt

i iπ π π π−

− − − − −−= = − =∫ ∫ ∫ ∫

( ) ( )( ) ( )

0 0

0 0

1 1 12 2 ( ) 2 ( )

t ti s t i s ti s i s

t t

e e e e e e e ei i s i s i i s i i s

π ππ π π π

= =− − −− − −

= =

= − = − + − = − − − − +

( ) ( )2 2 2 2

11 12 (1 ) 2 (1 ) 2 (1 ) 1

ss si s i s i s i s ee e

i s i s i s s

ππ π

−− − − − − + − − − + +

= − − + = − − = + + + +

Sustituyendo: [ ]2

2

( 1) (1 ) 1( ) 01 (1 )(1 )

s s

s s

e s ef t se s e

π π

π π

− −

− −

+ + += = ∀ >

− + −L

Con Mathematica comprobaré al menos que la integral del numerador la he hecho bien: Integrate[Sin[t] Exp[-s t], {t, 0, Pi}]

(1 + E^(-Pi s)) / (1 + s^2)

Si quieres calcular la integral indefinida de sin(t) e−st antes de evaluar en los límites 0 y π, también puedes hacerlo usando exponenciales sin integrar por partes; dicha primitiva resulta ser −e−st (cos t + s sin t) / (s2 + 1).

8.- Encontrar la transformada inversa de Laplace de las siguientes funciones, sin resolver las integrales y utilizando el teorema de convolución:

a) 4 2

1( )( 1)

F ss s

=+

Resol.: 1 1 1 1 34 2 4 2 4

1 1 1 1 3! 1* *sin *sin( 1) 1 3! 6

t t ts s s s s

− − − − = = = = + + L L L L

3 3

0 0

1 1sin( ) ( ) sin( )6 6

t tt d t dτ τ τ τ τ τ= − = −∫ ∫

Page 65: Capítulo 1: Números Complejos

16

Nos dicen que lo hagamos sin resolver las integrales (personalmente); pero podemos hacer que las resuelva Mathematica: InverseLaplaceTransform[1 / (s^4 (s^2 + 1)), s, t] // Simplify Integrate[1/6 u^3 Sin[t - u], {u, 0, t}] // Simplify Integrate[1/6 (t - u)^3 Sin[u], {u, 0, t}] // Simplify

1/6 t (-6 + t^2) + Sin[t] 1/6 t (-6 + t^2) + Sin[t] 1/6 t (-6 + t^2) + Sin[t]

b) 2 2( )( 1) ( 4)

sF ss s

=+ +

Resol.: 1 1 1 12 2 2 2 2

1

1 1* *cos(2 )( 1) ( 4) ( 1) 4 s s

s s ts s s s s

− − − −

→ +

= = = + + + + L L L L

12

1! *cos(2 ) ( )*cos(2 )t te t e t ts

− − − = = = L

( ) ( )

0 0cos 2( ) ( ) cos(2 )

t t te t d e t dτ ττ τ τ τ τ τ− − −= − = −∫ ∫

El resultado final (aunque no se pedía) es: InverseLaplaceTransform[s / ((s + 1)^2 (s^2 + 4)), s, t] // Simplify Integrate[Exp[-u] u Cos[2 (t - u)], {u, 0, t}] // Simplify Integrate[Exp[-(t - u)] (t - u) Cos[2 u], {u, 0, t}] // Simplify

1/25 E^-t (3 - 5 t - 3 E^t Cos[2 t] + 4 E^t Sin[2 t]) 1/25 E^-t (3 - 5 t - 3 E^t Cos[2 t] + 4 E^t Sin[2 t]) 1/25 E^-t (3 - 5 t - 3 E^t Cos[2 t] + 4 E^t Sin[2 t])

c) 2 2

1( )( 1) ( 4)

F ss s

=+ +

Resol.: 1 1 12 2 2 2

1 1 1 2*( 1) ( 4) ( 1) 2 4s s s s

− − − = = + + + + L L L

1 12 2

1

1 1 1! 1 1* sin(2 ) * sin(2 ) ( )* sin(2 )2 2 2

t t

s s

t e t e t ts s

− − − −

→ +

= = = =L L

( ) ( )

0 0

1 1sin 2( ) ( ) sin(2 )2 2

t t te t d e t dτ ττ τ τ τ τ τ− − −= − = −∫ ∫

El resultado final (aunque no se pedía) es: InverseLaplaceTransform[1 / ((s + 1)^2 (s^2 + 4)), s, t] // Expand Integrate[Exp[-u] u 1/2 Sin[2 (t - u)], {u, 0, t}] // Expand Integrate[Exp[-(t - u)] (t - u) 1/2 Sin[2 u], {u, 0, t}] // Expand

(2 E^-t)/25+(E^-t t)/5-2/25 Cos[2 t]-3/50 Sin[2 t] (2 E^-t)/25+(E^-t t)/5-2/25 Cos[2 t]-3/50 Sin[2 t] (2 E^-t)/25+(E^-t t)/5-2/25 Cos[2 t]-3/50 Sin[2 t]

9.- Resolver el siguiente problema de valor inicial (problema de datos desplazados):

2 con , 2 24 2 4

y y t y yπ π π ′′ ′+ = = = −

Resol.: N.B.: Si no has hecho ya el ejercicio 10, quizá mejor que lo hagas antes que este. La idea básica para datos desplazados es obtener la solución general de la EDO en tér-minos de las condiciones iniciales (que serán las “constantes arbitrarias” de la solución gene-

Page 66: Capítulo 1: Números Complejos

17

ral; y(0), y'(0) en nuestro caso) y luego calcular esas constantes imponiendo el cumplimiento de las condiciones desplazadas (en t = π /4 en nuestro caso).

Aplicando la transformada de Laplace a toda la EDO (= Ecuación Diferencial Ordinaria):

[ ]2 22 2

1! 2( ) (0) (0) ( ) 2 2 ( 1) ( ) (0) (0)s Y s s y y Y s t s Y s s y ys s

′ ′ − − + = = ⇒ + = + + L

2 3 2

2 2 2

2 (0) (0) 2 (0) (0)( )1 ( 1)

s s y y s y s yY ss s s

′ ′+ + + += =

+ +

1 1 12 2 2 2

2 1( ) (0) (0)( 1) 1 1

sy t y ys s s s

− − − ′= + + + + + L L L

Para la primera descomponemos en fracciones simples:

3 2 3 2

2 2 2 2 2 2

2 020 0( 1) 1 ( 1)

B B DA B Cs D As As Bs B Cs DsA A Cs s s s s s s= + =+ + + + + +

= + + = ⇒ = + =+ + +

1 1 12 2 2 2

2 2 2 2 2sin( 1) 1

t ts s s s

− − − − = + = − + + L L L

Sustituyendo arriba: ( ) 2 2sin (0)cos (0)siny t t t y t y t′= − + + InverseLaplaceTransform[(2 + s^3 y0 + s^2 yp0) / (s^2 (s^2 + 1)), s, t]

2 t + y0 Cos[t] - 2 Sin[t] + yp0 Sin[t]

Ahora determinaremos los valores de y(0), y'(0) imponiendo las condiciones en t = π /4:

2 2sin (0)cos (0)sin

4 4 4 4 4

2 2 22 2 (0) (0) (0) (0) 24 2 2 2 2

y y y

y y y y

π π π π π

π π

′= − + + =

′ ′= − + + = ⇒ + =

( ) 2 2cos (0)sin (0)cosy t t y t y t′ ′= − − +

2 2cos (0)sin (0)cos4 4 4 4

y y yπ π π π ′ ′= − − + =

2 2 22 2 (0) (0) 2 2 (0) (0) (0) (0) 12 2 2

y y y y y y′ ′ ′= − − + = − ⇒ = → = =

Sustituyendo: ( ) 2 2sin cos sin ( ) 2 sin cosy t t t t t y t t t t= − + + = = − +

edo = y''[t] + y[t] == 2t; (* ecuación diferencial ordinaria *) dd = {y[Pi/4] == Pi/2, y'[Pi/4] == 2 - Sqrt[2]}; (* datos desplazados *) DSolve[{edo, dd}, y[t], t]

{{y[t] -> 2 t + Cos[t] - Sin[t]}}

10.- Una bola de acero de masa m = 2 está unida al final de un muelle cuya constante es k = 10. Sea y(t) el desplazamiento de la bola de su posición de equilibrio. Determinar las vi-braciones libres (sin fuerzas externas aplicadas, siendo por tanto la ecuación diferencial homogénea) cuando la bola parte de la posición y(0) = 2 con una velocidad inicial y'(0) = −4 y suponiendo un rozamiento proporcional a la velocidad con una constante de proporcionalidad c = 2. La ecuación diferencial que resulta al aplicar las leyes de la Mecánica es y" + 2y' + 5y = 0.

Page 67: Capítulo 1: Números Complejos

18

Resol.: El plan es aplicar la transformada de Laplace a ambos lados de la EDO, despe-jar Y(s) (¡ingeniosa manera de transformar un problema diferencial en uno algebraico!) y cal-cular su transformada inversa:

[ ]2 ( ) (0) (0) 2 ( ) (0) 5 ( ) 0s Y s s y y sY s y Y s′ − − + − + =

( )20 0 0 2

22 5 ( ) 2 2 4 2 2 ( )2 5ss s Y s s y y y s Y s

s s′+ + = + + = − + ⋅ ⇒ =

+ +

1 1 1 12 2 2 2

1

2 2 2( 1) 2 2 2( )2 5 ( 1) 4 ( 1) 4 4 s s

s s s sy ts s s s s

− − − −

→ +

+ − − = = = = = + + + + + + + L L L L

( )1 1 12 2 2

2 2 22 2cos(2 ) sin(2 )4 4 4

t t ts se e e t ts s s

− − − − − −− = = − = − + + + L L L

InverseLaplaceTransform[2 s / (s^2 + 2 s + 5), s, t] // FullSimplify E^-t (2 Cos[2 t] - Sin[2 t])

edo = y''[t] + 2 y'[t] + 5 y[t] == 0; ci = {y[0] == 2, y'[0] == -4}; (* condiciones iniciales *) DSolve[{edo, ci}, y[t], t]

{{y[t] -> E-t (2 Cos[2 t] - Sin[2 t])}}

Aquí hemos sustituido y(0) e y'(0) por sus valores, 2 y −4 respectivamente, tan pronto como hemos podido. También se puede resolver el problema sin sustituir dichos valores hasta el final, obteniendo la solución general a la EDO en términos de ellos:

[ ]2 0 0 00 0 0 2

2( ) 2 ( ) 5 ( ) 0 ( )2 5

sy y ys Y s sy y sY s y Y s Y ss s

′+ +′ − − + − + = ⇒ = + +

1 1 10 0 0 0 0 0 0 0 02 2 2

2 2 ( 1)( )2 5 ( 1) 4 ( 1) 4

sy y y sy y y s y y yy ts s s s

− − −′ ′ ′ + + + + + + + = = = = + + + + + + L L L

1 10 0 0 00 02 2

1 1

2 cos(2 ) sin(2 )4 2 4 2

t t

s s s s

y y y ysy y e t e ts s

− − − −

→ + → +

′ ′ + += + = + + +

L L

y[t] /. DSolve[{edo, y[0] == y0, y'[0] == yp0}, y[t], t][[1]] // Simplify 1/2 E^-t (2 y0 Cos[2 t] + (y0 + yp0) Sin[2 t])

Particularizando, ahora sí, para y0 = 2, y'0 = −4, obtenemos el mismo resultado que an-tes. Sin embargo, la expresión de y(t) en términos de y0, y'0 se puede utilizar para resolver problemas de datos desplazados (como el del ejercicio 9): imponiendo el cumplimiento de los datos desplazados se obtiene un sistema lineal de 2 ecuaciones en y0, y'0 que se resuelve, y los valores obtenidos se sustituyen en la solución general.

11.- Resolver la siguiente ecuación no homogénea:

2 ; (0) 1 , (0) 0ty y y e t y y′′ ′ ′− + = + = =

Resol.: Es más que una ecuación diferencial: es un problema diferencial, concretamente uno de valor inicial, consistente en la EDO más sus condiciones iniciales. Aplicaremos la trans-formada de Laplace a ambos lados de la EDO, despejaremos Y(s) (¡ingeniosa manera de trans-formar un problema diferencial en uno algebraico!) y obtendremos y(t) como la transformada inversa de Y(s):

Page 68: Capítulo 1: Números Complejos

19

[ ]2 ( ) (0) (0) 2 ( ) (0) ( ) ts Y s s y y sY s y Y s e t′ − − − − + = + L

( ) [ ]20 0 0 2

1 12 1 ( ) 2 1 0 2 11

ts s Y s e t s y y y ss s

′ − + = + + + − = + + ⋅ + − ⋅ ⇒ −L L

2 2 4 3 22

2 2 2 2 3

1 1 2 ( 1) ( 1)( 2) 3 3 11( )2 1 ( 1) ( 1) ( 1)

s s s s s s s s s ss sY ss s s s s s s

+ + − + − + − − − + + −−= = =− + − − −

Para calcular la transformada inversa de Y(s) la descompongo en fracciones parciales. No me gusta hacer aquí estas tediosas operaciones, pero menos me gustaría hacerlas en la pizarra:

3 3 2 2 2 2

2 2 3 2 3

( 1) ( 1) ( 1) ( 1)1 ( 1) ( 1) ( 1)

A B C D E As s B s Cs s Ds s Ess ss s s s s

− + − + − + − ++ + + + = =

− − − −

4 3 2

2 3

( ) ( 3 2 ) (3 3 ) ( 3 ) ( )( 1)

A C s A B C D s A B C D E s A B s B F ss s

+ + − + − + + − + − + + − + −= =

1; 3 1 1; 2 ; 2 1; 1;

3 2 1 2( 1) 3 ; 0 ; 3 2 3 1 1 0 3 ; 1B A A C C

D D E E⇒ = − + ⋅ = = + = = −

− ⋅ + − − + = − = ⋅ − ⋅ − − + = =

Sustituyendo: 2 3

2 1 1 1( )1 ( 1)

Y ss s s s

−= + + +

− −

F = (s^2 + (s - 1) + s^2 (s - 1) (s - 2)) / (s^2 (s - 1)^3); Apart[F]

1 / (-1 + s)^3 - 1 / (-1 + s) + 1 / s^2 + 2 / s

1 1 1 12 3

1 1

1 1 1 1( ) 2s s s s

y ts s s s

− − − −

→ − → −

= + − + = L L L L

1 1 23

1 1 22 1 22 2

tt t t et e e t e t

s s− − = ⋅ + − + = + − +

L L

InverseLaplaceTransform[F, s, t] 2 - E^t + t + (E^t t^2) / 2

edo = y''[t] - 2 y'[t] + y[t] == Exp[t] + t; ci = {y[0] == 1, y'[0] == 0}; (* condiciones iniciales *) DSolve[{edo, ci}, y[t], t] // Simplify

{{y[t] -> 2 + t + 1/2 E^t (-2 + t^2)}}

12.- Resolver la ecuación diferencial:

2 ( )y y r t′′ + = con y(0) = 0, y'(0) = 0, y 1 0 1

( )0 en otro caso

tr t

< <=

de dos formas distintas: a) Aplicando directamente la transformada de Laplace a los dos términos. b) Aplicando el teorema de convolución.

Resol.: a) Esto es lo mismo que venimos haciendo en recientes ejercicios:

[ ] [ ]22 ( ) (0) (0) 2 ( ) ( ) ( )y y s Y s s y y Y s r t R s′′ ′ + = − − + = = L L

[ ] [ ] [ ]2 ( ) 0 0 2 ( ) ( ) ( 1) ( ) ( 1) 1s Y s Y s H t H t H t H t − − + = − − = − − ⋅ = L L L

Page 69: Capítulo 1: Números Complejos

20

[ ] [ ]11

1 1 1 11 1 1 (1 )s s s st t

e e e es s s s

− − − −→ +

= − = − = − = − L L L

22

1 1( 2) ( ) (1 ) ( )( 2)

ss es Y s e Y s

s s s

−− −

+ = − ⇒ =+

1 1 1 12 2 2 2

1

1 1 1( ) ( 1)( 2) ( 2) ( 2) ( 2)

s

t t

ey t H ts s s s s s s s

−− − − −

→ −

= − = − − + + + + L L L L

2 2

2 2 2

0 1 21 2 0 1 2

( 2) 2 ( 2)2 1 0

A B AA Bs C As A Bs Cs C B

s s s s s sA C

+ = = + + + + = + = ⇒ = ⇒ = − + + + = =

1 / (s (s^2 + 2)) // Apart (* comprobación parcial *) 1 / (2 s) - s / (2 (2 + s^2))

1 1 1 12 2 2

1 1 2 1 2 1 1 1( 2) 2 2 2 2

s ss s s s s s

− − − − = − = − = + + + L L L L

( )1 1 1cos( 2 ) 1 cos( 2 )2 2 2

t t= − = −

InverseLaplaceTransform[1 / (s (s^2 + 2)), s, t] (* comprob. parcial *) 1/2 - 1/2 Cos[Sqrt[2] t]

Sustituyendo: ( ) ( )1

1 1( ) 1 cos( 2 ) ( 1) 1 cos( 2 )2 2 t t

y t t H t t→ −

= − − − − =

( ) ( )( )1 11 cos( 2 ) ( 1) 1 cos 2 ( 1)2 2

t H t t= − − − − −

Esta solución se puede escribir también de otras maneras. Por ejemplo, para t < 1 el segundo término es cero, con lo que f (t) es igual al primero; y para t ≥ 1 el valor de f (t) será:

( ) ( )( ) ( )( )1 1 11 cos( 2 ) 1 1 cos 2 ( 1) cos( 2 ) cos 2 ( 1)2 2 2

t t t t− − ⋅ ⋅ − − = − + −

con lo que la solución también se puede escribir así:

( )

( )( )

1 1 cos( 2 ) 0 12( )1 cos( 2 ) cos 2 ( 1) 12

t ty t

t t t

− ≤ <= − + − ≥

La solución que da Mathematica es equivalente a esta, pero simplificada de maneras algo dis-tintas; por ejemplo: r = UnitStep[t] - UnitStep[t - 1]; edo = y''[t] + 2 y[t] == r; ci = {y[0] == 0, y'[0] == 0}; (* condiciones iniciales *) DSolve[{edo, ci}, y[t], t] // Simplify

Page 70: Capítulo 1: Números Complejos

21

b) Estudia la teoría antes de hacer ejercicios (o, como mínimo, haz una lectura comprensi-va). Este ejercicio está resuelto como ejemplo en el apartado “Aplicación del teorema de convolución a la resolución de ecuaciones diferenciales”.

Ahí se resolvía particularizando a este problema concreto la teoría, más general, desarrollada en el apartado. Aquí, por variar un poco, hagamos algo ligeramente distinto. Comencemos, igual que en el apartado a), aplicando la transformada de Laplace a ambos lados de la EDO que nos ocupa; pero sin llegar a calcular la transformada R(s) de su lado derecho r(t) (que es el estímulo o entrada al sistema). Dicho cálculo de R(s) nos lo ahorraremos aplicando el teo-rema de convolución. Observa: [ ] [ ]2 ( ) 2 ( )y y r t y y r t′′ ′′+ = ⇒ + =L L

2 2 20 0( ) 2 ( ) ( ) 0 0 2 ( ) ( 2) ( ) ( )s Y s s y y Y s s Y s Y s s Y s R s′ − − + = − − + = + =

[ ]12

( )

1( ) ( ) ( ) ( ) ( ) ( ) ( )2

G s

Y s R s G s R s y t G s R ss

−⇒ = = ⇒ =+

L

Recuerda que la función G(s) se llama función de transferencia, y es la transformada de Laplace de la respuesta del sistema entre la de la entrada bajo condiciones iniciales nulas (o la salida en frecuencia cuando la entrada es una delta de Dirac, cuya transformada en s es 1).

Como hay que calcular la transformada inversa de un producto, podemos usar el teorema de convolución: [ ] [ ] [ ]1 1 1( ) ( ) ( ) * ( * )( )y t G s R s G R g r t− − −= ⋅ = =L L L

La transformada inversa de G(s) es muy sencilla:

1 12 2

1 1 2 1( ) sin( 2 )2 22 2

g t ts s

− − = = = + + L L

y la de R(s) ni siquiera hace falta calcularla porque es r(t), dato del problema! A cambio de no calcular la transformada inversa del apartado anterior, bastante más complicada que la de g(t) por requerir descomposición en fracciones parciales, hay que calcular una convolución:

0 0

1( ) ( * )( ) ( ) ( ) sin( 2 ) ( )2

t ty t g r t g r t d r t dτ τ τ τ τ τ= = − = −∫ ∫

o bien ( )0 0

1( ) ( * )( ) ( ) ( ) sin 2 ( ) ( )2

t ty t r g t r g t d t r dτ τ τ τ τ τ= = − = −∫ ∫

Utilicemos primero esta última integral, correspondiente a (r*g)(t), que puede resultar más sencilla de manejar. Como r(t) es una función definida a trozos, el valor de la integral depen-de de si t es mayor o menor que 1 (suponiendo siempre t ≥ 0). Si t ≤ 1, r(τ ) ≡ 1 en todo el intervalo de integración (r es la función caja unidad entre 0 y 1) con lo que:

( ) ( )00

1 11: ( ) sin 2 ( ) 1 cos 2( )22

tt

t y t t d tτ

τ

τ τ τ=

=

≤ = − ⋅ = − =∫

( ) ( ) ( )1 1cos 2( ) cos 2( 0) 1 cos( 2 )2 2

t t t t = − − − = −

En cambio si t ≥ 1 la integral entre 0 y t es igual a la integral entre 0 y 1, porque r(τ ) ≡ 1 en dicho intervalo (0, 1) de integración, pero r(τ ) ≡ 0 fuera de él:

Page 71: Capítulo 1: Números Complejos

22

( ) ( )1

1

00

1 11: ( ) sin 2 ( ) 1 cos 2( )22

t y t t d tτ

τ

τ τ τ=

=

≥ = − ⋅ = − =∫

( ) ( ) ( )( )1 1cos 2( 1) cos 2( 0) cos 2( 1) cos( 2 )2 2

t t t t = − − − = − −

que es exactamente lo mismo que obteníamos en el apartado a).

Si utilizamos la integral que escribíamos como correspondiente a (g*r)(t) en lugar de la utili-zada, que es la escrita como (r*g)(t), el resultado tiene que ser el mismo, porque, como es fácil demostrar, la convolución es conmutativa. En esa otra integral, cuando τ toma valores desde 0 hasta t, (t − τ) toma valores desde t hasta 0. Por tanto, de nuevo, si t ≤ 1, r(t − τ) ≡ 1 en todo el intervalo de integración, y se tiene:

( ) ( )00

1 1 1 11: ( ) sin( 2 ) cos( 2 ) cos( 2 ) cos 0 1 cos( 2 )2 2 22

tt

t y t d t tτ

τ

τ τ τ=

=

− −≤ = = = − = −∫

Por otra parte, si t ≥ 1, la integral para τ entre 0 y t es igual a la integral entre t − 1 y t, en que r(t − τ) ≡ 1, porque la integral entre 0 y t − 1 es cero, dado que r(t − τ) ≡ 0:

( )( )11

1 1 11: ( ) sin( 2 ) cos( 2 ) cos( 2 ) cos 2 ( 1)2 22

tt

tt

t y t d t tτ

τ

τ τ τ=

−= −

− −≥ = = = − −∫

con, otra vez, igual resultado.

13.- Sea el sistema formado por la masa unida al resorte del problema 10.

El sistema está inicialmente en reposo. En el instante t = a actúa sobre el sistema un martillazo que se puede modelizar matemáticamente utilizando la Delta de Dirac como δ (t − a). El pro-blema diferencial que se obtiene aplicando las leyes de la Mecánica es:

3 2 ( ) ; (0) (0) 0y y y t a y yδ′′ ′ ′+ + = − = =

Hallar el desplazamiento y(t).

Resol.: Aplicando la transformada de Laplace a ambos lados de la EDO y propiedades (y suponiendo a > 0):

[ ] [ ]2 ( ) (0) (0) 3 ( ) (0) 2 ( ) ( )s Y s s y y sY s y Y s t aδ′ − − + − + = − L

[ ]2

0 (muestreo)( ) 0 0 3 ( ) 0 2 ( ) ( ) st sas Y s sY s Y s t a e dt eδ

∞ − − − − + − + = − = ∫

( )223 2 ( ) ( )

3 2

asas es s Y s e Y s

s s

−−+ + = ⇒ =

+ +

[ ]1 1 12 2

1( ) ( ) ( )3 2 3 2

as

t t a

ef t Y s H t as s s s

−− − −

→ −

= = = − + + + + L L L

3

1 1 1 122 2 2 2

3 2

1 1 1 13 2 ( 3 2) 1 4 1 4 1 4

t

s s

es s s s s

−− − − −

→ +

= = = = + + + − − − L L L L

3 3 3 2 2

1 22 2 22

1 22 2 sinh 21 4 2 2

t tt t t t tt e ee e e e es

− − − −− − − − = = = = − −

L

( )2 ( ) 2( )( ) 2( )

0 0( ) ( ) ( )t t t a t a

t a t at t a

t af t H t a e e H t a e e

e e t a− − − − − −

− − − −→ −

≤ < = − − = − − = − ≥

Page 72: Capítulo 1: Números Complejos

23

edo = y''[t] + 3 y'[t] + 2 y[t] == DiracDelta[t - a]; ci = {y[0] == 0, y'[0] == 0}; sol = DSolve[{edo, ci}, y[t], t, Assumptions -> a > 0] // Simplify; y[t] /. sol[[1, 1]]

E^(a - 2 t) (-E^a + E^t) HeavisideTheta[-a + t]

Page 73: Capítulo 1: Números Complejos

Capítulo 2: Transformada de Laplace. Resolución de ejercicios de pasados exámenes.

AVISO IMPORTANTE: Antes de seguir lee este AVISO.

Resoluciones completas:

Ejercicio 1 (de examen de diciembre de 2015):

Sea el sistema: ( ) ( )

( ) tr t y tg t e

donde g(t) es la transformada de Laplace inversa de la función de transferencia, que se obtiene con condiciones iniciales nulas.

a) Calcular la transformada de Laplace de la entrada r(t) mostrada en la siguiente gráfica:

b) Obtener la transformada de Laplace de la respuesta del sistema a la función delta de Dirac.

c) Obtener la respuesta del sistema a la función escalón.

d) Obtener la respuesta y(t) correspondiente a la función r(t) dada en el apartado a) utili-zando la función de transferencia del sistema.

e) Plantear, para los datos específicos de este problema, las integrales que habría que re-solver para obtener y(t) mediante la convolución de funciones.

f) Comprobar razonadamente si la siguiente ecuación diferencial sirve para modelizar el sistema dado o no:

1* ( ) 1*6 ( ) 3 ( ) ( ) ( )

2t y t y t y t y t r t

Resol.: a) Para calcular R(s) cómodamente convendrá expresar r(t) mediante escalones unitarios de esta manera: ( ) ( ) ( 1)( 1) ( 2)( 2)r t H t H t t H t t

Esta expresión es fácil de escribir directamente pensando un poquito. El primer sumando da r(t) = 1 para t entre 0 y 1 (siendo la función causal). Con el segundo ya se tiene 1 − (t − 1) = 2 − t para t entre 1 y 2. Y con tercero la suma es 2 − t + (t − 2) = 0 para t de 2 en adelante. El dibujo lo confirma.

Page 74: Capítulo 1: Números Complejos

2

r[t_] = UnitStep[t] - UnitStep[t - 1] (t - 1) + UnitStep[t - 2] (t - 2); Plot[r[t], {t, 0, 4}, AspectRatio -> Automatic]

1 2 3 4

0.2

0.4

0.6

0.8

1.0

Figura 1. Función r(t) reproducida con escalones de Heaviside.

Aplicando propiedades de linealidad y desplazamiento en el tiempo, y transformadas inmedia-tas: ( ) ( ) ( ) ( 1)( 1) ( 2)( 2)R s r t H t H t t H t t L L L L

2

22 2

11 ( )

s ss s e e

e t e t R ss s s

L L L

Rs = LaplaceTransform[r[t], t, s] // Simplify (* compruebo *) (E^(-2 s) - E^-s + s) / s^2

Si no quieres pensar casi nada (“método sistemático”) y no te importa operar un poco más, otra manera sencilla de expresar r(t) en términos de escalones es usar “ventanas” o “cajas uni-tarias” en cada subintervalo de interés. Así, para t entre 0 y 1, r(t) vale 1, por lo cual escribi-remos un primer sumando que sea 1·[H(t) − H(t − 1)]. Para t entre 1 y 2, r(t) vale 2 − t, por lo cual añadiremos como segundo sumando (2 − t)·[H(t − 1) − H(t − 2)]. Finalmente, para t mayor que 2, r(t) vale 0, con lo que no hay que añadir nada más. Poniéndolo todo junto y agrupando lo que multiplica a cada escalón sale lo mismo que antes:

r(t) = 1·[H(t) − H(t − 1)] + (2 − t)·[H(t − 1) − H(t − 2)] =

= H(t) + H(t − 1) (−1 + 2 − t) − H(t − 2) (2 − t)

Ahora las traslaciones en el tiempo se aplicarán así (con igual resultado, como debe ser):

( ) ( ) ( 1)(1 ) ( 2)(2 )R s H t H t t H t t L L L

2 21 2

11 (1 ) (2 ) 1 ( 1) 2 ( 2)s s s s

t t t te t e t e t e ts

L L L L L

2 22 2

1 1 1 1s s s se t e t e es s s s

L L

b) La función de transferencia no se pide, pero será útil para muchos cálculos:

1( ) ( ) ( ) 1

1tG s g t e G s s

s

L L

g[t_] = Exp[-t]; Gs = LaplaceTransform[g[t], t, s]

1 / (1 + s)

En el dominio de la frecuencia s, y con condiciones iniciales nulas, la salida es igual a la en-trada multiplicada por la función de transferencia: Y(s) = R(s) G(s). Si la entrada es una delta de Dirac, cuya transformada de Laplace es 1, la salida será 1·G(s) = G(s), con lo que la función de transferencia G(s) siempre es la respuesta en frecuencia del sistema a la delta de Dirac con condiciones iniciales nulas:

1( ) ( ) ( ) ( ) ( ) 1 ( ) ( )

1Y s R s G s t G s G s G s

s

L

Page 75: Capítulo 1: Números Complejos

3

c) Si la entrada es la función escalón de Heaviside H(t):

1 1 1 1( ) ( ) ( ) ( ) 1

1 1 1 1 ( 1)

A B As A BsY s R s G s H t

s s s s s s s s

L L

1 1 10 1 1 1 1; ( ) ; ( ) ( ) 1

1 1 1tA B

Y s y t Y s eA s s s s

L L L

InverseLaplaceTransform[1/s Gs, s, t] 1 - E^-t

d) Si la entrada es la función r(t) proporcionada en la figura:

2 2

2 2 2 2

1 1 1( ) ( ) ( ) ;

1 ( 1) ( 1) ( 1)

s s s se e e eY s R s G s

s s s s s s s s s s

2 2

2 2 2

01

0( 1) 1 ( 1)

1

A CA B C As As Bs B Cs

A Bs s s s s s s

B

1 1 1 1 12 2 2

1 1 1 1 1 1 11

( 1) 1 1tt e

s s s s s s s s

L L L L L

1 / s^2 / (s + 1) // Apart (* voy comprobando *) 1 / s^2 - 1 / s + 1 / (1 + s)

InverseLaplaceTransform[1 / s^2 / (s + 1), s, t] (* voy comprobando *) -1 + E^-t + t

2

1 1 12 2

1( )

( 1) ( 1) ( 1)

s se ey t

s s s s s s

L L L

1 12 2

1 2Apdo. c)

1 11 ( 1) ( 2)

( 1) ( 1)t

t t t t

e H t H ts s s s

L L

1 2

1 ( 1) 1 ( 2) 1t t t

t t t te H t t e H t t e

( 1) ( 2)1 ( 1) 1 ( 1) ( 2) 1 ( 2)t t te H t t e H t t e

1 2( ) 1 ( 1) 2 ( 2) 3t t ty t e H t t e H t t e

yt = InverseLaplaceTransform[Rs Gs, s, t] // Simplify (* ok *) 1 - E^-t + (-3 + E^(2 - t) + t) HeavisideTheta[-2 + t] - (-2 + E^(1 - t) + t) HeavisideTheta[-1 + t]

También podemos expresar y(t) por trozos:

1 ≤ t ≤ 2: 1 1( ) 1 ( 2 ) 3 3 (1 )t t t t ty t e t e e t e t e e

t ≥ 2: 2 2( ) 3 (1 ) 3 ( 1 )t t ty t t e e t e e e e

Todo junto: 2

1 1

( ) 3 (1 ) 1 2

( 1 ) 2

t

t

t

e t

y t t e e t

e e e t

Page 76: Capítulo 1: Números Complejos

4

Pongo desigualdades no estrictas (≤, ≥ en lugar de <, >) porque, aunque sea definir y(t) más de una vez en t = 1 y en t = 2, la falta de contradicción expresa el hecho de que y(t) es conti-nua. Su representación gráfica (aunque no se pedía) es la de la Fig. 2.

Plot[yt, {t, 0, 6}]

1 2 3 4

0.1

0.2

0.3

0.4

0.5

0.6

0.7

Figura 2. Función y(t).

e) En este problema son dato r(t) y g(t). El teorema de convolución nos evitaría tener que calcular R(s) y G(s), así como la transformada inversa de su producto; a cambio, habría que calcular una integral de convolución. En efecto:

1 1 1 1( ) ( ) ( ) ( ) ( ) * ( ) ( )* ( )y t Y s R s G s R s G s r t g t L L L L

siendo estas dos últimas funciones dato del problema. Así:

( )

0 0( ) ( * )( ) ( ) ( ) ( ) ( 1)( 1) ( 2)( 2)

t t ty t r g t r g t d H H H e d

También podríamos haber intercambiado r y g (la convolución es conmutativa) pero así será algo más sencillo. Teniendo en cuenta solo las funciones escalón que valen 1 en cada subintervalo:

( )

01: ( ) ( 1)

t tt y t e d t

1 ( ) ( )

0 1

1 ( ) ( )

0 1

1 2 : ( ) 1 1 ( 1)

(2 ) (1 2)

tt t

tt t

t y t e d e d

e d e d t

1 2( ) ( ) ( )

0 1 0

01 2( ) ( )

0 1

2 : ( ) (2 ) 1 1 ( 1) 1 ( 2)

(2 ) 0 ( 2)

tt t t

t t

t y t e d e d e d

e d e d t

ytconv = Integrate[r[tau] g[t - tau], {tau, 0, t}, Assumptions -> t >= 0] E^-t (-1 + E^t + (E^2 + E^t (-3 + t)) UnitStep[-2 + t] - (E + E^t (-2 + t)) UnitStep[-1 + t])

Plot[yt - ytconv, {t, 0, 6}]

1 2 3 4 5 6

4. 10 16

2. 10 16

2. 10 16

4. 10 16

Figura 3. Igual y(t) usando y sin usar convolución (salvo errores de redondeo).

Page 77: Capítulo 1: Números Complejos

5

La Fig. 3 muestra que las funciones y(t) obtenidas sin usar (antes) y usando (ahora) el teorema de convolución son la misma. Los errores, del orden de 10−16, son solo de redondeo.

f) Apliquemos la transformada de Laplace a ambos lados de la ecuación integrodiferencial dada a ver qué observamos. Usaremos las propiedades de linealidad, teorema de convolución, transformada de las derivadas, y que todas las condiciones iniciales son nulas:

1* ( ) 1*6 ( ) 3 ( ) ( ) ( )

2t y t y t y t y t r t

L L

6( ) 1 ( ) 3 ( ) ( ) ( )

2t y t y t y t y t R s L L L L L L

3 2 22

1 1( ) 0 0 0 3 ( ) 0 0 3 ( ) 0 ( ) ( )s Y s s s s Y s s sY s Y s R s

s s

13 3 1 ( ) ( 1) ( ) ( ) ( ) ( ) ( ) ( )

1s s s Y s s Y s R s Y s R s G s R s

s

Esta última ecuación nos da, en el dominio de la frecuencia, la salida en función de la entrada; por tanto, la función de transferencia es G(s) = 1 / (s + 1). Es la misma que en apartados anteriores. Esto significa que iguales entradas al sistema provocan iguales salidas (y no solo en frecuencia, sino también en tiempo, pues una función de t solo tiene una transformada de Laplace en s, y viceversa). Por tanto, podemos afirmar que la ecuación integrodiferencial pro-porcionada, aunque no nos digan de dónde sale ni cómo ha sido obtenida, describe o modeliza adecuadamente el comportamiento del sistema de apartados anteriores.

Ejercicio 2 (de examen de enero de 2016):

Un circuito eléctrico está formado por una fuente de alimentación que proporciona v(t) voltios

siendo 0 2

( )2 2

t tv t

t

, una resistencia, un condensador y una autoinducción en serie.

La ecuación que modeliza el circuito es 0

2 ( ) ( )tdi

i i u du v tdt , con i(0) = 0. Se pide:

a) 1) Hallar la función de transferencia del sistema.

2) Definir v(t) utilizando la función escalón y calcular su transformada de Laplace.

3) Calcular la intensidad que circula por el circuito, i(t), aplicando la transformada de Laplace a la ecuación integro-diferencial.

b) Calcular la siguiente integral, indicando en cada paso las propiedades de la transforma-da de Laplace que se utilizan:

5 3

0 0sin(2 )

tt xI e xe x dx dt

Resol.: a1) La función de transferencia del sistema es la función G(s) por la que hay que multipli-car, en el domino de la frecuencia, la entrada V(s) para obtener la salida I(s) bajo condiciones iniciales nulas:

I(s) = V(s) G(s) o bien G(s) = I(s) / V(s)

Si la entrada es una delta de Dirac v(t) = (t), su transformada es V(s) = 1, con lo que la fun-ción de transferencia es simplemente la transformada I(s) de la salida (siempre con condicio-nes iniciales nulas). Por ello voy a aplicar la transformada de Laplace a la ecuación

Page 78: Capítulo 1: Números Complejos

6

integrodiferencial dada, pero con (t) en vez de v(t), y entonces la salida I(s) será la función de transferencia G(s) pedida. Aplicando propiedades como linealidad, transformada de la de-rivada, de la integral, etc.:

0

12 ( ) ( ) ; ( ) (0) 2 ( ) ( ) 1

tdii i u du t sI s i I s I s

dt s L L

2 2

1 12 ( ) 1 ( ) ( )

2 1 2 1 ( 1)

s ss I s I s G s

s s s s s s

a2) Se puede hacer de más de una manera, pero la que luego permitirá aplicar propiedades como el desplazamiento en el tiempo más fácilmente es:

( ) ( ) ( 2) ( 2)v t H t t H t t

En efecto, para t < 0, ambos escalones valen cero, con lo que v(t) = 0 (función causal); para t entre 0 y 2, el escalón H(t) vale 1 y el escalón H(t − 2) vale cero, con lo que v(t) = t como que-remos; y para t ≥ 2 ambos escalones valen 1, con lo que v(t) = t − (t − 2) = 2 como queremos. Si no te resultara fácil escribir directamente expresiones como la de arriba, un pequeño dibujo te puede ayudar:

vt = UnitStep[t] t - UnitStep[t - 2] (t - 2); Plot[vt, {t, -1, 4}, AspectRatio -> Automatic]

1 1 2 3 4

0.5

1.0

1.5

2.0

Figura 4. Voltaje v(t).

Y, si no, otra manera de escribir “sistemáticamente”, o sea, casi sin pensar, la expresión de v(t) en términos de escalones unitarios, es usar “ventanas” o “cajas” unitarias como sigue. Para t entre − y 0, v(t) es 0, con lo que no hay que escribir nada. Para t entre 0 y 2, v(t) = t, con lo que escribiremos un primer sumando t·[H(t) − H(t − 2)]. Y para t a partir de 2, v(t) = 2, con lo que añadimos el sumando 2·H(t − 2). Juntando y agrupando:

v(t) = t·[H(t) − H(t − 2)] + 2·H(t − 2) = H(t) t + H(t − 2)(−t + 2) como antes.

Su transformada es: ( ) ( ) ( ) ( 2) ( 2)V s v t H t t H t t L L

2

22 2

1 11 ( 2) ( 2) ( )

ss e

t H t t e t V ss s

L L L

Y si has escrito el último sumando como H(t − 2)(−t + 2), su transformada la escribes:

2 2 2 22 2

1( 2)( 2) ( 2) ( 2) 2s s s s

t tH t t e t e t e t es

L L L L

Vs = LaplaceTransform[vt, t, s] // Simplify (* compruebo *) (1 - E^(-2 s)) / s^2

Page 79: Capítulo 1: Números Complejos

7

a3) Antes ya he aplicado la transformada de Laplace a la ecuación dada, pero con término independiente (t); ahora solo hay que cambiar esa delta de Dirac por v(t). En el dominio de la frecuencia, poniendo a la derecha V(s) en lugar de 1 (que era la transformada de (t)):

2 2 2 2 2

2 2 2

1 1 (1 ) 1 12 ( ) ( ) ( )

2 1 ( 2 1) ( 1)

s s s se e s e es I s V s I s

s s s s s s s s s

Gs = s / (s + 1)^2; Is = Vs Gs (* no se puede usar I por ser la unidad imaginaria *)

(1 - E^(-2 s)) / (s (1 + s)^2)

Solo queda calcular la transformada inversa de I(s):

2

1 12

1( ) ( ) ;

( 1)

sei t I s

s s

L L

2 2

2 2 2

1 2

( 1) 1 ( 1) ( 1)

A B C As As A Bs Bs Cs

s s s s s s s

2 2

11 1 1 1

2 0 1( 1) 1 ( 1)

0 1

A

A B C Cs s s s s

A B B

1 / s / (s + 1)^2 // Apart (* comprobación intermedia *) 1/s - 1/(1 + s)^2 - 1/(1 + s)

1 1 1 1 12 2 2

1 1 1 1 1 1 1

( 1) 1 ( 1) 1 ( 1)s s s s s s s s

L L L L L

1 1 1 12 2

1 1

1 1 1 11 1 1 1 1 ( 1)t t t t t

s s s s

e e e e t e ts s s s

L L L L

InverseLaplaceTransform[1 / s / (s + 1)^2, s, t] // FullSimplify 1 - E^-t (1 + t)

2

1 1 12 2 2

2

1 1( ) 1 ( 1) ( 2)

( 1) ( 1) ( 1)

st

t t

ei t e t H t

s s s s s s

L L L

2

1 ( 1) ( 2) 1 ( 1)t t

t te t H t e t

( 2)( ) 1 ( 1) ( 2) 1 ( 1)t ti t e t H t e t

it = InverseLaplaceTransform[Is, s, t] E^-t (-1 + E^t - t - (E^t -E^2 (-1 + t)) HeavisideTheta[-2 + t])

b) Lo primero es mirar: 5 3

0 0sin(2 )

tt xI e xe x dx dt

Empezamos por identificar la integral exterior I como una transformada de Laplace evaluada en s = 5. ¿De qué? De la integral interior (entre corchetes), que es una función de t, primitiva de la función subintegral interior, en que la variable x es “ficticia” (“dummy variable”). Esto lo sabes por derivación paramétrica pero, mucho más fundamentalmente, por el Teorema Fundamental del Cálculo (Newton, Leibniz). Así, planteamos:

3

0( ) sin(2 ) ; ( ) ( ) ; ( ) ( ) con (0) 0

ttg t te t f t g x dx f t g t f

En estas condiciones puedo aplicar la propiedad de transformada de la integral:

Page 80: Capítulo 1: Números Complejos

8

5 3

5 50 0 5 5

1 1( ) ( ) ( ) ( ) sin(2 )

5

tt t

s ss s

I e f t dt f t g d g t te ts

L L L L

Ahora hay que calcular la transformada de g(t). Primero nos “desharemos” de la exponencial mediante la propiedad de desplazamiento o traslación en s:

3

3sin(2 ) sin(2 )t

s ste t t t

L L

Aquí vemos a t multiplicando a una función cuya transformada es inmediata, así que podemos aplicar la propiedad de derivada de la transformada y casi habremos terminado:

2 2 2 2 2

2 2 2 4sin(2 ) sin(2 )

4 ( 4) ( 4)

d d s st t t

ds ds s s s

L L

Lo que queda es una simple sustitución regresiva:

32 23 25 5

2

1 1 1 1 4 1sin(2 ) sin(2 ) sin(2 )

5 5 5 5 ( 4) 40t

s s ss ss

sI te t t t t t

s

L L L

Integrate[Exp[-5t] Integrate[x Exp[3x] Sin[2x], {x,0,t}], {t,0,Infinity}] 1 / 40

Ejercicio 3 (de examen de junio de 2016):

Se considera el sistema ( )

( )

x y tH t

x y t t

con x(0) = 1, x'(0) = 1, y(0) = 0, y'(0) = 1.

Se pide: a) Obtener ( ) ( ) , ( ) ( )X s x t Y s y t L L .

b) Obtener x(t), y(t).

Resol.: a) Transformadas de los lados derechos:

2 2

1 1 1( ) ; o bien ( ) ( )

d dtH t t tH t H t

s ds ds s s

L L L L

0

0( ) ( ) 0 0 ; o bien ( ) ( ) 1 0st s d d

t t t t e dt e t t tds ds

L L L

Aplicando la transformada de Laplace a todo el sistema de EDOs:

2 2

2 2

( ) (0) (0) ( ) (0) 1( )

( ) ( ) (0) (0) ( ) (0) (0) 0

s X s sx x sY s y sx y tH t

x y t t s X s sx x s Y s sy y

L L

L L

2 2 2 2

2 22 2

( ) 1 ( ) 0 1 ( ) ( ) 1 1

( ) ( ) 2( ) 1 ( ) 0 1 0

s X s s sY s s s X s sY s s s

s X s s Y s ss X s s s Y s

Restando la 1ª ecuación de la 2ª: 22 2

1 1( ) ( ) 1 ; ( 1) ( ) 1s s Y s s s Y s

s s

Page 81: Capítulo 1: Números Complejos

9

2

3 3 3 3 2 3

1 1 1 ( 1)( 1) 1 1 1( ) ( )

( 1) ( 1) ( 1) ( 1)

s s s sY s Y s

s s s s s s s s s s s

2 2 3 2 2 3

2 1 1 1 2 1 1 1( ) ( ) ( )

sX s Y s X s

s s s s s s s s

b) 2

1 1 1 1 12 3 2 3

1 1 1 1 1 1( ) ( ) 1

2

tX s x t t

s s s s s s

L L L L L

2

1 1 1 12 3 2 3

1 1 1 1( ) ( )

2

tY s y t t

s s s s

L L L L

edo1 = x''[t] - y'[t] == t UnitStep[t]; edo2 = x''[t] + y''[t] == t DiracDelta[t]; cir = {x[0] -> 1, x'[0] -> 1, y[0] -> 0, y'[0] -> 1}; (* como regla *) cie = {x[0] == 1, x'[0] == 1, y[0] == 0, y'[0] == 1}; (* para Dsolve *) sis = {LaplaceTransform[edo1,t,s], LaplaceTransform[edo2,t,s]} /. cir; sol = Solve[sis, {LaplaceTransform[x[t],t,s], LaplaceTransform[y[t],t,s]}]

{{LaplaceTransform[x[t], t, s] -> -((-1 - s - s^2) / s^3), LaplaceTransform[y[t], t, s] -> -((1 - s) / s^3)}}

Xs = sol[[1, 1, 2]]; Ys = sol[[1, 2, 2]]; xt = InverseLaplaceTransform[Xs, s, t]

1 + t + t^2 / 2 yt = InverseLaplaceTransform[Ys, s, t]

t - t^2 / 2 DSolve[{edo1, edo2, cie}, {x[t], y[t]}, t, Assumptions -> t >= 0]

{{x[t] -> 1/2 (2 + 2 t + t^2), y[t] -> 1/2 (2 t - t^2)}}

Ejercicio 4 (de examen de diciembre de 2014):

a) Hallar las siguientes transformadas inversas de Laplace:

i) 12

1

16s

L ii) 12 2( 16)

s

s

L iii) 12 2( 16)

ase s

s

L

b) Sea cos(4 ) 0

( )0 en otro caso

t tf t

.

Se pide: i) Definirla utilizando la función escalón H(t). ii) Hallar ( )f tL .

iii) Resolver el siguiente problema de valor inicial:

16 ( ) con (0) 0 , (0) 1y y f t y y

Resol.:

a) i) Es inmediata: 1 12 2

1 1 4 1sin(4 )

16 4 16 4t

s s

L L

ii) Me doy cuenta de que la función F(s) cuya transformada inversa me piden ahora es casi la derivada de la del apartado anterior, así que espero poder aplicar la propiedad de derivada de la transformada (y la de linealidad, y el resultado del apartado anterior):

Page 82: Capítulo 1: Números Complejos

10

12 2 2 2

1 2 1 1sin(4 )

16 ( 16) 16 4

d st t t

ds s s s

L L L

12 2 2 2

1 1 sin(4 ) sin(4 )sin(4 )

( 16) 2 4 8 ( 16) 8

s t t s t tt t

s s

L L L

InverseLaplaceTransform[s / (s^2 + 16)^2, s, t] 1/8 t Sin[4 t]

Si no me doy cuenta de lo anterior, otra manera de resolver puede ser mediante el teorema de convolución, aprovechando que F(s) es el producto de dos funciones cuyas transformadas inversas conozco (son inmediatas). La integral de convolución resultante se calcula más fácilmente haciendo uso de alguna fórmula trigonométrica que, aunque de fácil deducción, no se usa todos los días; aun así las operaciones salen un poco más largas que antes:

1 1 1 12 2 2 2 2 2

1 1 4*

( 16) 16 16 4 16 16

s s s

s s s s s

L L L L

0

1 1sin(4 )*cos(4 ) sin(4 )cos 4( )

4 4

tt t t d

0 0 0

sin 4 4 4 ) sin 4 4 4 )1 1 1sin(4 ) sin(8 4 )

4 2 8 8

t t tt td t d t d

0

sin(4 ) 1 cos(8 4 ) sin(4 ) 1 sin(4 )( 0) cos(4 ) cos( 4 )

8 8 8 8 64 8

tt t t t t t

t t t

igual que antes.

Integrate[1/4 Sin[4 u] Cos[4 (t - u)], {u, 0, t}] 1/8 t Sin[4 t]

iii) Aplicando la propiedad de desplazamiento en el tiempo:

1 12 2 2 2

sin(4 )( ) ( )

( 16) ( 16) 8

as

t t at t a

e s s t tH t a H t a

s s

L L

( )sin(4( ))

( ) ( 0)8

t a t aH t a a

Esta expresión es válida solo para a ≥ 0, pues la propiedad de desplazamiento en el tiempo, que hemos aplicado para obtenerla, así lo requiere (siempre retraso en el tiempo, no adelan-to). Además con a < 0 la función F(s) = e−as s / (s2 + 16)2 no tendería a 0 cuando s → , con lo que ni siquiera sería una transformada de Laplace válida.

Por su relación con lo que estamos tratando, y aunque no lo pide el ejercicio, recordemos cómo se deduce la propiedad de desplazamiento, traslación o retraso en el tiempo:

0

( ) ( ) ( ) ( ) 1 ( ) ...st st

aH t a f t a H t a f t a e dt f t a e dt

L

Esta última igualdad solo se justifica si a ≥ 0; de lo contrario la integral seguiría yendo desde 0 hasta , no desde a hasta (y además la presencia de la función escalón sería irrelevante). Haciendo ahora el cambio t − a = dt = d :

( )

0 0( ) ( ) ( ) ( ) ( 0)s a as s as asf e d e f e d e f t e F s a

L

o bien 1 1( ) ( ) ( ) ( 0)as

t t ae F s H t a F s a

L L ■

Page 83: Capítulo 1: Números Complejos

11

b) i) Como tendré que calcular F(s), escribir f (t) de la siguiente manera me permitirá aplicar propiedades de la transformada de Laplace con facilidad:

( ) ( ) ( ) cos(4 ) ( ) cos(4 ) ( ) cos(4 )f t H t H t t H t t H t t

ft = UnitStep[t] Cos[4 t] - UnitStep[t - Pi] Cos[4 t]; Plot[ft, {t, -Pi/2, 3Pi/2}, AspectRatio -> Automatic, PlotStyle -> Thick]

1 1 2 3 4

1.0

0.5

0.5

1.0

Figura 5. Función f (t).

ii) Aplicando propiedades (linealidad, desplazamiento en t) y transformadas inmediatas:

( ) ( ) cos(4 ) ( ) cos(4 ) cos(4 ) ( )cos(4 )f t H t t H t t t H t t L L L L

2 2cos 4( ) cos(4 4 )

16 16s ss s

e t e ts s

L L

2 2cos(4 ) ( ) 1

16 16s ss s

e t F s es s

L

LaplaceTransform[ft, t, s] // Simplify (* compruebo *) (s - E^(-Pi s) s) / (16 + s^2)

iii) Aplicando la transformada de Laplace a ambos lados de la EDO:

22

16 ( ) ; ( ) (0) (0) 16 ( ) 116

s sy y f t s Y s sy y Y s e

s

L L

2 22 2 2 2

1( ) 0 1 16 ( ) 16 ( ) 1 ( )

16 16 ( 16)

s ss se s ses Y s Y s s Y s Y s

s s s

edo = y''[t] + 16 y[t] == ft; cir = {y[0] -> 0, y'[0] -> 1}; (* condiciones iniciales como reglas *) cie = {y[0] == 0, y'[0] == 1}; (* ídem como ecuaciones, para DSolve *) ecs = LaplaceTransform[edo, t, s] /. cir // Simplify

(16 + s^2) LaplaceTransform[y[t], t, s] == 1 + (s - E^(-Pi s) s) / (16 + s^2)

Ys = Solve[ecs, LaplaceTransform[y[t], t, s]][[1, 1, 2]] // Simplify (16 + s - E^(-Pi s) s + s^2) / (16 + s^2)^2

Las transformadas inversas necesarias para calcular y(t) las tengo del apartado a):

12 2 2

1 1 sin(4 ) ( )sin(4( ))( ) sin(4 ) ( )

16 ( 16) 4 8 8

ss se t t t ty t t H t

s s

L

sin(4 )( ) 2 ( ) ( )

8

ty t t H t t

yt = InverseLaplaceTransform[Ys, s, t] // Simplify -(1/8) (-2 - t + (-Pi + t) HeavisideTheta[-Pi + t]) Sin[4 t]

ytdsolve = DSolve[{edo, cie}, y[t], t][[1, 1, 2]] // Simplify -(1/8) Sin[4 t] (-4 - Pi + 2 UnitStep[t] + UnitStep[Pi - t] (2 + Pi - (2 + t) UnitStep[t]))

Page 84: Capítulo 1: Números Complejos

12

Plot[yt - ytdsolve, {t, 0, 3 Pi / 2}, PlotStyle -> Thick]

Ejercicio 5 (de examen de junio de 2015):

La respuesta de un sistema con condiciones iniciales nulas a la función escalón H(t) es

21( ) 1

2ty t e . Se pide:

i) Hallar la función de transferencia del sistema. ii) Hallar la respuesta del sistema a la función delta de Dirac (t).

Resol.: i) La función de transferencia G(s) es, en el dominio de la frecuencia y con condiciones iniciales nulas, la función por la que hay que multiplicar la entrada R(s) para obtener la salida Y(s); o bien G(s) = Y(s) / R(s). Cuando en el dominio del tiempo la entrada es r(t) = H(t) , en el de la frecuencia es: ( ) ( ) 1 1R s H t s L L

y entonces la salida es:

2 21 1 1 1 1 1 1 2 1 1 2( ) 1 2 1 2

2 2 2 2 1 2 1t t tY s e e e

s s s s s s

L L

Por tanto: ( ) ( ) 2 1

( )( ) 1 2 1 2

Y s Y s s sG s

R s s s s

yt = 1/2 (Exp[t] - 1)^2; Ys = LaplaceTransform[yt, t, s]

1/2 (1 / (-2 + s) – 2 / (-1 + s) + 1 / s) Gs = s Ys

1/2 (1 / (-2 + s) - 2 / (-1 + s) + 1 / s) s

Lo anterior valdría (puesto que no nos piden simplificar de ninguna forma concreta, y esa expresión es bastante sencilla). Si lo queremos factorizado, hay que operar otro poco:

2 2 21 2 1 2 4 3 2 1

( ) 12 2 1 2 ( 2)( 1) ( 2)( 1)

s s s s s s s sG s

s s s s s s

Gs // Factor 1 / ((-2 + s) (-1 + s))

Por cierto: el dato y(t) proporcionado (respuesta en el tiempo del sistema a la función escalón con condiciones iniciales nulas) se llama admitancia del sistema, hemos visto cómo sirve para obtener su función de transferencia.

ii) Si la entrada en el tiempo es r(t) = (t), en frecuencia es R(s) = 1, con lo que la salida en frecuencia es igual a la función de transferencia. Para obtenerla en el tiempo basta calcular la transformada inversa de G(s). Podemos partir de la expresión factorizada del apar-tado anterior; en ese caso, el primer paso suele ser descomponer en fracciones simples1:

01 2

( )2 1( 2)( 1) 2 1 ( 2)( 1)

A BA B As A Bs BG s

A Bs s s s s s

1 1 21 1 1( ) ( )

1 2 1t tB

y t G s e eA s s

L L

1 También se puede, por ejemplo, calcular la integral de convolución de dos transformadas inversas inmediatas; pero las operaciones no van a ser más sencillas.

Page 85: Capítulo 1: Números Complejos

13

InverseLaplaceTransform[Gs, s, t] E^t (-1 + E^t)

O también podemos hacerlo a partir de la expresión sin factorizar; por ejemplo, así:

1 1 2 1( ) ( )

2 1 2

s sy t G s

s s

L L

2 0

1 1 2 1 2 2

2

2 ( 2) 2 1 2 ( )( ) 2 ( )

2 2 2 2 2

tt t t

s s

s s s e e te t t e e

s s s

L L L

1 1 1 0

1

( 1) ( 1)( ) 1 ( ) ( )

1t t t t

s s

s s se e t e t e t e

s s s

L L L

2 1 2 2( ) 1 ( ) ( )( ) ( ) ( )

2 2 2 2t t t t t tt t t

y t e t e e e y t e e

L

como antes.

Ejercicio 6 (de examen de junio de 2014):

a) i) Hallar ( ) cos(3 ) cos(2 )F s t t t L

ii) Teniendo en cuenta el resultado anterior, hallar el valor de la integral:

0

cos(3 ) cos(2 ) tI t t t e dt

b) Resolver mediante la transformada de Laplace el siguiente sistema:

( ) ( ) ( )

con (0) (0) 1( ) ( ) ( )

t

t

x t x t y t ex y

y t x t y t e

.

Resol.:

a-i) ( ) cos(3 ) cos(2 ) cos(3 ) cos(2 )F s t t t t t t t L L L ;

2 2

2 2 2 2 2

9 2 9cos(3 ) cos(3 )

9 ( 9) ( 9)

d d s s s s st t t

ds ds s s s

L L

2 2

2 2 2 2 2

4 2 4cos(2 ) cos(2 )

4 ( 4) ( 4)

d d s s s s st t t

ds ds s s s

L L

2 2

2 2 2 2

9 4( )

( 9) ( 4)

s sF s

s s

ft = t (Cos[3 t] - Cos[2 t]); F[s_] = LaplaceTransform[ft, t, s]

-((-4 + s^2) / (4 + s^2)^2) + (-9 + s^2) / (9 + s^2)^2

a-ii) Se trata de una transformada de Laplace evaluada en s = 1. Concretamente, de la función f (t) del apartado anterior. Por tanto:

2 2

2 2 2 2

1 9 1 4 8 3 8 12 4 1(1) 0.04

(1 9) (1 4) 100 25 100 100 25I F I

F[1] 1 / 25

b) Aplicamos la transformada de Laplace a todo el sistema:

Page 86: Capítulo 1: Números Complejos

14

( ) ( ) ( )con (0) (0) 1

( ) ( ) ( )

t

t

x t x t y t ex y

y t x t y t e

L L

L L

1 1( ) (0) ( ) ( ) ( ) ( ) ( ) 1

1 11 1

( ) (0) ( ) ( ) ( ) ( ) ( ) 11 1

sX s x X s Y s sX s X s Y ss s

sY s y X s Y s X s sY s Y ss s

Para resolver esto “a mano” escribo la matriz ampliada del sistema y opero por filas:

2 12

11 1 1 11 1 111 ( 1)

1 112 0 1 ( 1) 11 1 1

1 11

ssss F s F

s sss ss

2 2

2 11 1 1 1 2 1 1( 2) ( ) ( ) ; ( )

1 1 1 1ts s s s

s X s X s x t es s s s

L

1 1

( ) ( 1) ( ) 1 1 1 ( ) ( ) ( )1 1

tY s s X s X s x t y t es s

edo1 = x'[t] == x[t] + y[t] - Exp[t]; edo2 = y'[t] == x[t] - y[t] + Exp[t]; cir = {x[0] -> 1, y[0] -> 1}; (* como reglas, para Laplace *) cie = {x[0] == 1, y[0] == 1}; (* como ecuaciones, para DSolve *) ecs = LaplaceTransform[{edo1, edo2}, t, s] /. cir // Simplify

{s LaplaceTransform[x[t],t,s] == 1 + 1 / (1 - s) + LaplaceTransform[x[t],t,s] + LaplaceTransform[y[t],t,s], (1 + s) LaplaceTransform[y[t],t,s] == 1 + 1 / (-1 + s) + LaplaceTransform[x[t],t,s]}

sol = Solve[ecs, {LaplaceTransform[x[t],t,s], LaplaceTransform[y[t],t,s]}] {{LaplaceTransform[x[t],t,s] -> -1 / (1 - s), LaplaceTransform[y[t],t,s] -> 1 / (-1 + s)}}

Xs = sol[[1, 1, 2]] - (1 / ( 1 - s))

Ys = sol[[1, 2, 2]] 1 / (-1 + s)

xt = InverseLaplaceTransform[Xs, s, t] E^t

yt = InverseLaplaceTransform[Ys, s, t] E^t

Ejercicio 7 (de examen de enero de 2014):

i) Sea ( ) ( )F s f t L . Deducir, en función de F(s), 0

( )t

f x dx L .

ii) Sean ( )

0 0( ) 1 2 ( ) ( )

t tt t xf t e e g x dx f x dx y 12

1( )

( 1)g t

s

L .

Hallar ( ) ( )F s f t L .

iii) Hallar 1( ) ( )f t F s L .

Resol.: i) Primero deducimos la transformada de la derivada:

Page 87: Capítulo 1: Números Complejos

15

0

( ) ( ) stf t f t e dt I L ; integrando por partes:

( ) ( )st st

f t dt dv f t v

e u se dt du

0

0 0 0( ) ( )( ) lim ( ) (0) ( )

tst st st st

t tI e f t f t se dt e f t e f s f t e dt

0 (0) ( ) ( ) ( ) (0) ( orden exp. de )f s f t f t sF s f s f L L

Si f es de orden exponencial , entonces el límite de arriba es cero s > ; y, por otra parte, s se pudo sacar de la integral porque la variable de integración era t.

Ahora consideramos la función 0

( ) ( )t

g t f d (que suele notarse F(t) en Cálculo, pero aquí

usamos la F para la transformada de Laplace). Por el Teorema Fundamental del Cálculo2 sa-bemos que g'(t) = f (t). Esto deberías entenderlo bien y ser una de esas pocas cosas que no se olvidan con el tiempo. Por si acaso, dada su importancia, y porque este ejercicio trata sobre deducir, te lo recuerdo informalmente. Observa:

0 0

0 0 0 0

( ) ( ) ( )( ) ( ) ( )

( ) lim lim lim lim ( )

t t t t t

t

t t t t

f d f d f dg t t g t t f t

g t f tt t t t

porque (hazte un pequeño dibujo) la última integral es, en el límite, el área de un rectángulo de anchura “infinitesimal” t y altura f (t) (o f (t + t), da igual). Esto no es una demostración formal, pero la idea debe entenderse.

Además de que g'(t) = f (t), g(t) también cumple g(0) = 0, pues es una integral entre 0 y 0. Aplicando ahora a g'(t) la propiedad de transformada de la derivada, deducida antes:

1 1 1( ) ( ) (0) ( ) 0 ( ) ( )g t sG s g s g t g t g t

s L L L L

y por tanto: 1

0

1( ) ( )

tf d f t

s L L c.q.d.

ii) Debes ser capaz de reconocer una integral de convolución cuando la veas. En este caso la

primera lo es: 0

( ) 1 2 * ( ) ( )tt tf t e e g t f x dx

Esto es un pequeño abuso de notación, porque e−t y g(t) son los valores de sendas funciones en el punto t, mientras que la convolución * es una operación entre funciones, no entre núme-ros. Lo que se quiere decir con e−t * g(t) es el valor en t de la convolución de la función g con la función que a t hace corresponder e−t. Como con cualquier abuso de notación, todo está bien si sabemos lo que hacemos.

La función f aparece en ambos lados (ecuación funcional, o ecuación integral, implícita). Aplicando la transformada de Laplace a ambos lados espero transformarla en una ecuación algebraica de la que poder despejar F(s). Aplicando linealidad, teorema de convolución, y demás propiedades (asegúrate de entender bien cada paso):

0

( ) 1 2 * ( ) ( )tt tf t e e g t f x dx L L

2 Una integral simple es una suma o un área, mientras que una derivada es una tasa de variación. Animales muy distintos. Sin embargo para calcular una integral se busca una función, llamada primitiva de la función subintegral, cuya derivada sea esta!

Page 88: Capítulo 1: Números Complejos

16

0

( ) 1 2 ( )tt tF s e e g f x dx L L L L L

2

1 1 1 1 1 1 1 1 12 ( ) ( ) 2 ( )

1 1 1 1 ( 1)G s f t F s

s s s s s s s s s

L

3 3

1 1 1 2 1 1 21 ( ) ( )

1 ( 1) 1 1 ( 1)

sF s F s

s s s s s s s s

2 4 2 2 4 2 4

1 2 1 2 1 2

1 1 ( 1) 1 1 ( 1) 1 ( 1)

s s s s s s

s s s s s s s s

Aunque, como se ha dicho, deberías darte cuenta a golpe de vista de que la primera es una integral de convolución, el ejercicio parece preparado para que, si no te percatas de ello, tam-bién puedas resolverlo (aunque haciendo más operaciones3). En efecto, se puede empezar calculando g(t) —que en ningún momento ha habido que calcular antes— para sustituir en la ecuación original:

1 1 12 2 2

1

1 1 1( )

( 1)t t

s s

g t e e ts s s

L L L

g[t_] = InverseLaplaceTransform[1 / (s + 1)^2, s, t] E^-t t

Sustituyendo: ( )

0 0 0( ) 1 2 ( ) 1 2( )*( ) ( )

t t tt t x x t t tf t e e e xdx f x dx e e t e f x dx

y a partir de aquí se puede calcular explícitamente la integral. Dejemos que Mathematica haga las operaciones para comprobar que el resultado anterior es correcto:

ec = f[t] == 1 - Exp[t] + 2 Integrate[Exp[-(t - x)] g[x], {x, 0, t}] - Integrate[f[x], {x, 0, t}]

f[t] == 1 - E^t + E^-t t^2 – Integral... ecs = LaplaceTransform[ec, t, s]

LaplaceTransform[f[t],t,s] == -(1 / (-1 + s)) + 1 / s + 2 / (1 + s)^3 -LaplaceTransform[f[t],t,s] / s

Ys1 = Solve[ecs, LaplaceTransform[f[t],t,s]][[1, 1, 2]] (-1 - 5 s - s^2 - s^3) / ((-1 + s) (1 + s)^4)

Ys = -1 / (s^2 - 1) + 2 s / (s + 1)^4; Ys == Ys1 // Simplify

True

Se llega al mismo resultado.

iii) 1 1 12 4 2 4

1

1 2 1 1( ) 2

1 ( 1) 1 s s

s sf t

s s s s

L L L

1 1 2 33 4

1 1 1 1sinh 2 sinh 2

2 6t tt e t e t t

s s

L L

3 3 3

2 2 21sinh

3 2 3 2 2 3

t t tt t tt e e t e t

t e t e t e t

ft = InverseLaplaceTransform[Ys, s, t] // Simplify 1/6 E^-t (3 - 3 E^(2 t) + 6 t^2 - 2 t^3)

3 Como dice el refrán, “quien no tiene cabeza, tiene que tener pies”. Si no te das cuenta de ciertas cosas, debes estar preparado para hacer más operaciones (“caminar más”).

Page 89: Capítulo 1: Números Complejos

17

ft == -Sinh[t] + Exp[-t] (t^2 – t^3 / 3) // Simplify True

ft == -Exp[t] / 2 + Exp[-t] (1/2 + t^2 – t^3 / 3) // Simplify True

Ejercicio 8 (de examen de noviembre de 2013):

Ejercicio 9 (de examen de noviembre de 2013):

Ejercicio 10 (de examen de junio de 2013):

a) Hallar la transformada inversa de Laplace de 3

2

( 1)( )

4 7

ss eF s

s s

.

b) Resolver la siguiente ecuación diferencial con condiciones iniciales:

2( ) ( ) 5 ty t y t e con y(0) = 0, y'(π) = 2e2π

Resol.:

a) 3

1 12 2

3

( 1) 1( ) ( 3)

4 7 4 7

s

t t

s e sf t H t

s s s s

L L

1 1 1 2 12 2 2 2

2

1 ( 2) 3 3 3

4 7 ( 2) 3 3 3t

s s

s s s se

s s s s s

L L L L

1 1 12 2 2

3 33 sin( 3 ) 3 cos( 3 )

3 3 3

s st t

s s s

L L L

Sustituyendo todo: 2( 3)( ) ( 3) sin 3 ( 3) 3 cos 3 ( 3)tf t H t e t t

Fs = (s - 1) Exp[-3 s] / (s^2 + 4 s + 7) InverseLaplaceTransform[Fs, s, t] (* comprobación *)

E^(6 - 2 t) HeavisideTheta[-3 + t] (Cos[Sqrt[3] (-3 + t)] - Sqrt[3] Sin[Sqrt[3] (-3 + t)])

b) Aplicando L a ambos lados:

2 2 5( ) ( ) 5 ; ( ) (0) (0) ( )

2ty t y t e s Y s sy y Y s

s

L L L

2 2 00 0 2 2

5 5 5( ) 0 ( ) ; ( 1) ( ) ; ( )

2 2 ( 2)( 1) 1

ys Y s y Y s s Y s y Y s

s s s s s

edo = y''[t] + y[t] == 5 Exp[2 t]; cir = {y[0] -> 0, y'[Pi] -> 2 Exp[2 Pi]}; (* como reglas para Laplace *) cie = {y[0] == 0, y'[Pi] == 2 Exp[2 Pi]}; (* como ecuaciones a DSolve *) ecs = LaplaceTransform[edo, t, s] /. cir // Simplify

(1 + s^2) LaplaceTransform[y[t],t,s] == 5 / (-2 + s) + (y^\[Prime])[0] Ys = Solve[ecs, LaplaceTransform[y[t], t, s]]; Ys = Ys[[1, 1, 2]] // Apart

5 / ((-2 + s) (1 + s^2)) + (y^\[Prime])[0] / (1 + s^2)

Comprobado Y(s), calculo su transformada inversa y(t), que me quedará en función de y'(0):

Page 90: Capítulo 1: Números Complejos

18

1 102 2

5 1( )

( 2)( 1) 1y t y

s s s

L L

2 2

2 2 2

05 2 2

2 0( 2)( 1) 2 1 ( 2)( 1)

2 5

A BA Bs C As A Bs Bs Cs C

B Cs s s s s s

A C

Para resolver estos sistemas lineales a mano, salvo que estén totalmente desacoplados o sean extremadamente sencillos, y aunque parezca matar pulgas a cañonazos, es cómodo (porque hay que pensar poco) y rápido, y se presta poco a errores, utilizar la matriz ampliada y hacer operaciones elementales por filas, normalmente aplicando el método de Gauss sin grandes cambios. Pruébalo:

23 1 3

1 1 0 0 1 1 0 0 1 1 0 0 2

0 2 1 0 0 2 1 0 0 2 1 0 12

1 0 2 5 0 1 2 5 0 0 5 2 5 1

CF

F F F B

A

Sustituyendo: 1 1 1 22 2

5 1 2cos 2sin

( 2)( 1) 2 1ts

e t ts s s s

L L L

1 1 20 02 2

5 1( ) cos 2sin sin

( 2)( 1) 1ty t y e t t y t

s s s

L L

yt[t_] = InverseLaplaceTransform[Ys, s ,t] E^(2 t) - Cos[t] - 2 Sin[t] + Sin[t] (y^\[Prime])[0]

Solo queda imponer la condición desplazada (la que se impone en t ≠ 0) para obtener el valor de y'0 = y'(0) y sustituir en y(t):

2 20 0( ) cos 2sin sin ( ) 2 sin 2cos cost ty t e t t y t y t e t t y t

2 2 20 0 0( ) 2 sin 2cos cos 2 0 2 2 2y e y e y e y

2( ) costy t e t

Solve[yt'[Pi] == 2 Exp[2 Pi], y'[0]] {{(y^\[Prime])[0] -> 2}}

yt[t] /. Solve[yt'[Pi]==2 Exp[2 Pi],y'[0]] {E^(2 t) - Cos[t]}

Ejercicio 11 (de examen de enero de 2013):

Ejercicio 12 (de examen de enero de 2013):

Ejercicio 13 (de examen de noviembre de 2012):

Sea f una función tal que 0

( ) 4 3 ( )sin( )t

f t t f x x t dx .

i) Hallar F(s), transformada de Laplace de f (t). ii) Resolver la siguiente ecuación diferencial:

0

( ) ( ) ( ) ( ) 2cos con(0) (0) 0

ay t y t H t a f t a t

y y

siendo f la función del apartado anterior.

Page 91: Capítulo 1: Números Complejos

19

Resol.: i) Identificando la integral como una de convolución (cambiada de signo; recuerda que la función seno es impar):

0 0

( ) 4 3 ( )sin( ) 4 3 ( )sin( ) 4 3( *sin)( )t t

f t t f x x t dx t f t d t f t

Aplicando L a ambos lados y el Teorema de Convolución:

2

4( ) 4 3 *sin 3 sinf t t f f

s L L L L L

2 2

2 2 2 2 2 2

4 1 1 3 4 4( 1)( ) 3 ( ) ( ) ( )

1 1 ( 4)

s sF s F s F s F s

s s s s s s

N.B. En realidad es más correcto escribir L[ f ] que L[ f (t)]; L[sin] que L[sin(t)],

etc.; porque las transformadas, como la de Laplace, hacen corresponder a una función otra función, y poniendo (t) en realidad no estamos escribiendo la función propiamente dicha, sino su valor en t. Se trata de un abuso de notación benigno para recordarnos en cada caso si esta-mos trabajando en el dominio del tiempo o de la frecuencia.

ec = f[t] == 4 t + 3 Integrate[f[x] Sin[x – t], {x, 0, t}]; ecs = LaplaceTransform[ec, t, s] // Simplify

((4 + s^2) LaplaceTransform[f[t], t, s]) / (1 + s^2) == 4 / s^2 Fs = Solve[ecs, LaplaceTransform[f[t], t, s]][[1, 1, 2]]

{{LaplaceTransform[f[t], t, s] -> (4 (1 + s^2)) / (s^2 (4 + s^2))}}

ii) Se puede calcular la transformada inversa de la función F(s) del apartado anterior para obtener f (t) y sustituirla en la ecuación diferencial; pero no se nos pide explícitamente, y tam-poco la vamos a necesitar (porque lo primero que haríamos con f (t) sería volver a aplicarle la transformada de Laplace). Por curiosidad, o por si tú la has calculado, es esta:

InverseLaplaceTransform[Fs, s, t] // Simplify t + 3 Cos[t] Sin[t]

Pero aquí transformaremos directamente, sin usar f (t), ambos lados de la EDO:

( ) ( ) ( ) ( ) 2cosy t y t H t a f t a t L L

2 ( ) (0) (0) ( ) ( ) ( ) 2 coss Y s s y y Y s H t a f t a t L L

22

( ) 0 0 ( ) ( ) 2 ( 0)1

as ss Y s Y s e f t a

s

L

2

22 2 2 2 2 2 2

4( 1) 4 2( 1) ( ) 2 ( )

( 4) 1 ( 4) ( 1)

asas s s e s

s Y s e Y ss s s s s s

La comprobación con Mathematica de Y(s) es antipática, porque no aplica la propiedad de desplazamiento en el tiempo, ni siquiera añadiendo la cláusula Assumptions -> a > 0. Sin embargo se puede hacer con sustituciones ad hoc:

edo = y''[t] + y[t] == UnitStep[t - a] f[t - a] + 2 Cos[t]; ci = {y[0] -> 0, y'[0] -> 0}; ecs = LaplaceTransform[edo, t, s] /. ci // Simplify; ecs = ecs /. LaplaceTransform[f[-a + t] UnitStep[-a + t], t, s] -> Exp[-a s] LaplaceTransform[f[t] ,t, s]; ecs = ecs /. LaplaceTransform[f[t], t, s] -> Fs

(1 + s^2) LaplaceTransform[y[t],t,s] == 2 s / (1 + s^2) + 4 E^(-a s) (1 + s^2) / (s^2 (4 + s^2))

Page 92: Capítulo 1: Números Complejos

20

Ys = Solve[ecs, LaplaceTransform[y[t], t, s]]; Ys = Ys[[1, 1, 2]] // FullSimplify

2 s / (1 + s^2)^2 + 4 E^(-a s) / (s^2 (4 + s^2))

Solo queda calcular la transformada inversa de Y(s). Teniendo en cuenta que a > 0:

1 1 12 2 2 2 2 2 2 2

4 2 4 2( ) ( )

( 4) ( 1) ( 4) ( 1)

as

t t a

e s sy t H t a

s s s s s s

L L L

3 2 3 2

2 2 2 2 2 2

0 4 04 4 4;

0 4 4( 4) 4 ( 4)

A C AA B Cs D As A s Bs B Cs Ds

B D Bs s s s s s s

1 1 12 2 2 2 2

4 1 1 1 2 sin(2 )

( 4) 4 2 4 2

tt t

s s s s s

L L L

1 1 12 2 2 2

2 1 1sin

( 1) 1 1

s dt t t

s ds s s

L L L

sin 2( )sin(2 )

( ) ( ) sin ( ) sin2 2t t a

t aty t H t a t t t H t a t a t t

InverseLaplaceTransform[Ys, s, t] HeavisideTheta[-a + t] (-a + t + 1/2 Sin[2 (a - t)]) + t Sin[t]

Ejercicio 14 (de examen de de ):

Ejercicio 15 (de examen de de ):

Ejercicio 16 (de examen de de ):

Ejercicio 17 (de examen de de ):

Ejercicio 18 (de examen de de ):

Ejercicio 19 (de examen de de ):

Ejercicio 20 (de examen de de ):

Ejercicio 21 (de examen de de ):

Ejercicio 22 (de examen de de ):

Ejercicio 23 (de examen de de ):

Ejercicio 24 (de examen de de ):

Ejercicio 25 (de examen de de ):

Ejercicio 26 (de examen de de ):

Ejercicio 27 (de examen de de ):

Ejercicio 28 (de examen de de ):

Page 93: Capítulo 1: Números Complejos

Capítulo 3: Funciones de Variable Compleja. Resolución de ejercicios de fin de capítulo.

AVISO IMPORTANTE: Antes de seguir lee este AVISO.

Resoluciones completas:

1.- Expresar las siguientes funciones: 31( )f z z , 2

1( )

1f z

z

, 3

1( )

1

zf z

z

en la

forma f (x + i y) = u(x,y) + i v(x,y).

Resol.: Ejercicio sencillo, para empezar. Se trata de sustituir z = x + i y, desarrollar, y separar par-tes real e imaginaria. Intentaremos darle algo de valor añadido con el uso de Mathematica.

Para f1(z): si, de utilizarlo, te has aprendido de memoria el cubo de un binomio (aunque siempre puedes aplicar tú mismo la propiedad distributiva):

(a + b)^3 // Expand a^3 + 3 a^2 b + 3 a b^2 + b^3

sale fácil: 3 3 3 2 2 3 3 2 2 31

( , ) ( , )

( ) ( ) 3 ( ) 3 ( ) ( ) 3 (3 )u x y v x y

f z z x iy x x iy x iy iy x xy i x y y

z = x + I y; (* mantener esta definición en Mathematica en adelante *) z^3 // Expand (* no agrupa como quiero porque no asume x, y reales *)

x^3 + 3 I x^2 y - 3 x y^2 - I y^3 z^3 // ComplexExpand (* con ComplexExpand sí *)

x^3 - 3 x y^2 + I (3 x^2 y - y^3)

Para f2(z): como en el tema de Números Complejos, multiplicar y dividir por el conjuga-do del denominador:

2 2 2 2 2 2 2

( , ) ( , )

1 1 1 1 1( )

1 1 1 (1 ) (1 ) (1 )u x y v x y

x iy x iy x yf z i

z x iy x iy x y x y x y

(y claro, también se puede expandir el (1 − x)2 del denominador).

uxy = 1 / (1 - z) // Re // ComplexExpand // FullSimplify (1 - x) / ((-1 + x)^2 + y^2)

vxy = 1 / (1 - z) // Im // ComplexExpand y / ((1 - x)^2 + y^2)

Para f3(z):

2 2 2 2

3 2 2 2 2 2 2

( , ) ( , )

1 1 1 ( ) 1 2( )

1 1 ( 1) ( 1) ( 1)u x y v x y

x iy x iy x y i xy y xy y x y yf z i

x iy x iy x y x y x y

uxy = (z - 1) / (z + 1) // Re // ComplexExpand // Simplify (-1 + x^2 + y^2) / (1 + 2 x + x^2 + y^2)

vxy = (z - 1) / (z + 1) // Im // ComplexExpand 2 y / ((1 + x)^2 + y^2)

Quizá lo más importante a sacar de este ejercicio es entender que, para toda función compleja de variable compleja f (z), siempre existen funciones u(x,y), v(x,y) tales que f (z) = u(x,y) + i v(x,y), porque tanto z como f (z) quedan biunívocamente determinados por sus partes real e imaginaria. En otras palabras: f (z) debe tener una parte real, que será función

Page 94: Capítulo 1: Números Complejos

2

de x e y por serlo de z, y que llamo u(x,y); y una parte imaginaria, que también será función de x e y por serlo de z, y que llamo v(x,y).

2.- Determinar en función de x e y las partes real e imaginaria de las funciones 1( ) 1f z z

y 2 ( )f z z z . Comprobar si se verifican las ecuaciones de Cauchy-Riemann.

Resol.: Antes de mover un dedo ya sé que las ecuaciones de Cauchy-Riemann no se van a cumplir en general en ninguno de los dos casos, porque z aparece tanto en la expresión de f1(z) como en la de f2(z); y no es una aparición “ficticia” que desaparece simplificando, como p. ej. en la expresión “ 1z z ”. Si aparece z , es que la derivada parcial respecto de z no es cero1. Y recordemos que la forma compleja de las ecuaciones de Cauchy-Riemann (C-R) es

0f z (ver demostración de esto en las trasparencias en eGela). Así que, si z aparece, las ecuaciones de C-R no se cumplen (y la función no puede ser derivable en esos puntos).

Las derivaciones parciales z “para z constante” y z “para z constante” son operado-res de derivación simbólica puramente formal, porque no tiene sentido hacer variar a z sin que varíe z o viceversa. En el plano x, y, la derivación parcial x “para y constante” es una

derivada direccional por una recta horizontal; y la derivación parcial y “para x constante” es una derivada direccional por una recta vertical; y ambas cosas tienen sentido estricto por-que x, y son variables independientes. Pero z y z dependen biunívocamente una de la otra: mover una (en cualquier sentido) implica mover la otra. Cuando aplicamos los operadores formales z , z hacemos abstracción de este hecho2: se trata de operadores de deriva-ción formal “ficticia”.

Al lío: 1 2 2 2 2 2 2

( , ) ( , )

1 1( )

u x y v x y

x iy x iy x yf z i

z x iy x iy x y x y x y

2 2 2 22 2

2 2 2 2 2 22 2 2 2

2 2 2 2

2 2 2 2 2 2

( , ) 2 0( ) ( )

;( , ) 2

0( ) ( )

x

x y

y

u x y x y x x x y x yux x y x y

u v x y x yv x y x y y y x y

y xvy x y x y

2 2

2 2 2 2 2 2

0, 002 2; ;

0, 0( ) ( ) 2 0y x y x

x yx yx y y xu v u v

y xx y x y xy

La primera ecuación se cumple solo en las dos bisectrices de los cuatro cuadrantes (salvo en z = 0), mientras que la segunda se cumple solo en los ejes real e imaginario (salvo en z = 0). Por tanto, ambas simultáneamente no se cumplen en ningún punto: Las ecuaciones de C-R no se cumplen en ningún punto del plano complejo.

Lo comprobamos con Mathematica:

u = 1 / Conjugate[z] // Re // ComplexExpand x / (x^2 + y^2)

1 Salvo quizá en algunos puntos, aunque en este caso ni siquiera eso. 2 Como se suele decir, hacemos “de tripas corazón”. Pero es un artificio matemático que funciona muy bien y que es muy útil para ciertas cosas, con alguna aplicación muy ingeniosa que no estudiaremos.

Page 95: Capítulo 1: Números Complejos

3

v = 1 / Conjugate[z] // Im // ComplexExpand y / (x^2 + y^2)

ux = D[u, x] // Simplify (-x^2 + y^2) / (x^2 + y^2)^2

vy = D[v, y] // Simplify x^2 - y^2 / (x^2 + y^2)^2

uy = D[u, y] // Simplify - 2 x y / (x^2 + y^2)^2

vx = D[v, x] // Simplify - 2 x y / (x^2 + y^2)^2

Solve[ux == vy, {x, y}] Solve::svars: Equations may not give solutions for all "solve" variables. {{y -> -x}, {y -> x}}

Solve[uy == -vx, {x, y}] Solve::svars: Equations may not give solutions for all "solve" variables. {{x -> 0}, {y -> 0}}

Solve[{ux == vy, uy == -vx}, {x, y}] {}

Pero espero llegar a la misma conclusión mucho más rápidamente con la forma compleja de

las ecuaciones de C-R: 12

1 10

f

z z z z

para ningún z . Efectivamente, ¡más

rápido es difícil!

Para f2: 2 2 2 2

2 2 2 2 2 2 2

( , ) ( , )

2 2( )

u x y v x y

z x iy x iy x y xyi x y xyf z i

z x iy x iy x y x y x y

u = Conjugate[z] / z // Re // ComplexExpand // Simplify (x^2 - y^2) / (x^2 + y^2)

v = Conjugate[z] / z // Im // ComplexExpand - 2 x y / (x^2 + y^2)

Rápidamente (utilizando la forma compleja de las ecs. de C-R): 2 10

f z

z z z z

para ningún z , con lo que las ecs. de C-R no se cumplen en ningún punto del plano com-

plejo. Lo compruebo en coordenadas cartesianas:

2 2

2 2 2 2 22 2 2

2 2 2 2 2 2

2 2 2 2

2 2 22 2 2 2 2 2

0( )2 ( )2 4

4 2 ( )( ) ( )

; 0, 0( )( 2 ) 2 2 2 ( )

4 2 2( ) ( )0,

x

x y

y

x yx y x x y x xy

u xy x x yx y x y

u v x yx y x xy y x x y

v y x yx y x yx y

2 2

2 2 2 2 22 2 2

2 2 2 2 2 2

2 2 2 2

2 2 22 2 2 2 2 2

0( )( 2 ) ( )2 4

4 2 ( )( ) ( )

; 0, 0( )( 2 ) 2 2 2 ( )

4 2 2( ) ( )0,

y

y x

x

x yx y y x y y x y

u x y y x yx y x y

u v y xx y y xy x y x y

v x x yx y x yy x

Page 96: Capítulo 1: Números Complejos

4

La primera ecuación se cumple solo en el eje imaginario (salvo en z = 0), mientras que la se-gunda se cumple solo en el eje real (salvo en z = 0). Por tanto, ambas simultáneamente no se cumplen en ningún punto: Las ecuaciones de C-R no se cumplen en ningún punto del plano complejo.

ux = D[u, x] // Simplify 4 x y^2 / (x^2 + y^2)^2

vy = D[v, y] // Simplify - 2 x (x^2 - y^2) / (x^2 + y^2)^2

uy = D[u, y] // Simplify - 4 x^2 y / (x^2 + y^2)^2

vx = D[v, x] // Simplify 2 y (x^2 - y^2) / (x^2 + y^2)^2

Solve[ux == vy, {x, y}] Solve::svars: Equations may not give solutions for all "solve" variables. {{x -> 0}}

Solve[uy == -vx, {x, y}] Solve::svars: Equations may not give solutions for all "solve" variables. {{y -> 0}}

Solve[{ux == vy, uy == -vx}, {x, y}] {}

3.- Determinar los puntos singulares de las funciones 1

1( )

2

zf z

z i

y

3

2

3( )

( 2)( 5 )

z zf z

z z i

.

Resol.: Los dos numeradores y los dos denominadores, al ser polinomios complejos, son funciones enteras (analíticas en todo el plano complejo). Ambos cocientes son funciones racionales (analíticas en todo en el plano complejo salvo en los ceros del denominador). Puntos singula-res son puntos donde una función no es analítica, sean aislados o no (y de paso recordemos que una función es analítica en un punto si es derivable no solo en él sino en algún entorno de él). Por tanto los únicos puntos singulares de f1 y f2 (como los de toda función racional en ge-neral) son los ceros del denominador: Puntos singulares de f1: 2i. Puntos singulares de f2: −2, 5i.

4.- Hallar z tal que ez = −1.

Resol.: Los valores de z buscados son, por definición, los del logaritmo multiforme3 de −1 (escrito log, en contraste con Log, que es cualquier determinación o rama, por ejemplo la principal):

log( 1) L 1 arg( 1)z i

3 Función unívoca, univaluada o uniforme (o simplemente función): la que a cada número (de su do-minio de definición, que no tiene por qué ser un dominio) le hace corresponder un único número. Ejemplos: la función exponencial, o el argumento principal Arg (−π, π].

Función multívoca, multivaluada o multiforme: correspondencia (no es realmente una función) que a al menos un número de su dominio de definición le hace corresponder más de un número. P.ej.: arg.

Cuando, por el contexto, sea importante distinguir una función (unívoca) de una “función” multívoca, se notará la primera con mayúscula y la segunda con minúscula.

Page 97: Capítulo 1: Números Complejos

5

donde L es el logaritmo neperiano real de variable real (función unívoca y siempre mayor que cero), | · | es el módulo de un número complejo, y arg es la función “argumento multiforme”, es decir, el conjunto de todos los posibles argumentos de un número complejo, separados por múltiplos de 2π (en contraste con Arg, que es cualquier argumento univaluado, como por ejemplo el principal, (−π, π]). Por tanto:

L(1) ( 2 ) 0 ( 2 ) ( 0, 1, 2, 3...)z i k i k k

( 2 )z i k k

En este capítulo cuando escribamos 2kπ, 2πk, π/2 + kπ y expresiones parecidas, prácti-camente siempre se sobreentenderá que es para todo k entero ( k ) aunque no se diga

explícitamente.

Podemos comprobar que efectivamente ez = −1:

( 2 ) cos( 2 ) sin ( 2 ) 1 0 1z i ke e k i k i

Y podemos comprobar que no hay más soluciones que esas; pero para eso prefiero recordarte la teoría y deducir la fórmula del logaritmo multiforme en general:

Sean z = x + i y, w = y + i v, y w = log(z), significando esto último, por definición de logaritmo multiforme, que w son todos los números en los que la función exponencial vale z:

ew = z

Piensa en z como dato y en w como incógnita a determinar en función de z; y piensa que bus-camos todas las posibles soluciones w. Y recuerda que ew = exp(w) es la función exponen-cial, que conocemos bien. (Esta la escribimos con minúscula porque siempre es unívoca, y por tanto no ha lugar a confusión.) Por tanto:

Arg( )

(cos sin ) ( )u iv u uv z

e z e v i v z e z

donde Arg(z) es cualquier argumento univaluado; por ejemplo, el principal.

Buscamos todos los valores w que cumplen lo anterior para cierto z. Para que dos números complejos distintos de cero sean iguales es condición necesaria y suficiente que sus módulos sean iguales y que sus argumentos difieran en algún múltiplo de 2π; por tanto:

Arg( ) 2 ( )

ue z

v z k k

Siendo | z | > 0 y u real, la primera ecuación tiene como única solución u = L(| z |). Por su-puesto al escribir | z | > 0 ya se entiende que | z | es real; y el caso | z | = 0 no importa porque el número z = 0 no tiene logaritmo (no hay número real u al que elevado e dé 0).

Respecto a la segunda, es más cómodo escribir simplemente v = arg(z), siendo arg el ar-gumento multiforme (todos los posibles argumentos de z). Sustituyendo:

log( ) L arg( )z w u iv z i z

Finalmente escribiendo L| z | en vez de L(| z |) (lo cual no suele dar lugar a confusión) obtene-mos la expresión general del logaritmo multivaluado de un número complejo z:

log( ) L arg( )z z i z

Page 98: Capítulo 1: Números Complejos

6

Como dijimos antes, el valor z = 0 no tiene logaritmo, lo cual concuerda con que L(|0|) = L(0) no existe (el límite de L(x) cuando x → 0+ es −, pero eso “no vale”).

Finalmente, una determinación o rama del logaritmo es una función unívoca y analítica en cierto dominio del plano complejo que se obtiene eligiendo un argumento unívoco Arg para la parte imaginaria. Por ejemplo, si para la parte imaginaria del logaritmo se elige el argumento principal, Arg(z) (−π, π], a esa determinación del logaritmo de z, notada Log(z), se la llama determinación o rama principal del logaritmo. (Las demás determinaciones también se suelen notar igual, Log.)

5.- Probar que: a) 2exp(2 3 )i e .

b) 2

exp 2(1 )4

ie i

c) exp( ) exp( )z i z

Resol.:

a) 2 2 2 23exp(2 3 ) ( ) cos( 3 ) sin( 3 ) 1 0i e e i e i e

b) 2 4

4

2 2 2exp 1 2(1 )

4 2 2 2

i ee e i i e i

c) exp( ) exp( ) ( ) ( ) exp( )x xy yz i x iy i e e z

Lo anterior deberías entenderlo directamente pensando en el plano complejo, pero por si aca-so aquí está más detallado: exp( ) exp( ) exp ( ) cos( ) sin( )xz i x iy i x i y e y i y

cos ( sin ) cos sin exp( )x x ze y i y e y i y e z 4

6.- Hallar z tal que: a) 2ze

b) 1 3ze i

c) 2 1 1ze

Resol.: a) Los números z tales que exp(z) = −2 son por definición los valores del logaritmo multi-

forme de −2: log( 2) L 2 arg( 2) L 2 ( 2 )z i i k k

b) log 1 3 L 1 3 arg 1 3 L 2 3 2z i i i i i k k

4 Frente a ez la notación exp(z), desde un punto de vista tipográfico, se lee mejor, porque no requiere que z vaya como superíndice (y si en lugar de z hay otra expresión con sus propios sub- y superíndices, con más razón). Pero sobre todo exp(z) tiene la ventaja de ser muy clara: se trata de la función expo-nencial (unívoca, elemental, entera) exp(z) = e x(cosy + i sin y). En cambio la notación ez también podría interpretarse como “la potencia multívoca ab, siendo la base a = e y el exponente b = z”. En par-te por esto, y también porque no requiere paletas ni caracteres especiales, en Mathematica donde mu-chos ponéis Ez o E^z yo suelo preferir Exp[z].

Page 99: Capítulo 1: Números Complejos

7

c) 2 1 log(1) L 1 arg(1) L1 2 0 2 2 1 2z i i k i k z i k

1 2z ik k

7.- Escribir exp(2 )z i y 2exp( )iz en función de x e y.

Resol.: Teniendo en cuenta que ez = (ex)y y por tanto |ez| = eRe(z):

2exp(2 ) exp(2 2 ) xz i x i y i e

2 2 2 2 2exp( ) exp ( ) exp ( 2 ) xyiz i x iy i x y i xy e

8.- Demostrar que 2 2exp( ) exp(| |)z z .

Resol.: Teniendo en cuenta que la función exponencial real de variable real es monótona creciente en toda la recta real, y que x, y son reales:

2 22 2 2exp( ) exp( 2 ) x yz x y i xy e

2 22 2 2exp(| |) exp( ) x yz x y e

2 2 2 22 2 2 2 2 20, 0 x y x yx y x y x y e e c.q.d.

9.- Probar que exp( 2 ) 1z si Re(z) > 0.

Resol.: 2 2Re( )exp( 2 ) exp( 2 2 ) 1x zz x i y e e

porque toda función exponencial decreciente (constante del exponente negativa) real de varia-ble real es estrictamente decreciente en toda la recta real, y e0 = 1.

10.- ¿Para qué valores de z se cumple: a) exp( ) 1z ; b) exp( ) 1z ?

Resol.:

a) exp( ) 1 0,xz e z x iy x y

es decir, todo el semiplano complejo de parte real estrictamente negativa.

b) exp( ) 1xz e z iy y

es decir, todo el eje imaginario.

11.- Hallar todas las soluciones de la ecuación sin z = cosh 4 igualando partes real e ima-ginaria. Resol.: Para ello usaré la expresión binomial de sin z, que es: sin sin cosh cos sinhz x y i x y

Se recuerda nemotécnicamente porque suena parecido al sin(x + y), y además se particulariza bien para z = x real. Pero por si acaso la deduzco:

( ) ( )1 1sin

2 2 2

iz izi x iy i x iy y ix y ixe e

z e e e ei i i

1(cos sin ) (cos sin )

2y ye x i x e x i x

i

1

sin cos sin cosh cos ( sinh )2 2

y y y ye e e ei x x x y i x y

i

ok

Page 100: Capítulo 1: Números Complejos

8

Sin[x + I y] // ComplexExpand (* en Mathematica se hace así *) Cosh[y] Sin[x] + I Cos[x] Sinh[y]

Por tanto: sin cosh cosh 4

sin cosh 4cos sinh 0 cos 0 ó sinh 0

x yz

x y x y

cos 0 sin 1 cosh cosh 4 cosh cosh 4 4x x y y y porque el coseno hiperbólico real de variable real solo toma valores positivos, y porque es par (mira su representación geométrica en las transparencias del capítulo). Así que un conjunto de soluciones es x = π/2 + k 2π, y = ±4 (puesto que sin x = +1). Por otra parte:

sinh 0 0 cosh 1 sin cosh 4y y y x x

porque el único cero del seno hiperbólico real es 0, y porque cosh 4 > 1 (como cualquier otro coseno hiperbólico real de variable real salvo el de 0; vuelve a mirar la figura susodicha), con lo que ningún x real tiene ese seno. Por tanto las únicas soluciones son:

( 2 2 ) 4z k i k

Hacerlo así está bien para practicar con los conceptos que hemos manejado arriba, pero vamos también a hacerlo deduciendo la expresión de la función arcsin y aplicándola. Inténtalo por tu cuenta primero… (parar vídeo).

Vale: para la expresión general del arcsin buscamos todos los valores w = arcsin(z) cuyo seno complejo sea z:

2sin( ) 2 ( ) 2 ( ) 1 02

iw iwiw iw iw iwe e

w z e e iz e iz ei

2

2 22 4 41 log 1

2iw iz z

e iz z iw iz z

2arcsin log 1w z i iz z

Mathematica devuelve la determinación principal, no la función multiforme, pero a mí me vale para comprobar que no me he equivocado en las operaciones:

ArcSin[z] // TrigToExp (* salida usa Log por ser func. inversa de Exp *) -I Log[I z + Sqrt[1 - z^2]]

Ahora aplico la fórmula anterior:

2 2arcsin(cosh 4) log cosh 4 1 cosh 4 log cosh 4 sinh 4w i i i i

donde hemos usado que cosh2 z − sinh2 z = 1. Si no recuerdas si era así o al revés, es muy fácil de deducir; o si no también puedes pensar en las gráficas de sinh x y cosh x reales de va-riable real. Seguimos:

2log cosh 4 sinh 4 log cosh 4 sinh 4 log (cosh 4 sinh 4)w i i i i i i i i

4 4 4 4

4cosh 4 sinh 42 2

e e e ee

4 4 4 4 4log L arg( ) arg( ) L( ) 2 2 ( 4)w i ie i ie i ie ie i e k i

con igual resultado que antes.

12.- Resolver la ecuación cos z = 2.

Page 101: Capítulo 1: Números Complejos

9

Resol.: Se puede hacer de cualquiera de las dos maneras utilizadas el apartado anterior. Prefiero usar la fórmula del arccos por ser más general. Primero la deduzco:

Buscamos todos los valores w = arccos(z) cuyo coseno complejo sea z:

2cos( ) 2 ( ) 2 ( ) 1 02

iw iwiw iw iw iwe e

w z e e z e z e

2

2 22 4 41 log 1

2iw z z

e z z iw z z

2arccos log 1w z i z z

Por tanto: 2cos 2 arccos(2) log 2 2 1 log 2 3z z i i

L 2 3 arg 2 3) L(2 3) 2 2 L(2 3)i i i i k k i k

habiendo tenido en cuenta que 2 3 son ambos reales positivos.

Nótese que ahora z es lo que antes llamábamos w, y lo que antes llamábamos z es ahora 2.

Y también nótese que L(2 3) y L(2 3) son opuestos, dado que:

L(2 3) L(2 3) L (2 3)(2 3) L 4 3 L1 0

por lo que la solución se puede escribir igualmente así: 2 L(2 3)k i k .

13.- Demostrar que:

a) 2 2 2sinh sinh sinz x y

b) 2 2

sinh sin( )z iz

c) 2 2 2sin( ) sin sinhy ix y x

Resol.: a) Me vendrá bien usar la forma binómica de sinh z, así que la deduzco:

1sinh (cos sin ) (cos sin )

2 2 2

z z x iy x iyx xe e e e

z e y i y e y i y

cos sin sinh sinh cos cosh sin2 2

x x x xe e e ey i y z x y i x y

Aunque se deduzca fácil, también la recuerdo nemotécnicamente porque suena parecido al sin(x + y), y además veo que se particulariza bien para z = x real.

Sinh[x + I y] // ComplexExpand (* nunca sé cómo lo pondrá Mathematica *) I Cosh[x] Sin[y] + Cos[y] Sinh[x]

Entonces:

2 2 2 2 2sinh (sinh cos ) (cosh sin ) sinh (1 sinz x y x y x y 2) (1 sinh x 2)sin y c.q.d.

b) Se puede hacer de varias maneras. Por ejemplo: por definiciones, sinh(iz) ~ sin(z), sin(iz) ~ sinh(z), cosh(iz) ~ cos(z), cosh(iz) ~ cos(z). Para este, caso nos la segunda dice

exactamente: ( ) ( )

sin( ) sinh2 2 2

i iz i iz z z z ze e e e e eiz i i z

i i

Page 102: Capítulo 1: Números Complejos

10

Sin[I z] I Sinh[z]

Y como el módulo de un producto es el producto de los módulos:

2 2 2 2 2 22sinh sin( ) sin( ) 1 sin( ) sin( )z i iz i iz iz iz c.q.d.

c) Por ejemplo así:

22 2 2 2 2

Apdo. b) Apdo. a)sin( ) sin ( ) sin( ) sinh sinh siny ix i x iy iz z x y c.q.d.

14.- Probar que sinh(z + πi) = −sinh(z), y que cosh(z + πi) = −cosh(z).

Resol.:

( 1) ( 1)

sinh( ) sinh2 2 2 2

z i z i z i z i z z z ze e e e e e e e e ez i z

c.q.d.

( 1) ( 1)

cosh( ) cosh2 2 2 2

z i z i z i z i z z z ze e e e e e e e e ez i z

c.q.d.

Pensemos en la relación de esto con la periodicidad de las funciones sin, cos, sinh, cosh. Las dos primeras son periódicas de período 2π (como en variable real); las otras dos, por su rela-ción con las primeras, son periódicas de período 2πi. Las dos relaciones anteriores, en lugar de avanzar z un período entero, avanzan medio, y entonces, como pasa con sin y cos, en lugar de volver al mismo valor se cambia el signo.

15.- Comprobar que las ecuaciones de Cauchy-Riemann se satisfacen para las funciones 2ze y sinh(4z).

Resol.: Como z no aparece, 0f z en ambos casos y queda comprobado (ver ejercicio 2).

Pero quien puso este ejercicio probablemente quiso que trabajásemos un poco más, así que comprobemos también en coordenadas cartesianas que se cumplen las ecs. de C-R (y, de nue-vo, antes de hacerlo sé que lo harán todo el plano complejo, pues ambas funciones son enteras por ser composición de funciones enteras). Calculando u(x,y), v(x,y) y sus derivadas parciales:

2 2 2 2 2 2 2 2 2 2( ) 2

( , ) ( , )

cos(2 ) sin(2 ) cos(2 ) sin(2 )z x iy x y i xy x y x y x y

u x y v x y

e e e e xy i xy e xy i e xy

2 2 2 2

2 2 2 2

2 cos(2 ) ( sin(2 )2 )

( 2 )sin(2 ) cos(2 )2

x y x yx

x y x yy

u e x xy e xy y

v e y xy e xy x

iguales: ok.

2 2 2 2

2 2 2 2

( 2 ) cos(2 ) ( sin(2 )2 )

2 sin(2 ) cos(2 )2

x y x yy

x y x yx

u e y xy e xy x

v e x xy e xy y

opuestas: ok.

fz = Exp[(x + I y)^2]; u = fz // Re // ComplexExpand

E^(x^2 - y^2) Cos[2 x y] v = fz // Im // ComplexExpand

E^(x^2 - y^2) Sin[2 x y] ux = D[u,x]

2 E^(x^2 - y^2) x Cos[2 x y] - 2 E^(x^2 - y^2) y Sin[2 x y] vy = D[v,y]

2 E^(x^2 - y^2) x Cos[2 x y] - 2 E^(x^2 - y^2) y Sin[2 x y] uy = D[u,y]

-2 E^(x^2 - y^2) y Cos[2 x y] - 2 E^(x^2 - y^2) x Sin[2 x y]

Page 103: Capítulo 1: Números Complejos

11

vx = D[v,x] 2 E^(x^2 - y^2) y Cos[2 x y] + 2 E^(x^2 - y^2) x Sin[2 x y]

ux == vy && uy == -vx True

Para la segunda función usaremos la expresión binomial del seno hiperbólico, que dedu-jimos en el ejercicio 13 apartado a). Sustituyendo z por 4z queda:

( , ) ( , )

sinh(4 ) sinh(4 )cos(4 ) cosh(4 )sin(4 )u x y v x y

z x y i x y

4cosh(4 )cos(4 ) sinh(4 )( 4sin(4 ))

iguales: ok; opuestas: ok.cosh(4 )4cos(4 ) sinh(4 )sin(4 )

x y

y x

u x y u x y

v x y v x y

fz = Sinh[4 (x + I y)]; u = fz // Re // ComplexExpand

Cos[4 y] Sinh[4 x] v = fz // Im // ComplexExpand

Cosh[4 x] Sin[4 y] ux = D[u,x]

4 Cos[4 y] Cosh[4 x] vy = D[v,y]

4 Cos[4 y] Cosh[4 x] uy = D[u,y]

-4 Sin[4 y] Sinh[4 x] vx = D[v,x]

4 Sin[4 y] Sinh[4 x] ux == vy && uy == -vx

True

16.- Resolver las ecuaciones: a) cosh 1 2z b) sinh z i

Resol.: Se puede igualar partes reales e imaginarias, pero es más general, y quizá se presta menos a errores en operaciones, deducir y utilizar las expresiones logarítmicas multiformes de las fun-ciones argumento o arco coseno hiperbólico (que notaré arccosh porque Mathematica usa ArcCosh para la rama principal) y argumento o arco seno hiperbólico multiforme (que notaré arcsinh por análogo motivo):

arccosh cosh 2 02

w ww we e

w z z w e z e

2

2 22 4 4( ) 2 ( ) 1 0 1

2w w w z z

e z e e z z

2arccosh log 1w z z z

ArcCosh[z] // TrigToExp (* solo rama ppal.; se parece lo bastante *) Log[z + Sqrt[-1+z] Sqrt[1+z]]

Page 104: Capítulo 1: Números Complejos

12

arcsinh sinh 2 02

w ww we e

w z z w e z e

2

2 22 4 4( ) 2 ( ) 1 0 1

2w w w z z

e z e e z z

2arcsinh log 1w z z z

ArcSinh[z] // TrigToExp (* solo rama ppal.; se parece más que antes *) Log[z + Sqrt[1 + z^2]]

a) Ahora se trata simplemente de calcular el arco coseno hiperbólico multiforme de 1/2:

3

1 1 1 3 1 3arccosh log 1 log log log 1

2 4 2 4 2 2z z i

3 3L 1 arg 1 L1 3 2 3 2i i k i k k

b) Ahora se trata simplemente de calcular el arco seno hiperbólico multiforme de i:

2arcsinh log 1 log( ) L arg( ) 0 2 2z i i i i i i i i k k

17.- Probar que:

a) Log( ) 12

ei i

b) 1

Log(1 ) L 22 4

i i

c) log( ) 1 2e k i k

d) 1

log( ) 22

i k i k

e) 1log 1 3 L 2 2

3i k i k

Resol.: Lo primero habría que decir que todas las funciones con mayúscula (uniformes) se entiende que son la rama principal; en este caso, del logaritmo, con parte imaginaria en (−, ].

a) La determinación principal del logaritmo tiene parte imaginaria en (−, ]:

Log( ) L Arg( ) L ( 2) 1 2ei ei i ei e i i

b) La determinación principal del logaritmo tiene parte imaginaria en (−, ]:

Log(1 ) L 1 Arg(1 ) L 2 ( 4) (L 2) 2 4i i i i i i

c) El logaritmo multiforme tiene por parte imaginaria un argumento multiforme:

log( ) L arg( ) 1 2e e i e i k k

d) El logaritmo multiforme tiene por parte imaginaria un argumento multiforme:

log( ) L arg( ) L1 ( 2 2 ) 0 (1 2 2 )i i i i i k i k k

e) El logaritmo multiforme tiene por parte imaginaria un argumento multiforme:

2 3log 1 3 log 2 L 2 2 3 2 L 2 2 1 3i i k i k k

Page 105: Capítulo 1: Números Complejos

13

18.- Calcular los logaritmos de los siguientes números complejos: 1, i, 1 + i, 1 − i, 3 + 4i y −2.

Resol.: Supondremos que se refiere a los logaritmos multiformes:

0log(1) log(1 ) L1 (0 2 ) 0 2i k i k k

2log( ) log(1 ) L1 ( 2 2 ) 0 (1 2 2 )i i k i k k

4log(1 ) log( 2 ) L 2 ( 4 2 ) (L 2) 2 (1 4 2 )i i k i k k

4log(1 ) log( 2 ) L 2 ( 4 2 ) (L 2) 2 ( 1 4 2 )i i k i k k

Arctan(4 3)log(3 4 ) log(5 ) L5 arctan(4 3) L5 Arctan(4 3) 2i i i k k

donde arctan(4/3) representa el arco tangente multivaluado en el primer cuadrante (o sea, entre 0 y π/2 + k 2π, pero no entre π y 3π /2 + k 2π; porque el punto −3 − 4i en ningún caso tie-ne igual argumento que el punto 3 + 4i aunque sus tangentes coincidan); y Arctan(4/3) es, por ejemplo, el valor único entre 0 y π/2 (o cualquiera que difiera de él en un múltiplo de 2π, pero no de π, es decir, siempre en el primer cuadrante). El valor numérico de Arctan(4/3) entre −π/2 y π/2 (valor devuelto por Mathematica) es, aproximadamente:

N[ArcTan[4 / 3], 60] 0.927295218001612232428512462922428804057074108572240527621866

pero su valor exacto no tiene ninguna expresión sencilla; de hecho, para Mathematica al me-nos, lo más sencillo para expresar el valor exacto es tal cual:

ArcTan[4 / 3] // FullSimplify ArcTan[4 / 3]

Por último: log( 2) log(2 ) L 2 ( 2 ) L 2 (1 2 )i k i k k

19.- Repetir el ejercicio anterior obteniendo esta vez el valor principal.

Resol.: Bastará copiar, pegar y modificar dejando valores únicos con parte imaginaria en (−π, π]5:

0Log(1) Log(1 ) L1 0 0i

2Log( ) Log(1 ) L1 2 0 2i i i

4Log(1 ) Log( 2 ) L 2 4 (L 2) 2 4i i i

4Log(1 ) Log( 2 ) L 2 ( 4) (L 2) 2 4i i i

Arctan(4 3)Log(3 4 ) Log(5 ) L5 Arctan(4 3)i i

donde Arctan(4/3) representa específicamente el valor en (0, π/2). No bastaría decir “el valor en (−π, π]” (para que la parte imaginaria corresponda a un logaritmo principal) porque hay dos valores distintos de arctan(4/3) en (−π, π], separados por π: uno en el primer cuadrante y otro en el tercero, pero solo el primero es un argumento de 3 + 4i.

5 Y cambiar “log” por “Log”.

Page 106: Capítulo 1: Números Complejos

14

Por último: Log( 2) Log(2 ) L 2 i

donde “casi” podríamos haber elegido −π, pero no.

20.- Resolver la ecuación Log( )2

z i

.

Resol.: Supondremos que Log se refiere a la determinación principal. La parte imaginaria del número dado, π/2, efectivamente pertenece al intervalo (−π, π], como corresponde a dicha determina-ción principal, con lo que puede haber soluciones (de lo contrario no las habría). Por poner un ejemplo, si Log se refiriera a la determinación del logaritmo de parte imaginaria en el interva-lo [−2π, 0), la ecuación no tendría ninguna solución; o si hubiera que resolver Log(z) = i 3π/2 siendo Log la determinación principal, tampoco habría solución. O sea: lo primero es comprobar que la parte imaginaria de Log(z) tiene un valor aceptable6 según la definición de la determinación del logaritmo utilizada. Dicho lo cual:

Por definición de cualquier determinación del logaritmo (natural), eLog(z) = z.

Por tanto: Log( ) 2 cos( 2) sin( 2) 0 1izz e e i i i

Esto es casi como lo de “de qué color es el caballo blanco de Santiago”. Si te resulta lioso, también puedes escribir “sin pensar” (donde Arg y Log son los principales):

2

L 0 1Log( ) L Arg( ) 1

2 Arg 2 ( , ]

z zz z i z i z i

z

21.- Obtener el valor principal del logaritmo de los siguientes números complejos:

exp(2 − 3i), 2

exp4

i

y exp(Log(2 − 9i)).

Resol.: Podemos aplicar fórmulas “casi sin pensar”:

2 3 2 23Log Log L( ) ( 3) 2 3 (pues 3 ( , ] es ppal.)ie e e i i

2

42

4

2 2 2Log Log 1 L1 0 (pues ( , ] es ppal.)

4 4 4

ie i i

((2 + π) / 4 (−π, π] porque 5.1416 / 4 es poco más de 1, y ciertamente menor que π.)

Log(2 9 ) 2 2Log Log(2 9 ) L 2 9 Arg(2 9 ) L 2 9 Arctan( 9 2)ie i i i i i

L 85 Arctan( 9 2) (L85) 2 Arctan(9 2)i i

donde Arctan(±9/2) (−π, π].

O podemos razonar como sigue. Para Log(exp(2 − 3i)) buscamos un número con parte imaginaria en (−π, π] (porque es un logaritmo principal) y tal que, elevando e a ese número, dé e2 − 3i. ¿Qué número habrá que, elevando e a ese número, dé e2 − 3i? Pues eso, 2 − 3i, que

6 Lo cual no tiene por qué definirse en términos de un intervalo fijo al que deba pertenecer: los cortes de las determinaciones del logaritmo tienen que ser curvas que partan del origen del plano complejo (llamado su punto de ramificación), pero no tienen por qué ser semirrectas. De todas formas, si no entiendes esto último, no te preocupes.

Page 107: Capítulo 1: Números Complejos

15

además tiene su parte imaginaria en el intervalo correcto, pues −3 (−π, π]: 2 3Log( ) 2 3ie i como antes.

Lo mismo para 2

Log exp4

i

. Comprobado que (2 + π) / 4 (−π, π], su valor es

i (2 + π) / 4 como antes.

E igualmente Log exp Log(2 9 ) Log(2 9 )i i , pues por definición su parte imagina-

ria está entre −π y π; y Log(2 − 9i) lo hemos calculado antes.

22.- Obtener 3 2

1 3 i .

Resol.: Lo calcularemos multiforme, como base compleja elevada a exponente complejo.

En general, por definición: 2 2 1log( )1 2 1exp log( )z z zz e z z

donde log es el logaritmo multiforme. El que esto dé lugar a uno o más números depende de z2. Por ejemplo, si z2 es un número natural7, la definición anterior da lugar a un único número, que es z1 multiplicado por sí mismo varias veces (ya sabes cuántas). Si es el inverso de un número natural, 1/n, da lugar a las n raíces enésimas de z1. En nuestro caso va a dar lugar a dos números:

3 2

2 3

3 31 3 exp log 1 3 exp log 2

2 2i i

3 2 3 3 2

exp L 2 2 exp L 2 22 3 2 2 3

i k i k

3

L2 3 3 223

exp L 2 3 2 ( 1) 8 2 22

i ki k e e

Estos dos números también se pueden calcular como las dos raíces cuadradas de 3( 1 3 )i , pero esto solo lo sabes si sabes lo que pasa cuando aplicas la definición general de base com-pleja elevada a exponente complejo con z2

= 3/2. Así que puedes desarrollar el cubo de ese binomio y luego calcularle sus raíces cuadradas. Me gusta más lo de arriba por ser más gene-ral, pero dejaré que Mathematica haga lo otro (y así compruebo):

(-1 + Sqrt[3] I)^3 // ComplexExpand 8

y obviamente sus dos raíces cuadradas son el resultado de antes.

23.- Hallar el valor principal de 3

1 32

ie

A i

.

Resol.: Como base compleja elevada a exponente complejo (multivaluado):

7 Dependiendo del área de las matemáticas, los números naturales empiezan en el 0 o empiezan en el 1. No hay una definición universal de .

Page 108: Capítulo 1: Números Complejos

16

3

2 31 3 exp 3 Log 1 3 exp 3 Log2 2

ie e

A i i i i e

2 2 2

exp 3 L exp 3 1 exp 3 33 3 3

i e i i i i

2 22 2 3 2exp 2 3 ii e e e

(E / 2 (-1 - Sqrt[3] I))^(3 Pi I) // ComplexExpand -E^(2 Pi^2)

24.- Hallar arcsin i.

Resol.: Como viene con minúscula, supondremos arco seno multiforme. Su expresión general la de-dujimos en el ejercicio 11:

2arcsin log 1z i iz z

En este caso: 2arcsin log 1 log( 1 2)i i ii i i

Por un lado: log( 1 2) L( 1 2) 2 2 L( 1 2)i i ik k i

y por otro: log( 1 2) L(1 2) ( 2 ) 2 L(1 2)i i i k k i

Podremos simplificar un poco más si nos damos cuenta de lo siguiente (algo parecido sucede a menudo al evaluar funciones trigonométricas e hiperbólicas inversas):

L( 1 2) L(1 2) L ( 1 2)(1 2) L1 0 L( 1 2) L(1 2)

Susti: 2 L(1 2)

arcsin arcsin ( 1) L(1 2)2 L(1 2)

kk ii i k i k

k i

Table[Sin[k Pi + I (-1)^k Log[1 + Sqrt[2]]], {k, -3, 3}] // Simplify {I, I, I, I, I, I, I}

25.- Hallar (todos los valores): a) arctan(2i) b) arccosh(−1) c) arctan(0)

Resol.: a) La expresión logarítmica del arctan multiforme aún no la hemos deducido:

sin ( ) (2 )

arctan( ) tan( ) ( )cos ( ) 2

iw iwiw iw iw iw

iw iw

w e e iw z z w e e iz e e

w e e

2 2 2 21 ( 1) (1 ) 1 (1 ) (1 ) ( ) ( )iw iw iw iwe iz e iz e iz e iz iz i z i z

1

2 log log log log arctan2 2 2

i z i z i i z i i ziw w z

i z i i z i z i z

Sigue un paréntesis (hasta el ■) con una serie de cosas que viene bien saber sobre el compor-tamiento de estas funciones. Por ejemplo, al final de la última línea hemos hecho uso (implíci-tamente) de lo siguiente:

log[(i − z) / (i + z)] = log(i − z) − log(i + z) = −[log(i + z) − log(i − z)] = −log[(i + z) / (i − z)]

Page 109: Capítulo 1: Números Complejos

17

lo cual hace uso de que log(z1 / z2) = log(z1)

− log(z2) (z2 ≠ 0). Esto es algo que hay que

saber y además es fácil demostrar:

1 2 1 2 1 2 1 2 1 2 1 2log L arg L arg( ) arg( ) L Lz z z z i z z z z i z z z z

1 2 1 1 2 2 1 2arg( ) arg( ) L arg( ) L arg( ) log( ) log( )i z i z z i z z i z z z

Recuérdese que esta no es una igualdad entre números, sino entre conjuntos: el de todos los valores log(z1

/ z2) es igual al de todos los que se pueden obtener como diferencia de un log(z1) y un log(z2). En otras palabras: cualquiera de los infinitos log(z1

/ z2) es igual a alguno de los infinitos log(z1) menos alguno de los infinitos log(z2); y cualquiera de los infinitos log(z1) me-nos cualquiera de los infinitos log(z2) es igual a alguno de los infinitos log(z1

/ z2). (Igual que interpretamos arg(z1

/ z2) = arg(z1)

− arg(z2), que es algo que hemos usado en la demostración.)

Por cierto, de igual manera, log(z1 z2) = log(z1)

+ log(z2) (demuéstrelo el lector). Sin em-bargo, por ejemplo (y cuidado con esto!), log(z2) ≠ 2 log(z). En efecto:

22 2 2log( ) L arg( ) L 2 Arg( ) 2 2 L 2 Arg( ) 2z z i z z i z k z i z k

2log( ) 2 L arg( ) 2L 2 Arg( ) 2 2L 2Arg( ) 4z z i z z i z k z i z k

Vemos que log(z2) tiene más valores! Sin embargo se podría pensar que “haciendo z1 = z2

= z se obtiene log(z2) = log(z) + log(z) = 2 log(z)”. El error de este razonamiento está en que log(z) + log(z) no es 2 log(z) porque hay que interpretarlo en términos de igualdades entre con-juntos: el primer log(z) y el segundo log(z) de la expresión log(z) + log(z) pueden ser distintos logaritmos de la misma z, y por tanto dar algo que no sea el doble de ningún log(z).

También viene bien saber que las dos raíces cuadradas de z son los valores multiformes de z1/2 como base compleja elevada a exponente complejo. En general, los valores multiformes de z1/n con n natural son las n raíces n-ésimas de z:

1L1Arg( ) 2arg( )

1 1exp log exp L arg( )

nz nn nz kznn

z z z i z e z zn n

Y viene bien saber que log(z1/2) = 1/2 log(z). Esto, de nuevo, se debe interpretar como igualdad de conjuntos, es decir: el de todos los logaritmos de ambas raíces cuadradas de z es igual al de las mitades de todos los logaritmos de z. O bien: cualquiera de los infinitos loga-ritmos de cualquiera de las dos raíces cuadradas de z es la mitad de alguno de los logaritmos de z; y la mitad de cualquiera de los logaritmos de z es alguno de los logaritmos de alguna de las dos raíces cuadradas de z. Y de igual manera, log(z1/n) = 1/n log(z). Demuéstrelo el lector. ■

Pero estamos divagando; volvamos al cálculo de arctan(2i). Usando la expresión logarítmica multiforme del arctan deducida antes (y que, por cierto, también se puede escribir de otras maneras parecidas):

2 3arctan(2 ) log log log 3 L 3 arg( 3)

2 2 2 2 2

i i i i i i ii i

i i i

L3 L3L3 ( 2 )

2 2 2 2 2

ii k i k k i k

Obsérvese que al final hemos cambiado −π/2 − k π por π/2 + k π, para simplificar un poco la respuesta. Podemos hacerlo porque, de nuevo, se trata de conjuntos: el conjunto de todos los valores −π / 2 − k π para todos los posibles enteros k es el mismo conjunto de valores π / 2 + k π para todos los posibles enteros k.

Page 110: Capítulo 1: Números Complejos

18

Mathematica da una expresión ligeramente distinta del arctan (aparte de que solo dé el valor principal) pero equivalente:

ArcTan[z] // TrigToExp 1/2 I Log[1 - I z] - 1/2 I Log[1 + I z]

Usándola: 3arctan(2 ) log 1 (2 ) log 1 (2 ) log(3) log( 1) log

2 2 2 2 2 1

i i i i ii i i i i

y a partir de aquí como antes.

b) La expresión logarítmica multiforme del arccosh ya la dedujimos en el ejercicio 16:

2 2arccosh log 1 arccosh( 1) log 1 ( 1) 1 log 1 0z z z

L 1 arg( 1) L1 ( 2 ) 0 ( 2 ) (1 2 )i i k i k i k k

c) Ya sabemos por la “teoría” que la tangente, como el seno y el coseno, tiene en el campo complejo los mismos ceros que en el real. Pensando en la circunferencia goniométrica vemos claramente que estos son 0, ±π, ±2π, ±3π… o bien k π k . Confirmémoslo aplicando la

expresión logarítmica multiforme del arctan, recientemente deducida:

0arctan(0) log log(1) L 1 arg(1) L1 2

2 0 2 2 2

i i i i ii i k

i

0 22

ii k k k k

Los ceros de las funciones trigonométricas e hiperbólicas también salen fácil igualando partes reales e imaginarias en vez de evaluando sus funciones inversas en z = 0. Por ejemplo, los ceros de la tangente, al ser seno entre coseno, serán los ceros del seno. Utilizando la expresión binomial del seno, que dedujimos al principio del ejercicio 11:

sin cosh 0

sin sin cosh cos sinh 0cos sinh 0

x yz x y i x y

x y

En la primera ecuación cosh y nunca es cero (x, y son reales), con lo que solo puede ser x = k π. Sustituyendo en la segunda, cos x no puede ser cero (será ±1), con lo que necesa-riamente sinh y = 0, cuya única solución en el campo real es y = 0. De nuevo llegamos a las mismas raíces de la tangente (o del seno), k π.

Si quisiéramos calcular arctanh(0)8 haríamos parecido. Podemos aplicar la relación entre las funciones tanh y tan:

sinh( ) ( ) 2 ( ) (2 ) sin( )

tanh( ) tan( )cosh( ) ( ) 2 ( ) 2 cos( )

iz iz iz iz

iz iz iz iz

iz e e e e i ziz i i i z

iz e e e e z

Tanh[I z] (* no hace falta ni pedir que simplifique *) I Tan[z]

Por tanto los valores que anulen la función tan (los ceros de la tangente, que son los del cam-po real, k π), multiplicados por i, anularán la función tanh. Además por la “teoría” sabemos que son los únicos ceros (los de sinh, cosh y tanh son los mismos de sin, cos, tan, respectiva-

8 Cosa que sospecho porque en los apuntes la solución que aparece es kπi y además en el enunciado pone argtan(0), no arctan(0), de modo que la errata está probablemente en la pregunta y no en la res-puesta.

Page 111: Capítulo 1: Números Complejos

19

mente, multiplicados por i, es decir, sobre el eje imaginario). Pero deduzcamos y apliquemos también la expresión logarítmica multiforme de la función arctanh:

sinh ( ) 2

arctanh( ) tanh( ) ( )cosh ( ) 2

w ww w w w

w w

w e ew z w z e e z e e

w e e

2 1 1(1 ) (1 ) ; ( ) ;

1 1w w w wz z

z e z e e ez z

1 2

1 1 1 1log log log arctanh

1 1 2 1

z z zw z

z z z

ArcTanh[z] // TrigToExp (* solo da valor principal, pero concuerda *) -(1/2) Log[1 - z] + 1/2 Log[1 + z]

Aplicándola:

1 1 0 1 1 1arctanh(0) log log(1) L 1 arg(1) 0 2

2 1 0 2 2 2i i k i k k

Finalmente lo haremos también igualando a cero las partes real e imaginaria de la expresión binomial de sinh(z), que dedujimos en el ejercicio 13 apartado a):

sinh cos 0

sinh sinh cos cosh sin 0cosh sin 0

x yz x y i x y

x y

En la 2ª ecuación cosh x > 0, con lo que y = k π; y sustituyendo en la primera, cos y = ±1, con lo que sinh x = 0 y por tanto x = 0, llegándose a los mismos ceros k π i.

26.- Resolver sin z = 2: a) Identificando partes reales e imaginarias. b) Utilizando la expresión logarítmica del arcsin z.

Resol.: a) La expresión binomial del sin z la dedujimos en el ejercicio 11. Igualando su parte real a 2 y su parte imaginaria a cero:

sin cosh 2

cos sinh 0 cos 0 ó sinh 0

x y

x y x y

sinh 0 0 cosh 1 sin 2

cos 0 sin 1 cosh 2 cosh 2 2 2

y y y x x

x x y y x k

2cosh 2 4 ( ) 4 1 02

y yy y y ye e

y e e e e

4 16 4

2 12 4 2 3 L(2 3) L(2 3)2

ye y

dado que L(2 3) L(2 3) L (2 3)(2 3) L(4 3) L1 0 .

Susti: 2 L(2 3) 2 L(2 3)2 2

z x iy k i k i k

ArcSin[2] // ComplexExpand (* solo va a dar el valor principal *) Pi / 2 - I Log[2 + Sqrt[3]]

Page 112: Capítulo 1: Números Complejos

20

Solve[Sin[z] == 2, z] // ComplexExpand (* todas las soluciones *) {{z->ConditionalExpression[ Pi / 2 + 2 Pi C[1] + I Log[2 + Sqrt[3]], C[1]\[Element]Integers]}, {z->ConditionalExpression[ Pi / 2 + 2 Pi C[1] - I Log[2 + Sqrt[3]], C[1]\[Element]Integers]}}

b) La expresión logarítmica multiforme del arco seno la obtuvimos en el ejercicio 11. Sus-tituyendo:

2arcsin 2 log 2 1 2 log 2 3 log (2 3)z i i i i i i

L (2 3) arg (2 3) arg (2 3) L(2 3) 2 L(2 3)2

i i i i i i k i

como antes.

27.- Hallar todos los valores de z que verifiquen las siguientes ecuaciones:

1

a) sinh 0 b) cosh 0

c) 0 d) 1 0

e) cos cosh 4 f) sin cosh 4

z z

z z

e e i e

z z

Resol.: Ya les hemos dado muchas vueltas a este tipo de problemas, así que ahora responderé más expeditivamente:

a) Sabemos por la teoría que los ceros del seno hiperbólico son los del seno (todos

reales) multiplicados por i (sobre el eje imaginario): z i k k

b) Sabemos por la teoría que los ceros del coseno hiperbólico son los del coseno (todos

reales) multiplicados por i (sobre el eje imaginario): ( 2 )z i k k

c) 1 1 1 1log( ) L arg( ) 1 2ze e z e e i e i k k

d) 4log(1 ) log( 2 ) L 2 4 2 L 2 2 4 2z i i k i k k

e) Aplicando la expresión logarítmica del arccos, que dedujimos en el ejercicio 12:

2arccos(cosh 4) log cosh 4 cosh 4 1 log cosh 4 sinh 4z i i

4 4 4 4

4 4 4log log L( ) arg( ) 4 22 2

e e e ei i e i e i e i i k

2 4k i k

ArcCos[Cosh[4]] (* solo valor principal *) 4 I

Solve[Cos[z] == Cosh[4]] (* todas las soluciones *) {{z->ConditionalExpression[-4 I + 2 Pi C[1], C[1]\[Element]Integers]}, {z->ConditionalExpression[ 4 I + 2 Pi C[1], C[1]\[Element]Integers]}}

f) Esa es la ecuación del ejercicio 11. Supongo que sigue teniendo las mismas soluciones:

( 2 2 ) 4z k i k

28.- Hallar el valor principal de las siguientes potencias complejas: 1i, (−1)i, ii, iLog(1+i).

Page 113: Capítulo 1: Números Complejos

21

Resol.:

Log11 exp L 1 Arg(1) exp 0 1i ie i i

Log( 1)( 1) exp L 1 Arg( 1) exp Arg( 1) exp( )i ie i i e

Log( ) 2exp L Arg( ) exp Arg( ) exp( 2)i i ii e i i i i i e

Log(1 ) Log(1 )Log4 2exp Log( 2 ) Log(1 ) exp L 2 4 L1 2i i ii e i i

22

8L2

4

L 2 L 2exp 0 exp

2 4 2 8 4i i i e

Como Mathematica siempre devuelve valores principales, comprobar estos resultados es más directo que otros multiformes anteriores:

1^I 1

(-1)^I // ComplexExpand E^-Pi

I^I // ComplexExpand E^(-Pi / 2)

I^Log[1 + I] // ComplexExpand // Simplify E^(-Pi^2 / 8) (Cos[1/4 Pi Log[2]] + I Sin[1/4 Pi Log[2]])

29.- Hallar todos los valores de: arcsin 2, arctanh i y arctan(1 + i).

Resol.: El arcsin 2 multiforme ya lo calculamos en el ejercicio 26 (de dos modos). Supongo que seguirá valiendo lo mismo:

2 L(2 3) 2 L(2 3)2 2

k i k i k

Para arctanh i usaremos la expresión logarítmica multiforme del arctanh que deduji-mos en el ejercicio 25, apartado c):

42

4

21 1 1 1 1arctanh log log log 1 L1 2

2 1 2 2 2 22

ii i k

i

4

i k k

Para arctan(1 + i) usaremos la expresión logarítmica multiforme del arctan que dedu-jimos en el ejercicio 25, apartado a). Aquí Arctan(2) es el valor entre −π/2 y π/2 (que estará entre 0 y π/2):

Arctan 21 2 1

arctan(1 ) log log log 1 2 log 52 1 2 1 2 2

i i i i i i ii i

i i

Arctan 2 2 L5L 5 Arctan 2 2

2 2 4

i ki k i

Arctan 2 L5

2 4k i k

30.- Calcular las partes real e imaginaria de: sin(1 + i), cos(1 − i) y tan(1 + 2i).

Page 114: Capítulo 1: Números Complejos

22

Resol.: Para sin(1 + i) usaremos la forma binomial del seno, que dedujimos al principio del ejercicio 11:

1 1

sin(1 ) sin1cosh1 cos1sinh1 sin1 cos12 2

e e e ei i i

.

Para cos(1 − i) usaremos la forma binomial del coseno, que deducimos aquí:

( ) ( ) (cos sin ) (cos sin )

cos( )2 2 2

i x iy i x iy y ix y ix y ye e e e e x i x e x i xz

cos sin cos cos cosh sin sinh2 2

y y y ye e e ex i x z x y i x y

que se recuerda mnemotécnicamente por su parecido con el cos(x + y), particularizándose bien para z = x real, y que se comprueba con Mathematica:

Cos[x + I y] // ComplexExpand Cos[x] Cosh[y] - I Sin[x] Sinh[y]

Con z = 1 − i: cos(1 ) cos1cosh( 1) sin1sinh( 1)i i

1 1

cos1cosh1 sin1sinh1 cos1 sin12 2

e e e ei i

Para tan(1 + 2i) usaremos la forma binomial de la tangente, que deducimos ahora.

sin sin cosh cos sinh

tancos cos cosh sin sinh

z x y i x yz

z x y i x y

Para escribir menos empezaré a usar sh, ch en lugar de sinh, cosh. En cualquier caso recuerda que: a) Mathematica usa Sin, Cos, Tan, Sinh, Cosh, Tanh, ArcSin, ArcCos,

ArcTan, ArcSinh, ArcCosh, ArcTanh; b) aunque las escribamos con minúsculas, estas funciones son siempre univaluadas (y por eso no hay problema en escribirlas así: no da lugar a posible confusión).

Multiplicando y dividiendo por el conjugado del denominador, y teniendo en cuenta que sin2x + cos2x = 1 y que ch2x − sh2x = 1:

sin ch cos sh cos ch sin sh

tancos ch sin sh cos ch sin sh

x y i x y x y i x yz

x y i x y x y i x y

2 2 2 2

2 2 2 2

sin cos ch sin cos sh (cos sh ch sin sh ch )

cos ch sin sh

x x y x x y i x y y x y y

x y x y

2 2 2 2

2

sin cos (ch sh ) (cos sin )sh ch

(1 sin

x x y y i x x y y

x

2 2 2) ch sin ( chy x y 2 2 2 2

sin cos sh chtan

ch sin ch sin1)

x x y yz i

y x y x

Sustituyendo x = 1, y = 2: 2 2 2 2

sin1cos1 sh 2ch 2tan(1 2 )

ch 2 sin 1 ch 2 sin 1i i

que es una primera manera de expresar la solución. Pero, para empezar, el denominador común de antes, 2 2 2 2cos ch sin shD x y x y , también se puede poner de otras maneras:

2 2 2 2 2 2 2 2 2 2cos (1 sh ) sin sh cos sh (cos sin ) cos shD x y x y x y x x x y

Page 115: Capítulo 1: Números Complejos

23

2 2 1 ch(2 ) 1 cos(2 ) cos(2 ) ch(2 )ch sin

2 2 2

y x x yD y x

Y los numeradores también; por ejemplo:

1 2sin cos sin(2 ) 2 ; sh ch sh(2 ) 2N x x x N y y y

Las tres últimas expresiones están en términos de 2x, 2y. A menudo es útil escribir expresio-nes trigonométricas (tanto circulares como hiperbólicas) en función del “ángulo” doble, pues suelen resultar más fáciles de integrar, derivar o manipular. Así, si decidimos escribir tan z en función de 2x y de 2y tendríamos:

1 2 sin(2 ) 2 sh(2 ) 2tan

cos(2 ) ch(2 ) 2 cos(2 ) ch(2 ) 2

N N x yz i i

D D x y x y

sin(2 ) sh(2 )

tancos(2 ) ch(2 ) cos(2 ) ch(2 )

x yz i

x y x y

que, por cierto, es la expresión que elige Mathematica devolver por defecto:

Tan[x + I y] // ComplexExpand Sin[2 x] / (Cos[2 x] + Cosh[2 y]) + I Sinh[2 y] / (Cos[2 x] + Cosh[2 y])

Utilizándola, la respuesta a este último ejercicio quedaría:

sin 2 sh 4

tan(1 2 )cos 2 ch 4 cos 2 ch 4

i i

Al final del documento 0_Trigonometría_repaso.pdf hay un apartado dedicado a la trigono-metría hiperbólica con algunas relaciones comunes. Como ejercicio adicional, de igual mane-ra que hemos deducido la expresión binomial de la tangente de z en términos de 2x y 2y, haz tú lo mismo para la tangente hiperbólica. Lo que sale es:

sh(2 ) sin(2 )

tanhcos(2 ) ch(2 ) cos(2 ) ch(2 )

x yz i

y x y x

Tanh[x + I y] // ComplexExpand I Sin[2 y] / (Cos[2 y] + Cosh[2 x]) + Sinh[2 x] / (Cos[2 y] + Cosh[2 x])

Page 116: Capítulo 1: Números Complejos

Capítulo 3: Funciones de Variable Compleja. Resolución de ejercicios de pasados exámenes. AVISO IMPORTANTE: Antes de seguir lee este AVISO.

Resoluciones completas:

Ejercicio 1 (de examen de junio de 2014):

Hallar todos los puntos singulares de la función 2 1( )

tan( ) 3 2zf z

z i−

=− +

.

Resol.: Rápido recordatorio teórico1: un punto singular de una función es cualquier punto (aislado o no) en que esa función no sea analítica (por la razón que sea). Una función es analítica en un punto ssi2 es derivable no solo en él, sino en algún entorno de él (por pequeño que sea su ra-dio—aunque por definición no puede ser cero). Si una función no está definida en un punto, o si no es continua en él, no puede ser ni derivable ni por tanto analítica en él. Una función es analítica en un conjunto de puntos ssi es analítica en todos ellos. Dominio de definición de una función (y no tiene por qué ser un domino, que es un conjunto conexo y abierto) es el conjunto de todos los puntos donde está definida. Dominio de analiticidad (que tampoco tiene por qué ser un dominio, aunque sí un conjunto abierto) es el conjunto de todos los puntos donde la función es analítica. El dominio D de analiticidad de una función y el conjunto S de sus puntos singulares son complementarios en el plano complejo, porque todo punto o es un punto de analiticidad de la función o es un punto singular, y no hay más opciones. La compo-sición, suma, producto, combinación lineal de funciones analíticas (en un punto o conjunto) es otra función analítica (en ese punto o conjunto); el cociente también, salvo en los ceros del denominador. Por ejemplo, los polinomios son funciones enteras (analíticas en todo el plano complejo), de modo que los cocientes de polinomios (funciones racionales) son analíticas en todo el plano complejo salvo en los ceros del denominador.

En la función f (z) dada supondremos raíces cuadradas principales, que son las obtenidas co-mo potencia compleja univaluada (base compleja elevada a exponente complejo) en cuyo cálculo se usa el logaritmo principal (de parte imaginaria en (−π, π]). Así, la raíz cuadrada principal de z es: ( ) ( )1 21 2 L Arg( ) L 2 Arg( ) 21 2 1 2Log( ) Arg( ) 2

Arg( ) 2

z i z z i zz i z

zz z e e e z e z+ += = = = = = +

donde e• es exp(•) (la función exponencial univaluada), L es la función logaritmo neperiano real de variable real, Log es el logaritmo principal y Arg el argumento principal (en (−π, π]). Vemos que, así definida, la raíz cuadrada principal está definida en todo el plano complejo, pero deja de ser analítica en el semieje real negativo (incluido el 0). Por ejemplo, la raíz cua-drada principal de −9 es 3i (argumento principal Arg(−9) = π, luego Arg(−9) / 2 = π/2) pero la de −9 − 0.001i es casi −3i (argumento principal Arg(−9 − 0.001i) ≈ −π, luego Arg(−9 − 0.001i) / 2 ≈ −π/2). Un cambio muy pequeño en z (de −9 a −9 − 0.001i) produce un cambio “grande” en su raíz cuadrada principal (salto de tamaño aproximado 6i). Por tanto la raíz cuadrada principal no es continua en −9 (y lo mismo en cualquier otro núme-

1 Sáltate esto si te has estudiado bien la teoría. O sea, no te lo saltes. 2 Si y solo si.

Page 117: Capítulo 1: Números Complejos

2

ro real negativo) y, no siendo continua, no puede ser derivable ni analítica en −9 (como sí lo es en cualquier otro punto del plano complejo fuera del semieje real negativo o cero3).

Dicho lo cual: el numerador de f (z) es 2 1z − (raíz cuadrada principal). Dado que z deja de ser analítica en el semieje real negativo, 2 1z − dejará de ser analítica en aquellos puntos z que hagan que z2 − 1 sea real negativo. Desde ahí, mínimos cambios en z harán que z2 − 1 “salte” a ambos lados del semieje real negativo, y por tanto la raíz cuadrada principal 2 1z − sufra saltos discretos entre el semieje imaginario positivo y el negativo, siendo discontinua. Por el contrario, en todo punto z donde z2 − 1 no sea real negativo o cero, 2 1z − será analíti-ca, porque la composición de funciones analíticas en un punto o conjunto es analítica en él.

En cuanto al denominador, es una función analítica salvo en los puntos en que tan z no lo es. Por tanto las únicas singularidades de f adicionales a las del numerador serán los ceros de cos z (puntos singulares de tan z, en que ni siquiera está definida) y los ceros del denominador.

Empecemos por el numerador. Podemos buscar los valores de z que hacen que z2 − 1 ≤ 0 operando de más de un modo. Por ejemplo, llamando p (de “positivo”) a un número real posi-tivo o cero (p ≥ 0): 2 21 1 1z p z p z p− = − ⇒ = − ⇒ = ± − y un pequeño dibujo nos muestra los posibles valores de z:

Figura 1. Gráfica para saber rápidamente dónde 2 1z − no es analítica.

Primero dibujamos la semirrecta 1 − p y enseguida vemos dónde pueden estar todas sus raíces cuadradas: [ 1,1] 0z x∈ − ∪ =

Por cierto, vamos a ir dejando subrayados todos los puntos singulares que vayamos encon-trando para al final juntarlos todos y recuadrar el resultado.

O podemos operar analíticamente, sin dibujar nada:

2 2 2 22 2

2 0 0 ó 01 ( ) 1 1 2 0

1 0xy x y

z x iy x y i xyx y

= ⇒ = =− = + − = − − + ≤ ⇔

− − ≤

2 20 1 0 ; 0 1 0 [ 1,1]x y y y x x= → − − ≤ ⇒ ∀ ∈ = → − ≤ ⇒ ∈ − con idéntico resultado.

Respecto al denominador, los puntos singulares de tan z = sin z / cos z son los ceros del coseno que, como sabemos por la teoría, son solo los que tiene en :

tan cos 0 2z z z kπ π⇔ = ⇔ = +

3 En z = 0 la raíz cuadrada principal tampoco es analítica, porque cualquier entorno de z = 0 contiene números reales estrictamente negativos en los que no es continua y, por tanto, tampoco derivable.

Page 118: Capítulo 1: Números Complejos

3

Y además hay que encontrar los ceros del denominador:

tan( ) 3 2 0 arctan( 3 2 )z i z i− + = ⇔ = −

Ahora deducimos la fórmula logarítmica multiforme de la función arctan. Lo hacemos para una z y una w genéricas (no el valor de los puntos z singulares en este ejercicio):

sin ( ) (2 )arctan( ) tan( ) ( )cos ( ) 2

iw iwiw iw iw iw

iw iw

w e e iw z z w e e iz e ew e e

−− −

−= ⇒ = = = ⇒ − = +

+

2 2 1(1 ) (1 ) (1 ) (1 )1

iw iw iw iw iz i ziz e iz e iz e iz eiz i z

− + −⇒ − = + ⇒ − = + ⇒ = = ⇒

− +

12 log log log log arctan2 2 2

i z i z i i z i i ziw w zi z i i z i z i z− − − − + = ⇒ = = = = + + + −

En esta última expresión sustituimos z por 3 2i− para encontrar los puntos singulares del denominador (a los cuales, con abuso de notación, volvemos a llamar z):

( ) ( 3 2 ) 3 3 3arctan 3 2 log log2 2( 3 2 ) 3 3 3 3i i i i i iz i

i i i i + − − − −

= − = = ⋅ = − − − + − −

3 3 ( 3 3 3) 6 2 3 3 3log log log2 3 9 2 12 2 6i i i i i i − − + − − − − −

= = = = +

9 3 2 3 3L arg( 3 3 ) L Arctan 22 6 2 6 3i ii i i kπ π

+ = + − − = + − + =

1 21 1 5 5 L3L 2 L3 22 6 2 2 6 12 43i ii k k k i kπ π ππ π π π− − = + − + = − + = + − ∀ ∈

Aquí, teniendo en cuenta que 3 3 i− − , está en el tercer cuadrante, hemos escrito su argu-mento multiforme arg( 3 3 )i− − como (Arctan( 3 3) ) 2kπ π− + , porque la función Arctan da el valor entre 0 y π /2 (en general, entre −π /2 y π /2), y restándole π (también podríamos haberle sumado π) obtenemos un argumento correspondiente al tercer cuadrante. Y hemos cambiado −kπ por kπ porque la expresión es ∀ k ∈

.

Y no hay más puntos singulares que los que hemos ido subrayando. Poniéndolos juntos, el conjunto S de puntos singulares de f (z) es:

{ } 5 L3[ 1,1]2 12 4

S iy y k k k i kπ ππ π = − ∪ ∈ ∪ + ∀ ∈ ∪ + − ∀ ∈

Representar este conjunto en el plano complejo es fácil pero un poco pesado; como no se pi-de, no lo hago.

Page 119: Capítulo 1: Números Complejos

4

Ejercicio 2 (de examen de enero de 2014): i) Resolver la siguiente ecuación para los distintos valores de α:

cosh( ) 0 con ( 1)2

iziz eα α α− = ∈ ≠

ii) Hallar el mayor dominio de analiticidad de 1( )f z zz

= + .

Resol.:

i) 2

iz ize e−+2α

− 2 2 1 10 ; (1 ) ; (1 ) 1;1 1

iz iz iz iz ize e e e eα αα α

− −= − = − − = − = =

− −

1 1 1 12 log ; log log1 2 1 2 1

iiz ziα α α

− = = = − − −

( )1 L( 1)1: L (2 ) L( 1) 2

2 1 2 21 L(1 )1: L 2 L(1 ) 2

2 1 2 2 2 4 2

i iz i k i k k i

i ii k i k k i

αα π α π πα

π π π αα π α π πα

− − − > = + = − − + = + − − − − < + + = − − + + = + + −

L( 1) ( 1)2L(1 ) ( 1)

4 2

k iz k

k i

απ α

π απ α

− + >= ∀ ∈ − + + <

ii) La función f (z) es analítica en la mayoría del plano complejo; por ello una manera or-denada de proceder en este tipo de ejercicios es ir encontrando sus puntos singulares y dejar-los subrayados para luego escribirlos todos juntos. Además, aunque no se pida, vamos a re-presentarlos gráficamente, de modo que también vamos a ir dibujándolos según los encontre-mos.

Para empezar, en el punto z = 0 la función ( ) 1f z z z= + no está definida4, así que ese es un primer punto singular (que dibujo).

Además, suponiendo raíz cuadrada principal, f (z) dejará de ser analítica en los puntos z que hagan que z + 1/z ≤ 0 (o sea, real negativo o cero). Para una explicación sobre de esto, ver ejercicio 1. Aquí simplemente vamos a buscar dónde se produce eso:

2 2 2 2 2 2

1 1 0x iy x iy x yz x iy x iy x i yz x iy x iy x y x y x y

− −+ = + + ⋅ = + + = + + − ≤ ⇒ + − + + +

2 22 2 2 2

2 ( 0 "visto":supongo 0)2 2

2 2 2 2

10 0 ó 1 0 1

Si 0 : 0 0 (y dibujo 0, 0)0

Si 1: 2 0 0 (y dibujo 1, 0)

zx

yy y x yx y x y

xy x x y xx xxx y

x y x x x y x

=≠

− = ⇒ = − = ⇒ + = + + = + ≤ ⇔ < = < + ≤ ⇒ + + = ≤ ⇔ ≤ + = ≤

4 En el plano complejo extendido la división por 0 daría ∞ (un único número complejo “nuevo”), pero no es esto lo que vamos a entender habitualmente por .

Page 120: Capítulo 1: Números Complejos

5

Por tanto el máximo dominio de analiticidad D de f (z) sería:

{ }2 2( ,0] 1, 0D x iy x y x= − −∞ − + + = ≤

D es todo el plano complejo excepto los puntos singulares representados en la figura5:

Figura 2. Puntos singulares, complementarios del dominio de analiticidad D.

Ejercicio 3 (de examen de junio de 2013): i) Obtener (demostrando) la expresión logarítmica de w = arccos z.

ii) Hallar los puntos singulares de 2

2

3( )(cos 2 )( )z

zf zz i e i

+=

− +.

Resol.:

i) Por definición de la función coseno: cos2

iw iwe ew−+

= .

Por definición de arco coseno multiforme, se trata de encontrar todos los puntos w cuyo co-seno sea un número dado z, es decir, de encontrar todas las soluciones en w de la ecuación:

2

iw iwe e z−+

=

Multiplicar ambos lados por 2 eiw no añade soluciones, porque la función exponencial ez no vale 0 en ningún punto del plano complejo:

2( ) 2 ( ) 2 1 0iw iw iw iw iw iwe e e e z e ze−+ = ⇔ − + =

Aquí hemos hecho eiw e−iw = e0 = 1 porque la función exponencial sigue cumpliendo, en el plano complejo como en la recta real, que 1 2 1 2z z z ze e e += .

La anterior es una ecuación cuadrática en eiw cuyas dos únicas soluciones son:

2

22 4 4 12

iw z ze z z± −= = ± −

Ahora buscamos todos los números iw en los que la función exponencial valga 2 1z z± − . Eso es, por definición de logaritmo multiforme:

( )2log 1iw z z= ± −

5 La figura está hecha enteramente a mano alzada (sin compás ni regla; solo con un boli borrable y un poco de cuidado). La circunferencia sale bien y muy rápido con esta técnica.

Page 121: Capítulo 1: Números Complejos

6

Finalmente, multiplicando ambos lados por −i:

( )2arccos log 1w z i z z= = − ± −

donde log( ) L arg( )z z i z= + , siendo L la función logaritmo neperiano real de variable

real, z el módulo de z, y arg(z) su argumento multiforme (infinitos valores separados por múltiplos de 2π).

ii) La función 2

2

3( )(cos 2 )( )z

zf zz i e i

+=

− + solo dejará de ser analítica (puntos singulares):

a) allá donde no esté definida (en este caso, por ser cero el denominador, pues todas las fun-ciones involucradas están definidas en todo ); b) allá donde el numerador no sea analítico (pues todas las demás funciones involucradas con enteras).

El numerador, que supondremos raíz cuadrada principal, es analítico en todo el plano comple-jo salvo en los puntos z que hagan que z2 + 3 sea real negativo o cero, porque ahí es donde no es analítico el logaritmo principal asociado a la raíz cuadrada. Para más explicaciones sobre esto, ver el ejercicio 1. Ahora solo voy a buscar qué valores de z hacen que z2 + 3 ≤ 0, o sea, z2 = −3 − p con p positivo o cero. Yo esto lo veo de inmediato, mucho más rápido que calculando cosas, y además prácticamente sin riesgo de equivocarme en las operaciones, con un pequeño dibujo a mano alzada:

Figura 3. Gráfica para saber rápidamente dónde 2 3z + no es analítica.

Viendo la figura sé que los puntos singulares del numerador son con 3z yi y= ≥ .

Y analíticamente:

2 2

3 2 2 2 3 03 ( ) 3 3 2 0

2 0 0 ó 0x y

z x iy x y i xyxy x y

− + ≤+ = + + = − + + ≤ ⇒

= ⇒ = =

2 2 2Si 0 : 0 3 0 3 3 (como antes)x y y y= − + ≤ ⇒ ≥ ⇒ ≥

2 2 2Si 0 : 0 3 0 3y x x x= − + ≤ ⇒ ≤ − ⇒ ∈

Todo lo demás son funciones enteras, así que bastará buscar los ceros del denominador. Em-pezamos por el más fácil:

220 2 log( ) log(1 ) L1ze i z i π−+ = ⇒ = − = = ( ) ( )2 2 4i k z i kπ π π π+ − + ⇒ = − +

El otro: ( )

Apdo. i)cos 2 0 cos 2 arccos( 2 ) log 2 2 1z i z i z i i i− = ⇒ = ⇒ = = − ± − − =

Page 122: Capítulo 1: Números Complejos

7

( )( ) ( )( )( ) ( )( )

2

2

log ( 2 3) log 2 3log( 2 3 ) log ( 2 3)

log ( 2 3) log 3 2

i i ii i i i i

i i i

π

π−

− + = − += − ± = − ± = − − = − −

( )( )( )( )

L( 2 3) 2 2 2 2 L( 3 2)

L( 3 2) 2 2 2 2 L( 3 2)

i i k k iz

i i k k i

π π π π

π π π π

− + + + = + − += − − + − + = − + − −

Pero ( )L( 3 2) L( 3 2) L ( 3 2)( 3 2) L 3 2 L1 0 + + − = + − = − = = ⇒

12 2 L( 3 2)

2 ( 1) L( 3 2)2 2 L( 3 2)

kk i

z z k i kk i

π ππ π

π π+

+ − += ⇒ = + + − + ∀ ∈− + + +

Y no hay más. He ido dejando subrayados los puntos singulares de f (z). Juntándolos:

{ } { } { }13 ( 4 ) 2 ( 1) L( 3 2)kS yi y i k k k i kπ π π π += ≥ ∪ − + ∀ ∈ ∪ + + − + ∀ ∈

Ejercicio 4 (de examen de enero de 2013): i) Deducir la expresión logarítmica de w = argtanh z.

ii) Hallar los puntos singulares de la función argtanh(3 )( )4sin cos

zf zz z i

−=

+.

Resol.: i) Escribiré arctanh porque Mathematica usa ArcTanh. Busco todos los números w cuya tangente hiperbólica sea un número z dado:

sinh ( ) 2arctanh tanhcosh ( ) 2

w w w w

w w w w

w e e e ew z w zw e e e e

− −

− −

− −= ⇔ = = = =

+ +

Por tanto busco todas las soluciones en w de esta última ecuación. Multiplicar todo por ew no añade soluciones porque la función exponencial no vale 0 en ningún punto de :

2 2 2( ) 1 ( 1) (1 ) 1w w w w w w we e z e e e z e z e z− −− = + ⇔ − = + ⇔ − = + ⇔

2 1 1 1 12 log arctanh log1 1 2 1

w z z ze w w zz z z

+ + + = ⇔ = ⇔ = = − − −

ArcTanh[z] // TrigToExp (* solo valor ppal.; compruebo es equivalente *) -(1/2) Log[1 - z] + 1/2 Log[1 + z]

ii) Supondremos que el argtanh del numerador de f (z) es la determinación principal del Arctanh, es decir, la que se obtiene en la expresión deducida en el apartado anterior cuando en lugar del logaritmo multivaluado log se usa el logaritmo principal Log. (En realidad habría sido mejor escribirlo con mayúscula, porque f (z) es una función y, por tanto, univaluada; pero nos enten-demos.) De modo que el numerador de f (z) es:

1 1 (3 ) 1 4Arctanh(3 ) Log Log2 1 (3 ) 2 2

z zzz z

+ − − − = = − − − +

Page 123: Capítulo 1: Números Complejos

8

Los puntos singulares del logaritmo principal (que es el semieje real negativo o cero) pasan a ser los de la determinación principal del Arctanh, por lo que habrá que buscar qué valores de z hacen que (4 − z) / (−2 + z) ≤ 0:

4 40 02 2

z zz z

− −≤ ⇔ ≥

− + −

Estos valores se encuentran muy fácilmente con un pequeño dibujo. El numerador, z − 4, es el vector que va desde el punto 4 sobre el eje real hasta un punto z; y el denominador, el que va desde el punto 2 hasta el punto z. Para que su cociente sea real ambos vectores deben tener la misma dirección (ser paralelos). Y para que el cociente sea real positivo, deben tener el mis-mo sentido.

En la Fig. 4 se ve que, manteniendo fijos los orígenes 2 y 4, las únicas posiciones que puede tomar el afijo z para que ambos vectores tengan igual dirección y sentido son sobre el eje real a la derecha de z = 4 (ambos vectores horizontales hacia la derecha) o a la izquierda de z = 2 (ambos vectores horizontales hacia la izquierda). También valdría z = ∞, pero no considera-mos el plano complejo extendido.

Figura 4. Puntos z para los que (4 − z) / (−2 + z) ≤ 0.

Por tanto el numerador de f (z) tendrá puntos singulares en las semirrectas z ≥ 4 y z ≤ 2. (Tanto z = 4 como z = 2 serán puntos singulares porque cualquier entorno de ellos contendrá otros puntos de ambas semirrectas, en que el numerador de f (z) es discontinuo; además, en el caso de z = 2, el cociente (4 − z) / (−2 + z) ni siquiera está definido).

También podemos operar analíticamente:

2 22 2

2 2

6 8 04 4 2 6 8 ( 2 ) 02 0 02 2 2 ( 2)x x yz x iy x iy x x y i y

y yz x iy x iy x y− + − − ≤− − − − − − + − − + − = ⋅ = ≤ ⇒ − = ⇒ =− + − + − − − +

2 2 2 2 26 8 0 0 6 8 ( 3) 1 0 ( 3) 1 3 1x x x x x x x− + − − ≤ ⇒ − + = − − ≥ ⇒ − ≥ ⇒ − ≥

con igual resultado.

La otra fuente de puntos singulares en f (z) (y no hay más) son los ceros del denominador, donde f (z) no está definida y por tanto no puede ser analítica. Podemos obtenerlos fácilmente escribiendo el denominador en términos del ángulo doble:

4sin cos 2sin(2 ) 0 sin(2 ) 2 2 arcsin( 2)z z i z i z i z i+ = + = ⇒ = − ⇒ = −

A continuación deduciremos la expresión logarítmica multiforme del arco seno y la emplea-remos para obtener todos los posibles valores de 2z:

2sin 2 0 ( ) 2 1 02

iw iwiw iw iw iwe ew z e iz e e ize

i

−−−

= = ⇒ − − = ⇒ − − = ⇒

( )2

2 22 4 4 1 log 12

iw iz ze iz z iw iz z± − += = ± − ⇒ = ± − ⇒

( )2arcsin log 1w z i iz z= = − ± −

Page 124: Capítulo 1: Números Complejos

9

ArcSin[z] // TrigToExp (* comprobación (solo determinación ppal.) *) -I Log[I z + Sqrt[1 - z^2]]

2 1 1 1 52 arcsin log 1 log 1 log

2 2 2 2 4 2 4i i iz i i i i

− − − − = = − ± − = − ± − = − ± =

( )

1 5 5 1 5 1log L 2 2 L2 2 21 5log

2 1 5 5 1 5 1log L 2 2 L2 2 2

i i i k k i

i

i i i k k i

π π

π π π π

+ + +− = − + = − ± = − = − − −− = − + + = + −

Pero como 5 1 5 1 5 1 5 1 5 1L L L L L1 02 2 2 2 4

+ − + − − + = ⋅ = = = ,

sustituyendo: 1

5 12 L2 5 12 ( 1) L

25 12 L2

k

k i

z k i

k i

π

π

π π

+

+− + = = + − ⇒ + + +

1( 1) 5 1L

2 2 2

kkz i kπ + − += + ∀ ∈

zz = Table[k Pi/2 + I (-1)^(k + 1) / 2 Log[(Sqrt[5] + 1) / 2], {k,-4,4}]; 4 Sin[zz] Cos[zz] + I // Simplify (* comprobación *)

{0, 0, 0, 0, 0, 0, 0, 0, 0}

Poniendo juntos los puntos singulares que hemos ido dejando subrayados:

1( 1) 5 1( , 2] [4, ) L

2 2 2

kkS i kπ + − + = −∞ ∪ ∞ ∪ + ∀ ∈

Ejercicio 5 (de examen de julio de 2012): i) Obtener (demostrar) la expresión logarítmica de la función f (z) = arcsin z. ii) Aplicarla para resolver la ecuación sin 3z i= .

Resol.: i) Se trata de encontrar todos los valores w = arcsin z cuyo seno sea el número z, es decir, de resolver en w (todas las soluciones) la ecuación sin w = z con z “dado”:

2sin 2 0 ( ) 2 ( ) 1 02

iw iwiw iw iw iwe ez w e e iz e iz e

i

−−−

= = ⇒ − − = ⇒ − − = ⇒

( )2

2 22 4 4 1 log 12

iw iz ze iz z iw iz z± − += = ± − ⇒ = ± − ⇒

( )2arcsin log 1w z i iz z= = − ± −

ArcSin[z] // TrigToExp (* comprobación (solo valor principal) *) -I Log[I z + Sqrt[1 - z^2]]

Page 125: Capítulo 1: Números Complejos

10

ii) ( ) ( )2arcsin( 3 ) log 3 1 ( 3 ) log 3 1 3 log( 3 2)z i i i i i i i= = − ± − = − − ± − − = − − ±

( ) ( )( ) ( )

0log( 3 2) log (2 3) L(2 3) 2 2 L(2 3)

log( 3 2) log (2 3) L(2 3) ( 2 ) ( 2 ) L(2 3)

i i i ik k i

i i i i k k iπ

π π

π π π π

− − + = − − = − − + = − −

− − − = − + = − + + + = + − +

Pero ( )L(2 3) L(2 3) L (2 3)(2 3) L(4 3) L1 0− + + = − + = − = = →

2 L(2 3)

( 1) L(2 3)2 L(2 3)

kk iz k i k

k i

ππ

π π

+ + = = + − + ∀ ∈ + − +

Ejercicio 6 (de examen de julio de 2012):

Hallar el dominio de analiticidad de 2( ) Log( 2)f z z iz= − + .

Resol.: Supondremos que Log se refiere al logaritmo principal y no a otra rama del logaritmo.

La función f es composición de una función entera (el polinomio z2 − i z + 2) y una función, el logaritmo principal, cuyos únicos puntos singulares son los del semieje real negativo (incluido z = 0). La composición de funciones analíticas en un punto o dominio es analítica en ese punto o dominio. Por tanto solo se trata de encontrar los valores de z que hacen que z2 − i z + 2 sea un número real negativo o cero, que es donde la función Log no es analítica:

2 2 2 22 ( ) ( ) 2 2 2z iz x iy i x iy x y i xy y ix− + = + − + + = − + + − + =

( )2 2

2 2 2 02 2 0

2 0 0 ó 1 2x y y

x y y i xy xxy x x y

− + + ≤= − + + + − ≤ ⇔

− = ⇔ = =

Si x = 0: 2 2 2 2 20 2 0 2 ( 1 2) 9 4 0 ( 1 2) 9 4y y y y y y− + + ≤ ⇔ − − = − − ≥ ⇔ − ≥ ⇔

1 2 3 2 2

1 2 3 2 sing. en con 2 ó 11 2 3 2 1

yy z iy y y

y≥ + =

− ≥ ⇔ ⇒ = ≥ ≤ − ≤ − = −

Si y = 1/2: 2 2 2(1 2) 1 2 2 9 4 0x x x− + + = + ≤ ⇒ ∈

Por tanto las únicas singularidades son las obtenidas al principio (subrayadas), y f (z) será ana-lítica en el complementario D de ese conjunto, es decir, en todo el plano complejo menos en esos puntos:

{ } { } { }2 1 2 1D z iy y y z iy y z iy y= − = ≥ ∨ ≤ − = − = ≥ − = ≤ −

El símbolo ∨ es un “o lógico” no exclusivo, como recordarás por las prácticas con Mathematica, en las que también vimos que el símbolo ∧ es un “y lógico”. Cuando no re-cuerdes cuál es cuál, recuerda que ∨ se parece al símbolo ∪ de unión de conjuntos (formado por los elementos que pertenecen a uno o a otro conjunto, o a ambos), mientras que ∧ se pa-rece al símbolo ∩ de intersección de conjuntos (formado por los elementos que pertenecen a uno y otro conjunto).

También se puede operar de forma ligeramente distinta y obtener el mismo resultado. Lla-mando p a cualquier número real positivo o cero:

Page 126: Capítulo 1: Números Complejos

11

2 2 1 4(2 ) 9 42 2 0

2 2 2i p piz iz p z iz p z± − − + − −

− + = − ⇒ − + + = ⇒ = = ±

y como p ≥ 0, la raíz anterior siempre lo es de un número negativo con lo que:

9 4 9 1 9

2 2 2 4 2 4p ii iz i p i p

+= ± = ± + = ± +

y, de nuevo, como p ≥ 0, la raíz positiva anterior siempre es 9 4 3 2≥ = , llegándose al mis-mo resultado de antes.

La representación gráfica de D sería:

i

2i

3i

−i

−2i

x

y

D

0

Figura 5. Dominio de analiticidad D.

Debes entender bien qué pasa en esos puntos singulares. Recuerda que el logaritmo principal Log(z) es analítico en todo el plano complejo salvo en el semieje real negativo. Dos puntos muy próximos como −9 y −9 − ε i con ε > 0 pero pequeño tienen logaritmos principa-les con partes reales muy próximas pero partes imaginarias que difieren casi en 2π: Log( 9) L9 ; Log( 9 ) L9i i iπ ε π− = + − − −

Cuando una variación muy pequeña en z (de distancia ε, en este caso) puede producir un salto no pequeño en f (z) (Log(z) en este caso, que cambia en casi 2πi), la función f (z) no es conti-nua ahí, y por tanto no puede ser derivable, y por tanto no puede ser analítica6.

En el ejercicio que nos ocupa, valores de z sobre las dos semirrectas de puntos singulares (z = iy con y ≥ 2, y z = iy con y ≤ −1, macadas en la figura), hacen que Log se evalúe en el semieje real negativo. El polinomio es una función continua, y algunas pequeñas variacio-nes de z harán que Log se evalúe a lados distintos de dicho semieje real negativo, produciendo saltos finitos en Log (de tamaño 2πi) y por tanto en f (z), con lo cual f (z) ahí no puede ser de-rivable ni analítica.

Ejercicio 7 (de examen de julio de 2011): En exámenes también se ponen a veces ejercicios facilísimos. (Pero no cuentes con ello.)

Ejercicio 8 (de examen de junio de 2009): Hallar los puntos singulares de la siguiente función y representarlos analítica y gráficamente:

2 4

( 1)1( )( 1 ) cosh sinh z

z zf z

z ix y i z z e−

= +− + − − − +

6 Si esta explicación informal no la has entendido perfectamente bien, repasa la teoría.

Page 127: Capítulo 1: Números Complejos

12

Resol.: Supondremos determinaciones principales y que x, y son las partes real e imaginaria de z.

Primer sumando: voy a poner el denominador en términos de z porque ya me he dado cuenta de que es un polinomio. Si no fuera tan fácil a simple vista, sustituiría ( ) 2x z z= + ,

( ) (2 )y z z i= − , simplificaría y vería lo que sale. Si z apareciera en la expresión simplifi-cada, las ecuaciones de C-R solo se cumplirían en donde 0f z∂ ∂ = , que serán típicamen-te puntos que no formen dominios y por tanto f no será analítica en ningún sitio.

( ) ( )4 42 4 2 2( 1 ) ( ) 1 ( 1 )z ix y i z i x iy i z iz i− + − − = − + − − = − + − −

Por tanto el primer sumando es un cociente de polinomios, o función racional, que sabemos que es analítica salvo en los ceros del denominador, que calculo:

2 1 4 4 3 41 02 2 2

i i i iz iz i z ± − + + +− − − = ⇒ = = ±

2 2

2 2 2 3( ) 2 3 4

2 4 , 0, 2a b

a bi a b i ab iab a b b a

− =+ = − + = + ⇒

= ⇒ ≠ =

2 2 2 2 2 2 4 23 9 16 3 5(2 ) 3 ( ) 3 4 02 2 1

aa a a a a

a⇒ = ±± + ±

− = ⇒ − − = ⇒ = = = − ∈

( )( ) 212 Compruebo: 1 1 1 ok (o con Ruffini)12 2

ii iz z i z z iz i++

= ± = − − + = − − −−

de modo que ya tengo unos primeros puntos singulares (subrayados)7.

Numerador del 2º sumando: será analítica salvo para z = 1 (porque ahí la función subradical no está definida) y, si la raíz es la principal, para los puntos z que hagan que w = z / (z − 1) sea real negativo o cero. Esto es así porque la raíz principal se define como

1 2 1 2Log ww w e= = y Log w deja de ser analítico solo en el semieje real ≤ 0. Un modo:

0 0 [0,1)1 1 1

z z pp p z pz p z zz z p

≤ ⇔ = − ∀ ≥ ⇔ = − + ⇔ = ⇔ ∈− − +

8

Otro modo:

2

2 2

0 01 ( 1) ( ) 0( 1) 01 1 1 ( 1)y yz x iy x iy x x y i yx y xy

x xz x iy x iy x y− = ⇒ =+ − − − + + − −

= ⋅ = ≤ ⇒ − ≤− − + − − − +

De la última desigualdad se deduce [0,1]x∈ , llegándose a los mismos resultados de arriba (subrayados).

Denominador del 2º sumando: es una función entera (por ser combinación lineal de funciones enteras), de modo que sus ceros serán puntos singulares de f:

7 Estos puntos singulares podrían, en el “mejor” de los casos, ser evitables si el resto de f (z) tuviera formas muy concretas; pero siempre serán singulares porque el primer sumando no está definido si su denominador es cero. 8 Lo cual supone z real, igual que si se escribe 0 ≤ z < 1. Los números complejos no son un conjun-to ordenado, de modo que cualquier relación de orden entre ellos implica suponerlos reales.

Page 128: Capítulo 1: Números Complejos

13

2cosh sinh 0 ( ) 12 2

z z z zz z z z ze e e ez z e e e e e

− −−+ −

− + = − + = + = ⇔ = − ⇔

( )log( ) L arg( ) L1 2 2 0 ( 2 )ze i z i i i i i k i kπ π π π= ± ⇔ = ± = ± + ± = + ± + = + +

Por supuesto aquí había que utilizar el logaritmo multiforme, y no su determinación principal, porque buscábamos todos los valores de z que anularan el último denominador, formado por funciones enteras que ni siquiera tienen ramas (principal o no).

Y no hay más puntos singulares que los que hemos ido subrayando. Poniéndolos juntos:

{ } { }1, 1 [0,1] ( 2 )S i i k kπ π= − + ∪ ∪ + ∀ ∈

Representación gráfica pedida:

Figura 6. Puntos singulares de f.

Ejercicio 9 (de examen de junio de 2008):

Hallar, gráfica y analíticamente, los puntos singulares de la función 2

6 3( )2 1

z izf zz z

−=

− +.

Resol.: Eso está mal expresado. No se trata de hallarlos analítica y gráficamente, sino de hallarlos y luego expresarlos analítica y gráficamente.

Empezaré por el denominador, que es más sencillo. Es un polinomio, luego función entera, pero sus ceros serán puntos singulares de f (z) (porque no estará definida en ellos, y por tanto no puede ser analítica en ellos). Habrá que resolver una ecuación bicúbica (que es una ecua-ción cuadrática en el cubo de la incógnita, o como una bicuadrada con z3 en vez de z2):

6 3 3 2 3 3 3 22 4 42 1 ( ) 2( ) 1 0 1 ( 1) 02

z z z z z z± −− + = − + = ⇒ = = ⇒ − = ...

Page 129: Capítulo 1: Números Complejos

14

Ejercicio 10 (de examen de junio de 2008):

Hallar todas las soluciones z ∈ de la ecuación:

2(1 )4sincos

izi ezz

−= .

Resol.: Nada más ver el producto sin z cos z pienso en ponerlo en términos del ángulo doble, má-xime cuando también veo 2z en el numerador. Veamos a dónde nos lleva hacerlo:

24sin cos (1 ) ; sin(2 ) sin cos cos sin 2sin cosizz z i e z z z z z z z= − = + =

2 2

2 2 22sin(2 ) (1 ) ; 2 (1 ) ;2

i z i ziz iz i ze ez i e i e e

i

−−= − = − 2 ( 1i ze i−− = + 2) i ze

( )4 41 ; 1 ; 4 log( ) L arg( ) 0 2 2i z i zi e e i i i z i i i i i kπ π− = = − = = = + = + +

2 2 14 8 2 2 4

k kz k kπ π π π π+ = = + = + ∀ ∈

Ejercicio 11 (de examen de junio de 2007):

Ejercicio 12 (de examen de septiembre de 2006):

Ejercicio 13 (de examen de junio de 2006):

Ejercicio 14 (de examen de enero de 2006): i) Deducir la expresión logarítmica de w = arcsinh z.

ii) Resolver la siguiente ecuación: sinh z = sinh(i z).

Resol.:

i) 2sh 2 0 ( ) 2 1 02

w ww w w we ez w e z e e ze

−−−

= = ⇒ − − = ⇒ − − = ⇒

( )2

2 22 4 4 1 arcsh log 12

w z ze z z w z z z± += = ± + ⇒ = = ± +

ArcSinh[z] // TrigToExp (* comprobación (solo valor principal) *) Log[z + Sqrt[1 + z^2]]

ii) Se podría plantear el sistema que resulta de igualar partes reales e imaginarias, pero no va a ser fácil de resolver directamente, y la existencia del apartado anterior sugiere que em-pleemos la fórmula obtenida en él. La ecuación sh z = sh(i z) equivale a que z coincida con cualquiera de los arco-seno hiperbólicos multivaluados del (univaluado) sh(i z):

( ) ( )2arcsh(sh( )) log sh( ) sh ( ) 1 log sh( ) ch( )z iz iz iz iz iz= = ± + = ± =

log( ) (caso 1)

log2 2 log( ) (caso 2)

iziz iz iz iz

iz

ee e e ee

− −

− + = ± = −

caso 1: ( )log( ) log( ) log( ) L( ) ( 2 ) ( 2 )iz i x iy y yxz e e e e i x k y i x k x iyπ π+ − −= = = = + + = − + + = +

; 2 ; 2 2 ; ; (1 )y x x k y x x k x k z k ik k i kπ π π π π π⇒ − = + = = − = − = = − = − ∀ ∈

donde kπ y −kπ daban lo mismo porque eran para todo k entero.

Page 130: Capítulo 1: Números Complejos

15

caso 2: ( )log( ) log( ) log( ) log( ) log( )iz i x iy y ix y yx xz e e e e e π

− − + −− − += − = − = − = − = =

L( ) ( 2 ) ( 2 ) ;ye i x k y i x k x iy y xπ π π π= + − + + = + − + + = + ⇒ =

2 ; 2 2 ; ; (1 )2 2

x k y x x k x k z k i kπ ππ π π π π π − + + = = = + = + = + + ∀ ∈

Juntando: (1 ), (1 )2

z k i k i kππ π = − + + ∀ ∈

Ejercicio 15 (de examen de junio de 2015):

Page 131: Capítulo 1: Números Complejos

Capítulo 1: Números Complejos. Resolución de ejercicios de seminario.

AVISO IMPORTANTE: Antes de seguir lee este AVISO.

Resoluciones completas:

Ejercicio 1 (de seminario de semana 4 de 2017):

A) Sean z1 y z2 dos vértices opuestos de un rombo. Calcular los otros dos vértices sabiendo que se cumplen las siguientes condiciones: i) z1 y z2 son las raíces de la siguiente ecuación: z2 − (4 + 2i) z + (3 − 4i) = 0.

ii) La longitud del lado del rombo es 2 10 .

B) Ejercicio 1 (de examen de diciembre de 2015), apartado c), enunciado y resuelto en: 1_Num_Comp_ejerc_exam_resol.pdf.

Resol.:

A) 24 2 (4 2 ) 4(3 4 ) 4 2 16 4 16 12 16

2 82 2

i i i i i iz i

i

1,28 2 2 2(1 ) 2 (2 2 ) 4 3 ,i i i z i i i i

No me fío de mis cuentas. Compruebo con Mathematica:

z /. Solve[z^2 - (4 + 2I) z + (3 − 4I) == 0] {-I, 4 + 3 I}

No me fío de Mathematica. Compruebo (ambas raíces simultáneamente!) con Ruffini:

1 4 2 3 4

3 4

1 4 3 0

i i

i i

i

i

Correcto. Ahora con z1, z2 ya puedo dibujar el rombo en su verdadera posición (aproximada-mente). Hazlo tú. Yo razonaré sobre un “rombo abstracto”. Primero calculo su centro C, que será el punto medio de la diagonal z1

− z2:

1 2 4 32

2 2

z z i iC i

i

Semidiagonal conocida: 1 1 4 3 (2 ) 2 2s z C i i

Su longitud: 1 2 2 2 2s i

Sea el triángulo rectángulo con ángulo recto en C e hipotenusa un lado del rombo. Por el Teo-rema de Pitágoras, la longitud de la otra semidiagonal del rombo (cateto del triángulo) es:

2 2

2 2 10 2 2 40 8 32 4 2s

Veo que la semidiagonal s2 es el doble de larga que s1 (y por supuesto perpendicular a ella), así que calcularé s2 girando s1 un ángulo recto y multiplicando por 2, es decir, multiplicando por 2i. Sumando y restando s2 al centro C obtendré los dos vértices pedidos:

2 1 3,4 2 3,42 2 (2 2 ) 4 4 ; 2 ( 4 4 ) 2 5 , 6 3s is i i i z C s i i z i i

Page 132: Capítulo 1: Números Complejos

También se puede resolver calculando las intersecciones de las circunferencias de radio 2 10 y centros los vértices z1, z2, o de ésta(s) con la mediatriz de la diagonal menor.

Ejercicio 2 (de seminario de semana 4 de 2017):

A) Resolver la ecuación z4 − (a − 1) i z = 0 en función de los valores del parámetro a .

B) Dada la ecuación 3 5 2z i 0 , se pide:

i) Sabiendo que una de las soluciones es 1 (1 2 )z i , representar gráficamente,

sin calcularlas, todas las soluciones de la ecuación.

ii) Hallar la expresión de las otras dos raíces z2 y z3 en forma binómica de la manera más sencilla posible.

C) Ejercicio 2 (de examen de enero de 2016), apartado b), enunciado y resuelto en: 1_Num_Comp_ejerc_exam_resol.pdf.

Resol.: A) Una raíz es obviamente z = 0 a . Además si a = 1 esa es la única raíz (pues z4 = 0).

Las restantes salen de z3 = (a − 1) i. Si a > 1, z son las tres raíces cúbicas de un número

imaginario puro positivo, concretamente 3 3 32 2 2 2 2 5, ,

3 3 3 3 3 6 6 21 1 , 1a a a 3, 1a

(y, como no se piden en forma binómica, aplico la ley del mínimo esfuerzo y las dejo así). Y si a < 1, z son las tres raíces cúbicas de un número imaginario puro negativo, concretamente

3 3 32 2 2 2 2 5, ,

3 3 3 3 3 6 2 61 1 , 1a a a

3, 1 a . Poniendo todo junto:

3 3 35

6 6 2

3 3 3 5

6 2 6

1 0

1 0, 1 , 1 , 1

1 0, 1 , 1 , 1

a z

a z a a a

a z a a a

B) B.i) No me fío. Comprobaré con Ruffini:

1 0 0 5 2

1 2 1 2 1 2 2 5 2

1 1 2 1 2 2 0

i

i i i

i i

i

No necesito comprobar más. Al tratarse de las tres raíces cúbicas de algo, deben tener igual módulo y argumentos separados por 2 / 3, lo cual me permite dibujar aproximadamente esto:

2

Page 133: Capítulo 1: Números Complejos

Figura 1. Posiciones de las tres raíces cúbicas.

B.ii) La forma más sencilla que se me ocurre es multiplicar la raíz z1 dada por 1±2/3:

2 2

1 3 1 6 2 3(1 2 )

2 2 2 2z i i z i

3 3

1 3 1 6 2(1 2 )

2 2 2 2z i i z i

3

Ejercicio 3 (de seminario de semana 4 de 2017):

Ejercicio 9 (de examen de enero de 2013), enunciado y resuelto en: 1_Num_Comp_ejerc_exam_resol.pdf.

Ejercicio 4 (de seminario de semana 4 de 2017):

Ejercicio 6 (de examen de enero de 2014), enunciado y resuelto en: 1_Num_Comp_ejerc_exam_resol.pdf.

Ejercicio 5 (de seminario de semana 4 de 2017):

A) Obtener los números complejos z y sabiendo que se cumplen las siguientes

condiciones: | z | = 2, arg4

z

, y una de las raíces novenas de es 13

2i .

B) Ejercicio 19 (de examen de septiembre de 2009), enunciado y resuelto en: 1_Num_Comp_ejerc_exam_resol.pdf.

Resol.:

A) 9 9

9515

762 2

1 13 1 12

i i

Pasando por alto que la “función” arg es multivaluada (es decir, abusando de notación al no añadir los términos ±2kπ):

arg arg( ) arg( ) arg( ) arg( )2 4 4 2 4

zz z z

Por tanto 42 2z i 2

Ejercicio 6 (de seminario de semana 4 de 2017):

Ejercicio 18 (de examen de enero de 2010), enunciado y resuelto en: 1_Num_Comp_ejerc_exam_resol.pdf.

3

Page 134: Capítulo 1: Números Complejos

Capítulo 2: Transformada de Laplace. Resolución de ejercicios de seminario. AVISO IMPORTANTE: Antes de seguir lee este AVISO.

Resoluciones completas:

Ejercicio 1 (de seminario de semana 7 de 2017): A) Sabiendo que la admitancia de un sistema es

( )1 1( ) 2cos(2 ) sin(2 )5 10

ta t e t t−= − + , obtener:

• La función de transferencia del sistema. • La respuesta del sistema a la función delta de Dirac. • La respuesta del sistema a la entrada r(t) = δ (t − 3) suponiendo que parte del reposo.

B) Enunciar la propiedad de la transformada de Laplace que permite obtener L−1[F'(s)]

y aplicar esta propiedad para calcular 2

12

1ln9

ss

− + +

L .

Resol.: A) • La admitancia1 es la respuesta en el tiempo de un sistema con condiciones iniciales nulas a la función escalón unidad. O sea, cuando la entrada es r(t) = H(t), la salida es y(t) = a(t):

SISTEMA( ) ( ) ( ) ( )(C.I. nulas)

r t H t y t a t= =→ →

Esto es en el dominio del tiempo. En el de la frecuencia, la salida Y(s) (siempre refiriéndonos a condiciones iniciales nulas) es igual a la entrada R(s) multiplicada por la función de transfe-rencia G(s): ( ) ( ) ( )Y s R s G s= ⋅ Por tanto: A(s) = H(s)·G(s)

Como la transformada de Laplace del escalón unitario es H(s) = 1 / s (idéntica a la de la función constante 1, porque a Laplace le daba igual lo que sucediera antes de t = 0), podemos despejar G(s): ( ) ( )G s s A s=

Solo queda calcular A(s) y sustituir:

[ ] ( )1 1( ) ( ) 2cos(2 ) sin(2 )5 10

tA s a t e t t− = = − + = L L

[ ] ( ) [ ] 1

1 1 1 1 11 2cos(2 ) sin(2 ) 2cos(2 ) sin(2 )5 10 5 10

ts s

e t t t ts

−→ +

= − + = − + = L L L

1 Admitancia viene de admitir como resistencia de resistir o impedancia de impedir. Cuanto “mayor” (siendo variable en el tiempo) es la respuesta de un sistema al escalón unidad, más “admite” ser influi-do por él (más respuesta da en términos de intensidad eléctrica, desplazamiento, o lo que sea). Un objeto de admitancia cero es un objeto “inamovible”.

Page 135: Capítulo 1: Números Complejos

2

2 2 2 21 1

1 1 2 1 1 1 1 1 225 10 4 4 5 5 4 5 5 ( 1) 4s s s s

s s ss s s s s s s→ + → +

+ + = − + = − = − = + + + + +

2 2

2 2 2 2

1 1 2 1 2 5 2 1 1( )5 2 5 5 ( 2 5) ( 2 5) 2 5

s s s s s G ss s s s s s s s s s s

+ + + − − = − = = ⇒ = + + + + + + + +

Comprobamos con Mathematica: at = 1/5 - 1/10 Exp[-t] (2 Cos[2 t] + Sin[2 t]); As = LaplaceTransform[at, t, s] // Simplify

1 / (5 s + 2 s^2 + s^3) Gs = s As // Simplify

1 / (5 + 2 s + s^2)

• Cuando la entrada en el tiempo es una delta de Dirac (r(t) = δ (t)), en frecuencia es R(s) = L[r(t)] = L[δ (t)] = 1, y la salida en frecuencia es R(s)·G(s) = 1·G(s) = G(s). Por tanto la función de transferencia G(s) es siempre la salida en frecuencia cuando la entrada en el tiempo es la delta de Dirac δ (t) y las condiciones iniciales son nulas.

En nuestro caso nos piden la respuesta en el tiempo a δ (t), que será la transformada inversa de G(s) = 1 / (s2 + 2s + 5):

[ ]1 1 1 12 2 2

1

1 1 1 2( ) ( ) ( )2 5 ( 1) 4 2 4 s s

y t g t G ss s s s

− − − −

→ +

= = = = = = + + + + + L L L L

12

1 2 1 1sin(2 ) 2sin cos sin cos2 4 2 2

t t t te e t e t t e t ts

− − − − − = = = = + L

InverseLaplaceTransform[Gs, s, t] // FullSimplify E^-t Cos[t] Sin[t]

• Ahora el sistema parte del reposo (condiciones iniciales nulas) y hasta t = 3 sigue en repo-so (y(t) ≡ 0), porque la entrada en el tiempo, r(t) = δ (t − 3), es un solo impulso o delta de Dirac en ese instante. Y a partir de t = 3 la respuesta será la misma respuesta impulsiva (o res-puesta impulsional, o respuesta a impulso) correspondiente a la entrada δ (t) de la pregunta anterior, pero 3 unidades de tiempo más tarde. Por ello la salida será la calculada antes, pero sustituyendo t por t − 3 (retrasada 3 unidades de tiempo) y multiplicada por H(t − 3) (anulada hasta dicho instante):

( )( 3) ( 3)1( ) sin 2( 3) ( 3) sin( 3)cos( 3)2

t ty t e t H t e t t− − − −= − = − − −

Y si el razonamiento anterior no te convence, aquí está la justificación formal. La salida en

frecuencia será: [ ] 32

1( ) ( ) ( ) ( 3) ( )2 5

sY s R s G s t G s es s

δ −= ⋅ = − ⋅ =+ +

L

La salida en el tiempo será su transformada inversa. Aplicando desplazamiento en t:

[ ]1 3 1 12 2 3

3

1 1( ) ( 3) ( 3) ( )2 5 2 5

st t

t t

y t e H t H t G ss s s s

− − − −→ −

→ −

= = − = − = + + + + L L L

( )( 3) ( 3)1( 3) ( 3) ( ) sin 2( 3) ( 3) sin( 3)cos( 3)2

t tH t g t y t e t H t e t t− − − −= − − = = − = − − −

como esperábamos.

B) Según la propiedad de derivada de la transformada:

Page 136: Capítulo 1: Números Complejos

3

[ ] [ ] [ ] [ ] [ ]1 1 1 11( ) ( ) ; ( ) ( ) ( ) ; ( ) ( )F s t f t F s t f t t F s F s F st

− − − −−′ ′ ′= − = − = − =L L L L L

Si no recuerdas esa propiedad, la deduces rápidamente:

[ ] ( )0 0

( ) ( ) ( ) ( )st std dF s f t f t e dt f t e dtds ds s

∞ ∞− −∂′ = = = =∂∫ ∫L

( ) [ ]0 0

( )( ) ( ) ( )st stf t te dt t f t e dt t f t∞ ∞− −= − = − = −∫ ∫ L

Por tanto, si F(s) es tal que su derivada F'(s) tenga transformada inversa más fácil de calcular que la de F(s), puede venir bien utilizar esta propiedad. En nuestro caso:

2 2 2

22 2 2 2 2

2

1 1 ( 9)2 ( 1)2 16( ) ln ( )19 ( 9) ( 1)( 9)9

s s s s s sF s F sss s s ss

+ + − +′= → = ⋅ = = ++ + + + +

3 2 3 2

2 2 2 2

009 9

9 161 9 ( 1)( 9)9 0

A CB DAs B Cs D As As Bs B Cs Cs Ds D

A Cs s s sB D

+ = + =+ + + + + + + + + = + = ⇒ + =+ + + + + =

2 2

2 22 ; 2 ; 0 ( )1 9

s sA C B D F ss s

′⇒ = = − = = ⇒ = −+ +

Fs = Log[(s^2 + 1) / (s^2 + 9)]; (* comprobación hasta aquí *) Fprima = D[Fs, s] // Apart

(2 s) / (1 + s^2) - (2 s) / (9 + s^2)

Bueno, esto era para practicar, no porque yo lo hubiera hecho así ;-)2. En realidad hay una manera mucho mejor de calcular la derivada F'(s), y además ya directamente separada en fracciones parciales, casi sin hacer ninguna operación. Observa:

2

2 22 2 2

1 2 2( ) ln ln( 1) ln( 9) ( )9 1 9

s s sF s s s F ss s s

+ ′= = + − + ⇒ = − + + +

Terminamos:

[ ] [ ] ( )1 1 1 12 2

1 2 2( ) ( ) ( ) cos cos(3 )1 9

s sf t F s F s t tt t s s t

− − − −− − ′= = = − = − + + + L L L L

InverseLaplaceTransform[Fs, s, t] (* comprobación *) 2 (-Cos[t] + Cos[3 t]) / t

Ejercicio 2 (de seminario de semana 7 de 2017): A) Resolver el siguiente problema diferencial:

y"(t) + y(t) = f (t) + δ (t − 2π); y(0) = 0, y'(0) = 1

siendo f (t) la función representada en la siguiente gráfica:

2 Como dice el refrán, “quien no tiene cabeza, tiene que tener pies”. Si no te das cuenta de ciertas co-sas, debes estar preparado para hacer más operaciones (“caminar más”).

Page 137: Capítulo 1: Números Complejos

4

¿Es continua y(t)? Razonar la respuesta.

Obtener los valores de y(t) para los siguientes valores de t: 3 5, y .2 2 2

t π π π=

B) Calcular 2sinh (2 ) sint t L .

C) Ejercicio 2 (de examen de enero de 2016), apartado b), enunciado y resuelto en: 2_Tr_Lap_ejerc_exam_resol.pdf.

Resol.: A) De la figura: f (t) = H(t − π) − H(t − 2π)

Aplicando la transformada de Laplace a ambos lados de la EDO:

[ ] [ ]( ) ( ) ( ) ( 2 )y t y t f t tδ π′′ + = + −L L

[ ] [ ]2 ( ) (0) (0) ( ) ( ) ( 2 ) ( 2 )s Y s s y y Y s H t H t tπ π δ π′ − − + = − − − + − L L

2

2 2 2( ) 0 1 ( ) ( 1) ( ) 1s s

se es Y s Y s s Y s es s

π ππ

− −− − − + = + − = − +

2 2

2 2 2 2

1( )1 ( 1) ( 1) 1

s s se e eY ss s s s s s

π π π− − −

= + − ++ + + +

2 2

1 1 1 12 2 2 2

1( )1 ( 1) ( 1) 1

s s se e ey ts s s s s s

π π π− − −− − − − = + − + = + + + +

L L L L

1 12 2

2

1 1sin ( ) ( 2 ) sin( 1) ( 1)t t t t

t H t H t ts s s sπ π

π π− −

→ − → −

= + − + − − + + +

L L

2 2

2 2 2

1 1; 0 ; 1( 1) 1 ( 1)

A Bs C As A Bs Cs A C Bs s s s s s

+ + + += + = ⇒ = = = −

+ + +

1 12 2

1 1 1 cos( 1) 1

s ts s s s

− − = − = − → + + L L

[ ] [ ]( ) sin ( ) 1 cos( ) ( 2 ) 1 cos( 2 ) sin( 2 )y t t H t t H t t tπ π π π π= + − − − + − − + − + − =

( ) ( )sin

sin ( ) 1 cos ( 2 ) 1 cos sin 1 sin cos 22(sin cos ) 2

t tt H t t H t t t t t t

t t t

ππ π π π

π

<= + − + + − − + + = + + ≤ < + ≥

Comprobamos con Mathematica:

Page 138: Capítulo 1: Números Complejos

5

ft = UnitStep[t - Pi] - UnitStep[t - 2 Pi]; edo = y''[t] + y[t] == ft + DiracDelta[t - 2 Pi]; cir = {y[0] -> 0, y'[0] -> 1}; (* condiciones iniciales como regla *) cie = {y[0] == 0, y'[0] == 1}; (* ídem como ecuación (para DSolve) *) ecs = LaplaceTransform[edo, t, s] /. cir; (* ecuación en s *) Ys = Solve[ecs, LaplaceTransform[y[t], t, s]][[1, 1, 2]]

(E^(-2 Pi s) (-1 + E^(Pi s) + s + E^(2 Pi s) s)) / (s (1 + s^2)) yt = InverseLaplaceTransform[Ys, s, t] // FullSimplify

(1 + Cos[t]) HeavisideTheta[-Pi + t] + Sin[t] + HeavisideTheta[-2 Pi + t] (-1 + Cos[t] + Sin[t])

ytdsolve = DSolve[{edo, cie}, y[t], t][[1, 1, 2]] // FullSimplify \[Piecewise] Sin[t] t < Pi 1 + Cos[t] + Sin[t] Pi <= t < 2 Pi 2 (Cos[t] + Sin[t]) t >= 2 Pi 0 True

ytpiece = Piecewise[{{Sin[t], t < Pi}, {1 + Sin[t] + Cos[t], Pi <= t < 2 Pi}, {2 (Sin[t] + Cos[t]), t >= 2 Pi}}]; Plot[yt, {t, 0, 4 Pi}]

2 4 6 8 10 12

3

2

1

1

2

3

Figura 1. Solución y(t), de clase C0((0, ∞)). Plot[yt - ytdsolve, {t, 0, 4 Pi}] (* solo errores de redondeo *) Plot[ytdsolve - ytpiece, {t, 0, 4 Pi}]

La función y(t) sí es continua:

• Se ve en la Fig. 1.

• Se demuestra por el hecho de que todas las funciones involucradas son continuas a trozos y, en los posibles puntos de discontinuidad, que son los instantes en que “entran en acción” H(t − π) y H(t − 2π), ambos escalones vienen multiplicados por cero: ( ) ( ) 2

1 cos 0 ; 1 cos sin 0t t

t t tπ π= =

+ = − + + = O bien de que y(t), en su expresión definida por trozos, es continua en t = π y en t = 2π: ( ) ( ) ( ) ( )2 2

sin 1 sin cos ; 1 sin cos 2(sin cos )t t t t

t t t t t t tπ π π π= = = == + + + + = +

• Y además cabía esperarlo pensando, por ejemplo, en un sistema mecánico masa-muelle-amortiguador gobernado por una EDO de orden 2 como la de este ejercicio,3 al que la fuerza aplicada es una caja o ventana unitaria (diferencia de dos saltos unitarios) y una delta de Dirac. Los saltos unitarios producen discontinuidades en la aceleración y"(t), pero no en la velocidad y'(t) ni en la posición y(t). Incluso la delta de Dirac, que representa una percusión,

3 En este caso ni siquiera habría amortiguador, pues el coeficiente de y'(t) es cero

Page 139: Capítulo 1: Números Complejos

6

es decir, una fuerza infinita durante un tiempo infinitesimal comunicando una cantidad de movimiento finita, produce una discontinuidad en la velocidad y'(t), pero nunca en la posición y(t).4 ¡No hay martillazo capaz de desplazar una masa de un sitio a otro instantáneamente!

El resto es trivial: sin 12 2

y π π = =

3 3 31 sin cos 1 1 0 02 2 2

y π π π = + + = − + =

( )5 5 52 sin cos 2 1 0 22 2 2

y π π π = + = + =

{yt /.t -> Pi / 2, yt /. t -> 3 Pi / 2, yt /.t -> 5 Pi / 2} {1, 0, 2}

B) Pasando el seno hiperbólico a exponenciales5, las transformadas que quedan son inmediatas o casi inmediatas (bastando aplicar la propiedad de desplazamiento en s):

( )22 2

2 4 41sinh (2 ) sin sin 2 sin2 4

t tt te et t t e e t

−−

− = = + − = L L L

[ ] [ ]4 424 4

1 1 1sin sin 2sin sin sin 24 4 1

t ts s s s

e t e t t t ts

−→ − → +

= + − = + − = + L L L

2 2 2

1 4 1 4 1 2( 4) 1 ( 4) 1 1s s s

= + −− + + + +

LaplaceTransform[Sinh[2 t]^2 Sin[t], t, s] // Apart (* comprobación *) -1/2 / (1 + s^2) + 1/4 / (17 - 8 s + s^2) + 1/4 / (17 + 8 s + s^2)

Sumando los tres términos y operando queda, con denominador común: LaplaceTransform[Sinh[2 t]^2 Sin[t],t,s]

(8 (-17 + 3 s^2)) / (289 + 259 s^2 - 29 s^4 + s^6)

es decir: 2

6 4 2

8(3 17)29 259 289

ss s s

−− + +

pero esta expresión suele ser menos útil, pues su destino final es a menudo calcular su trans-formada inversa, lo cual comienza por descomponerla en fracciones parciales.

Ejercicio 3 (de seminario de semana 7 de 2017): A) Una masa de 1 kg está sujeta a un resorte con una constante K = 9 N/m. Se suelta del

reposo 1 m por debajo de la posición de equilibrio y empieza a vibrar. Al cabo de t = π/2 s se golpea la masa hacia arriba con un martillo transmitiendo un impulso de

4 La discontinuidad de y'(t) en t = 2π se ve a simple vista (punto anguloso en la figura). La función es continua en (0, ∞), pero sin derivada primera continua, y por tanto de clase C0((0, ∞)). Las discontinui-dades en y"(t) (o en la curvatura) que se dan en t = π y en t = 2π (cuando “entra” cada función escalón) no es algo que el ojo humano detecte a simple vista como detecta los puntos angulosos. Quizá le falte al cerebro una capa más en su red neuronal. 5 Lo mismo se podría hacer con sin t, pero habría que trabajar temporalmente con números complejos, y además las operaciones quedan más largas.

Page 140: Capítulo 1: Números Complejos

7

3 N·s. La ecuación diferencial que modeliza el sistema es:

2

2 9 3 con (0) 1, (0) 02

d y y t y ydt

πδ ′+ = − − = =

Se pide: i) Obtener la posición de la masa y en función del tiempo t. ii) Hallar la posición de la masa al cabo de t = π/4 s y de t = π s.

B) Sea f (t) la función de la gráfica:

Hallar [ ]( )f tL enunciando las propiedades de la transformada de Laplace utilizadas.

C) Obtener la siguiente integral utilizando la transformada de Laplace:

4 2 2

0( )t tt e e dt

∞ − −−∫

Resol.: A) i) Aplicando la transformada de Laplace a la EDO:

2 20

( ) (0) (0) 9 ( ) 3 3 32 2

ssts Y s sy y Y s t t e dt eππ πδ δ

∞ −− ′ − − + = − − = − − = −

∫L

( )2 2 22 2

39 ( ) 1 3 ( )9 9

s sss Y s s e Y s es s

π π− −

⇒ + = ⋅ − ⇒ = −+ +

edo = y''[t] + 9 y[t] == -3 DiracDelta[t - Pi/2]; cir = {y[0] -> 1, y'[0] -> 0}; (* condiciones iniciales como reglas *) cie = {y[0] == 1, y'[0] == 0}; (* como ecuaciones, para DSolve *) ecs = LaplaceTransform[edo, t, s] /. cir // Simplify

3 E^(-Pi s / 2) + (9 + s^2) LaplaceTransform[y[t],t,s] == s Ys = Solve[ecs, LaplaceTransform[y[t],t,s]] // Simplify; Ys = Ys[[1, 1, 2]]

(-3 E^(-Pi s / 2) + s) / (9 + s^2)

Comprobado que Y(s) está bien, calculo su transformada inversa:

[ ]1 1 1 122 2 2

2

3 3( ) cos(3 )9 9 2 9

s

t t

sY s e t H ts s s

π

π

π−− − − −

→ −

= − = − − = + + + L L L L

( )cos(3 ) ( 2)sin 3( 2) cos(3 ) ( 2)sin(3 3 2)t H t t t H t tπ π π π= − − − = − − − =

( )cos(3 ) ( 2)cos(3 ) ( ) 1 ( 2) cos(3 )t H t t y t H t tπ π= − − = = − −

yt[t_] = InverseLaplaceTransform[Ys, s, t] (* ok *) -Cos[3 t] (-1 + HeavisideTheta[-Pi / 2 + t])

DSolve[{edo, cie}, y[t], t][[1, 1, 2]] (* son el mismo escalón! *) -Cos[3 t] (-1 + HeavisideTheta[-Pi + 2 t])

ii) Lo que queda es trivial:

( ) ( )( 4) 1 ( 4 2) cos(3 4) 1 0 cos(3 4) ( 4) 2 2y H yπ π π π π π= − − = − = = −

( ) ( )( ) 1 ( 2) cos( ) 1 1 cos( ) ( ) 0y H yπ π π π π π= − − = − = =

Page 141: Capítulo 1: Números Complejos

8

Será interesante representar gráficamente la función y(t) y, de paso, los dos puntos pedidos en este apartado ii): Show[Plot[yt[t], {t, 0, 1.5 Pi}, PlotRange -> {-1, 1}], Graphics[{PointSize[Large], Blue, Point[{Pi / 4, yt[Pi / 4]}]}], Graphics[{PointSize[Large], Blue, Point[{Pi, yt[Pi]}]}]]

1 2 3 4

1.0

0.5

0.5

1.0

Figura 2. Solución y(t). ¡El golpe en t = π/2 deja a la masa en reposo absoluto!

B) Para t entre 0 y 1, f (t) debe ser t; entre 1 y 2 debe ser 1; y entre 2 y 3 debe ser 3 − t (pendiente −1 y, prolongando hasta el origen, ordenada 3). Fuera de ahí debe ser cero. Por tanto:

( ) ( ) ( )( ) ( ) ( 1) 1 ( 1) ( 2) (3 ) ( 2) ( 3)f t t H t H t H t H t t H t H t= − − + − − − + − − − −

Para calcular F(s) será mejor agrupar todo lo que multiplica a cada función escalón, y hacerlo de la siguiente manera:

( ) ( ) ( 1)( 1) ( 2)( 2) ( 3)( 3)f t H t t H t t H t t H t t= − − − − − − + − − ft = UnitStep[t] t - UnitStep[t - 1] (t - 1) - UnitStep[t - 2] (t - 2) + UnitStep[t - 3] (t - 3); (* compruebo que el dibujo sale como espero *) Plot[ft, {t, -1, 4}, PlotStyle -> Thick, AspectRatio -> Automatic]

1 1 2 3 4

0.20.40.60.81.0

Figura 3. Función f(t). He operado bien.

Para calcular F(s) usaré la propiedad de linealidad, la de traslación o desplazamiento en el tiempo (siempre retraso), y la transformada de t, que es inmediata:

[ ] [ ] [ ] [ ] [ ]( ) ( ) ( 1)( 1) ( 2)( 2) ( 3)( 3)f t H t t H t t H t t H t t= − − − − − − + − − =L L L L L

[ ] [ ] [ ] [ ]2 3 2 3 2(1 )s s s s s st e t e t e t e e e s− − − − − −= − − + = − − +L L L L

LaplaceTransform[ft, t, s] // Apart 1 / s^2 + E^(-3 s) / s^2 - E^(-2 s) / s^2 - E^-s / s^2

C) 4 2 2 4 4 4 4

1 2 20 0 01 2 2( )t t st st

s ss sI t e e dt t e dt t e dt t t

∞ ∞ ∞− − − −

→ →→ → = − = − = − = ∫ ∫ ∫ L L

55 5 5

1 2 2

4! 4! 24 24 3 306924 2 24 32 768 767.252 32 4 4s s

Is s→ →

= − = ⋅ − = ⋅ − = − = = =

Page 142: Capítulo 1: Números Complejos

9

Ejercicio 4 (de seminario de semana 7 de 2017): A) Ejercicio 4 (de examen de diciembre de 2014), apartado a), enunciado y resuelto en: 2_Tr_Lap_ejerc_exam_resol.pdf.

B) i) Dada cos(4 ) 0

( )0 en otro caso

t tf t

π≤ <=

, definir f (t) utilizando la función escalón

H(t) y hallar L[ f (t)].

ii) Se considera el siguiente problema de valor inicial:

16 ( ) con (0) 0 , (0) 0y y f t y y′′ ′+ = = =

Se pide obtener su función de transferencia y calcular la respuesta y(t) de dos for-mas distintas: aplicando directamente la transformada de Laplace a la ecuación di-ferencial y utilizando el concepto de función de transferencia.

Resol.: B) Este ejercicio es muy parecido al Ejercicio 4 (de examen de diciembre de 2014), aparta-do b), enunciado y resuelto en 2_Tr_Lap_ejerc_exam_resol.pdf; pero cambia una condición inicial y, al ser ahora ambas nulas, se puede utilizar la función de transferencia. Pondré aquí la resolución completa de este sin referencias al otro.

i) Como tendré que calcular F(s), escribir f (t) de la siguiente manera me permitirá aplicar propiedades de la transformada de Laplace con facilidad:

( )( ) ( ) ( ) cos(4 ) ( ) cos(4 ) ( ) cos(4 )f t H t H t t H t t H t tπ π= − − = − −

ft = UnitStep[t] Cos[4 t] - UnitStep[t - Pi] Cos[4 t]; Plot[ft, {t, -Pi/2, 3Pi/2}, AspectRatio -> Automatic, PlotStyle -> Thick]

1 1 2 3 4

1.0

0.5

0.5

1.0

Figura 4. Función f (t). Aplicando propiedades (linealidad, desplazamiento en t) y transformadas inmediatas:

[ ] [ ] [ ] [ ]( ) ( ) cos(4 ) ( ) cos(4 ) cos(4 ) ( ) cos(4 )f t H t t H t t t H t tπ π= − − = − − =L L L L

( ) [ ]2 2cos 4( ) cos(4 4 )16 16

s ss se t e ts s

π ππ π− −= − + = − + = + +L L

[ ] ( )2 2cos(4 ) ( ) 116 16

s ss se t F s es s

π π− −= − = = −+ +

L

LaplaceTransform[ft, t, s] // Simplify (* compruebo *) (s - E^(-Pi s) s) / (16 + s^2)

ii) Aplicando la transformada de Laplace a ambos lados de la EDO:

[ ] [ ] ( )2216 ( ) ; ( ) (0) (0) 16 ( ) 1

16s sy y f t s Y s sy y Y s e

sπ−′′ ′ + = − − + = − +

L L

( )2 22 2 2( ) 0 0 16 ( ) 16 ( ) ( )

16 ( 16)

s ss se s ses Y s Y s s Y s Y ss s

π π− −− − − − + = + = ⇒ = + +

Page 143: Capítulo 1: Números Complejos

10

edo = y''[t] + 16 y[t] == ft; cir = {y[0] -> 0, y'[0] -> 0}; (* condiciones iniciales como reglas *) cie = {y[0] == 0, y'[0] == 0}; (* ídem como ecuaciones, para DSolve *) ecs = LaplaceTransform[edo, t, s] /. cir // Simplify

(16 + s^2) LaplaceTransform[y[t],t,s] == (s - E^(-Pi s) s) / (16 + s^2) Ys = Solve[ecs, LaplaceTransform[y[t], t, s]][[1, 1, 2]] // Simplify

(E^(-Pi s) (-1 + E^(Pi s)) s) / (16 + s^2)^2

Las transformadas inversas necesarias para calcular y(t) las tengo del apartado A):

12 2

sin(4 ) ( )sin(4( ))( ) ( )( 16) 8 8

ss se t t t ty t H ts

π π ππ−

− − − −= = − − = + L

( )sin(4 )( ) ( ) ( )8

ty t t H t tπ π= − − ⋅ −

yt = InverseLaplaceTransform[Ys, s, t] // Simplify -(1/8) (-t + (-Pi + t) HeavisideTheta[-Pi + t]) Sin[4 t]

ytdsolve = DSolve[{edo, cie}, y[t], t][[1, 1, 2]] // Simplify -(1/8) Sin[4 t] (-Pi + UnitStep[Pi - t] (Pi - t UnitStep[t]))

Plot[yt - ytdsolve, {t, 0, 3 Pi / 2}, PlotStyle -> Thick]

1 2 3 4

3. 10 17

2. 10 17

1. 10 17

1. 10 17

2. 10 17

3. 10 17

Figura 5. Solo errores de redondeo.

Como las condiciones iniciales son nulas, podemos obtener el mismo resultado aplicando el concepto de función de transferencia G(s). Es ésta la función por la que hay que multiplicar la entrada en el dominio de la frecuencia para obtener la salida. También es igual a la salida cuando la entrada es una delta de Dirac δ(t), porque L[δ(t)] = 1. De modo que, aplicando la transformada de Laplace a la EDO en estas condiciones:

[ ]2 22

1( ) 0 0 16 ( ) ( ) 1 ( 16) ( ) 1 ( )16

s Y s Y s t s Y s G ss

δ − − + = = ⇒ + = ⇒ = +L

Ahora basta multiplicar la transformada de la entrada por G(s):

( ) 2 2 2 2

1( ) ( ) ( ) 116 16 ( 16)

ss s s seY s F s G s e

s s s

ππ

−− −

= ⋅ = − ⋅ =+ + +

y lo demás exactamente igual que antes.

Ejercicio 5 (de seminario de semana 7 de 2017): A) Ejercicio 7 (de examen de enero de 2014), apartados ii), iii), enunciados y resueltos en: 2_Tr_Lap_ejerc_exam_resol.pdf.

B) Ejercicio 13 (de examen de noviembre de 2012), enunciado y resuelto en: 2_Tr_Lap_ejerc_exam_resol.pdf.

Page 144: Capítulo 1: Números Complejos

11

Ejercicio 6 (de seminario de semana 7 de 2017): Ejercicio 10 (de examen de junio de 2013), enunciado y resuelto en: 2_Tr_Lap_ejerc_exam_resol.pdf.

Page 145: Capítulo 1: Números Complejos

EJERCICIOS DE NÚMEROS COMPLEJOS (2017/18)

EJERCICIO 1 (Diciembre 2015)

a) Determinar los números complejos tales que  4 0z z .                                                           

b) Las dos raíces de la ecuación:  2 (3 3 ) 2 3 0z i z i  son uno de los vértices de un triángulo equilátero (la de mayor componente real) y el centro de la circunferencia circunscrita a dicho triángulo. Hallar los otros dos vértices.                                                    

c) Hallar, en forma binómica, las raíces de la ecuación:  7 49 8 0z z z                                        

SOLUCIÓN  

a) 2 4 6 8

5 5 5 52 4 6 8

5 5 5 5

0 , 1 , 1 , 1 , 1 , 1i i i i

z z z e z e z e z e

 

 

b) 1B  ,   2 3D i  

 

c) 1 2 3 4 5 6

7

1 30, 2, 1 3 , 1 3 , 1 , ,

2 2

1 3

2 2

z z z i z i z z i

z i

 

EJERCICIO 2 ( Enero 2016)

a) Pueden ser  1 2 3 42 , 2 , 1 2 1 2z i z i z i y z i   las cuatro raíces

cuartas de un número complejo. Razonar la respuesta.                           

b) Resolver la siguiente ecuación: 4

3z iz

           

c) Hallar dos números complejos tales que su suma sea 3 i , la parte real del primero es 2 y el producto de ambos es un número real.

SOLUCIÓN  

a) Los cuatro números complejos tienen el mismo módulo pero la diferencia de argumentos entre dos consecutivas no es la misma luego no pueden ser las cuatro raíces cuartas de un número complejo.

b) 4z i y z i

c) 1 22 2 , 1z i z i

EJERCICIO 3(Junio 2016)

a) El número complejo 1 2 4z i es el centro de una circunferencia de radio 2. Sea

ABC el triángulo equilátero circunscrito a la circunferencia anterior. Uno de los

Page 146: Capítulo 1: Números Complejos

puntos de tangencia es número complejo 2 4z i . Hallar el número complejo

correspondiente al vértice del triángulo de mayor componente imaginaria.

b) Sean 1z y 2z las raíces de la ecuación 4 2 1 0z z situadas en el semiplano

superior hallar el valor de 3 31 2z z

SOLUCIÓN  

a) (4 2 3)i

b) 3 31 2 0z z

EJERCICIO 4 (Diciembre 2014)

Hallar los afijos de los 4 vértices de un rombo regular 1 2 3 4, ,z z z y z , sabiendo que:

i) 1z es imaginario puro negativo y 1 3z

2z está situado en el 2º cuadrante

1 2z y z son vértices opuestos del rombo.

ii) 2 2

1 1

5 ; Re 1z z

z z

iii) 3 1 2 1

5

2z z z z

SOLUCIÓN  

1 3z i  ,  2 6 3z i  ,  3 3 6z i y 4 9 6z i

EJERCICIO 5 (Junio 2014)

A) Calcular, dando el resultado en forma binómica, el valor de la siguiente expresión:

8

12 12A cos i sin

B) Sea 1

3 3 33

2 2z i

un vértice de un triángulo equilátero inscrito en una

circunferencia de centro 3 2C i . Hallar el vértice de mayor componente imaginaria

de dicho triángulo.

SOLUCIÓN  

A) 1 3

2 2A i B) 2 3 3z i

Page 147: Capítulo 1: Números Complejos

EJERCICIO 6 (Enero 2014)

i) Obtener y representar gráficamente el lugar geométrico de los números

complejos, z x i y tales que 2

1z

z .

ii) De los infinitos puntos que pertenecen al lugar geométrico anterior, dar dos puntos 1 y 2 tales que 1 2 sea un número imaginario puro.

iii) Sean 1

3 1

2 2z i , 21

2 1z z y 3z la raíz situada en el 2º cuadrante de la

ecuación:  3 1 0z , expresar 2z y 3z en forma binómica.  SOLUCIÓN  

i) 2 2

0, 0

( 1) 1

ysalvo z

x y

ii) 1 21 1z i y z i .

iii) 21/6 21 /6 /21 2 1

3 1

2 2i i iz i e z z e e i

0

3 2 /333

4 /3

1 31 0 1

2 2

i

i

i

e

z z e i

e

EJERCICIO 7 (Nov. 2013)

i) Calcular

4

1 3

1

iw

i

, dando el resultado en forma binómica.

-2 -1 1 2

-1.5

-1.0

-0.5

0.5

1.0

1.5

Page 148: Capítulo 1: Números Complejos

ii) Hallar el vértice A, de menor componente imaginaria del triángulo equilátero

circunscrito a la circunferencia de centro 2 1 3C i y tangente al triángulo en el

punto que viene dado por el número complejo 3+2i.

SOLUCIÓN  

i) 4

1 32 1 3

1

iw i

i

ii) 2 3 3 2 3A i

EJERCICIO 8 (Junio 2013)

1- Expresar en forma binómica los siguientes números complejos:

i) 251 (1 )z i ii) 26

2 (1 )z i iii) 273 ( 1 )z i

SOLUCIÓN

25 121 (1 ) 2 (1 )z i i 26 13

2 (1 ) 2z i i 27 133 ( 1 ) 2 (1 )z i i

2- Hallar el lugar geométrico de los números complejos 13z i z , y representarlo

gráficamente, en los siguientes casos:

i) 1

1 1z

con 1

1

1

0,3

ii) 1

1 1z

con 1

1

2

2,

6 3

SOLUCIÓN

2-i) z

con

2

,6 2

2-ii) z

con

4

5,

3 6

EJERCICIO 9 (Enero 2013)

Dada la ecuación 2 ( 6 2 ) (11 2 ) 0z i z i . Se pide:

i) Hallar las raíces 1 2 y z z de la ecuación anterior. Considerar 2z la raíz situada en el primer cuadrante.

ii) Sean 1 2 y z z los números complejos cuyos afijos son los vértices de un lado de un

hexágono regular. Hallar el número complejo 3z  cuyo afijo es el otro vértice, el de

mayor componente imaginaria, contiguo con 2z . SOLUCIÓN

1 2 34 3 2 (1 2 3) (3 3)z i z i z i

Page 149: Capítulo 1: Números Complejos

EJERCICIO 10 (Nov. 2012)

i) Hallar las raíces 1 2 y z z de la ecuación 2 (3 4 ) 0z i , con 1z situada en el cuarto

cuadrante.

ii) Hallar los números complejos y , sabiendo que: 1 Re( ) 3

Im( ) 0arg( ) 2

y

y que el cociente 1

(3 2 )13

i .

iii) Sean 3 1 4 2 y z z z z los afijos de los vértices de la diagonal menor de un

rombo regular. Hallar los afijos de los otros dos vértices, sabiendo que la longitud de la diagonal mayor es el doble de la diagonal menor.

SOLUCIÓN

1 2i) 2 2z i z i ii) 1 3 2i

3 4 5 6iii) 3 1 3 2 6 3z i z i z i z i

EJERCICIO 11 (Julio 2012)

i) Hallar dos números complejos y , sabiendo que es la raíz imaginaria pura de

la ecuación: 3 8 0z i y que 13

ii) Hallar otro número complejo , tal que Re 0

SOLUCIÓN  

i) 3 2 2i i            4 3 3 3 3ii)

2 2i

 

EJERCICIO 12 (Enero 2012) i) La ecuación 3 2(4 4 ) (2 11 ) (7 ) 0z i z i z i tiene 3 raíces complejas:  

0 1 2, y z z z . Una de ellas, 0z es imaginaria pura y también es raíz de la ecuación:3 0z i . Hallar las otras dos raíces 1 2y z z .

ii) Sean 1 3z i y 2 1 2z i dos números complejos. Hallar otro número complejo,

3z , tal que: 3 2 3 1 2 1

5

2z z z z z z y 3 2Im Imz z

SOLUCIÓN

1 2i) 3 1 2z i z i 3

7ii) 3

2z i

Page 150: Capítulo 1: Números Complejos

EJERCICIO 13 (Nov. 2011)

Sea 3 1

2 2z i . Se pide:

1) Obtener en forma binómica: i) 5

1 ( )z z

ii) 92 ( )z z

2) Hallar el número complejo 3z , (el de mayor componente imaginaria), tal que los

afijos de 1 2 3, y z z z formen un triángulo rectángulo isósceles, sabiendo que los vértices

de la hipotenusa son los afijos de 1 2y z z .

SOLUCIÓN

1 2 3

3 1 (3 3) (1 3)

2 2 4 4

i iz z i z

EJERCICIO 14 (Mayo 2011)

Se consideran los números complejos:

7

5 2 22

2 2 2

iA y B i

i

Y sea M A B el vértice de un triángulo equilátero MNP inscrito en una circunferencia centrada en el origen. Hallar los afijos de los tres vértices del triángulo.

SOLUCIÓN

1 1 1 11 3 (3 3 1) ( 3 3) ( 3 1) (3 3)

2 2 2 2M i N i P i

EJERCICIO 15 (Enero2011)

Sea 1 3z i una raíz de la ecuación 30 0z z . Se pide:

i) Hallar las otras dos raíces 2 3 y z z de la ecuación anterior.

ii) Si los afijos de 2 3 y z z son los vértices de la diagonal menor de un rombo regular,

hallar los afijos de los otros dos vértices de dicho rombo, sabiendo que la relación entre

la diagonal menor y la diagonal mayor es 1

2.

SOLUCIÓN

2 3 4 5

3 1 3 1i) 3 2 ii) =( 3) ( 3 ) =( 3) ( 3 )

2 2 2 2z i z i z i z i

EJERCICIO 16 (Enero2011)

Hallar analítica y gráficamente el lugar geométrico de los números complejos en los siguientes casos:

Page 151: Capítulo 1: Números Complejos

a) z

z b)

2

z

z c)

3z

z

Cuando z recorre la semirrecta :

3

3

3

2

y x

x

SOLUCIÓN

a) 3

1

b) 2

1

c) 2

2

3

EJERCICIO 17 (Sept. 2010)

i) Hallar las raíces 1 2 3, z z y z de la ecuación 3(1 ) 8 2 0i z i siendo 1z la raíz

situada en el primer cuadrante. ii) Dibujar el cuadrado circunscrito a la circunferencia que pasa por 1 2 3, z z y z , y es

tangente en 1z a dicha circunferencia.

iii) Hallar en forma binómica el afijo del vértice de mayor componente imaginaria del cuadrado anterior.

SOLUCIÓN

1 2 3 3 17

12 4 12

i) 2 , z = 2 y 2 iii) 2 6z z i

EJERCICIO 18 (Enero2010)

Una raíz de la ecuación 40 0z z es 1 3 1 3 1z i . Se pide:

i. Módulo y argumento de 1z . Dato: 3 1

12 3 1tg

ii. Hallar 0z en forma binómica.

iii. Hallar las demás raíces de dicha ecuación. iv. Si los afijos de las raíces situadas en el tercer y cuarto cuadrante son los vértices A

y B del lado desigual de un triángulo isósceles, hallar el otro vértice C (el de menor componente imaginaria del triángulo) sabiendo que la longitud de la altura correspondiente al vértice C es igual a la longitud del lado AB.

SOLUCIÓN

51 012

i) 2 2 ii) 2 (1+ i 3)z z

1 2

3 4

iii) 3 1 3 1 , 3 1 3 1 ,

3 1 3 1 3 1 3 1

z i z i

z i z i

Page 152: Capítulo 1: Números Complejos

iv) C= 3 3 3 i

EJERCICIO 19 (Sept.2009)

Sean 1 2 y z z dos números complejos tales que su cociente está situado en el eje imaginario positivo, su suma es igual a 10+10i, y el módulo de uno de ellos es el triple que el del otro. Se pide:

i) Hallar 1 2 y z z .

ii) Si 1 2 y z z son los vértices opuestos de un cuadrado, hallar los otros dos vértices

3 4 y z z SOLUCIÓN

3 41 2

1 2 3 4

) z 12 4 2 6 ) 6 12 4 2 i) ii)

) 12 6 2 4 ) z 4 12 6 2

a i z ia z i z i

b z i z i b i z i

EJERCICIO 20 (Junio 2009)

Sea 1z la raíz de la ecuación 2 1 0z i z i de menor componente imaginaria, y sea

2z la otra raíz. Sea C la circunferencia de centro 1z que pasa por 2z , y sea T el triángulo

equilátero circunscrito a C que es tangente a C en 2z . Hallar el vértice de mayor

componente imaginaria de T. SOLUCIÓN

1 21 1z z i El vértice pedido: (1 3) (1 2 3)i

EJERCICIO 21 (Junio 2006)

Sea z x i y un número complejo no nulo, y sea z

z . Se pide:

i) Expresar el módulo y el argumento de en función de z y en función de x e y.

ii) Indicar gráficamente el lugar geométrico que describe el afijo de cuando el afijo de z recorre el arco, situado en el primer cuadrante, de una circunferencia de radio R=1 y centro el origen de coordenadas.

iii) Si el radio de la circunferencia del apartado anterior fuera R=3, indicar razonadamente cuál sería la respuesta en este caso.

SOLUCIÓN

i) 1 arg( ) 2arg( ) 2y

z arctgx

) y )ii iii Tienen el mismo lugar geométrico: Semicircunferencia de centro el origen y R=1, situada en el semiplano inferior.

Page 153: Capítulo 1: Números Complejos

EJERCICIO 22 (Febrero 2006)

Los afijos de las raíces 1 2 y z z de la ecuación: 2 (3 4 ) ( 1 7 ) 0z i z i son el

centro y el punto medio del lado AB de un hexágono regular, respectivamente. Hallar los afijos de los vértices A y B, sabiendo que 1 2Im Imz z .

SOLUCIÓN

1 22 1 3z i z i

Vértice A: 2 3 3

1 33 3

i

Vértice B:

2 3 31 3

3 3i

EJERCICIO 23 (Febrero 2005)

Hallar tres números complejos 1 2 3, y z z z  cuyos afijos equidisten entre sí, y del afijo de

otro número complejo 0z , sabiendo que 0 1y z z son respectivamente las soluciones de

las ecuaciones: 3 28 0 2 3 2 2 3 0z i y z i z i situadas en el primer

cuadrante.

SOLUCIÓN

1 2 32 3 2 2 3z i z i z i

EJERCICIO 24 (Enero2004)

Se consideran la circunferencia de radio R, cuyo centro C es el afijo de la raíz de la ecuación: 2 2(1 2 2 ) 0z i situada en el primer cuadrante, y el cuadrado de vértices A, B, D, E inscrito en ella, siendo A el de mayor componente imaginaria, B el de mayor componente real y (3 2 )M i el punto medio del lado AB.

Hallar el número complejo cuyo afijo es el vértice de mayor componente imaginaria del cuadrado de lados paralelos al anterior e inscrito en una circunferencia concéntrica a la

anterior y de radio 2 2R .

SOLUCIÓN

6 2 2 7 2 4i

EJERCICIO 25 (Enero2003)

Calcular el vértice de mayor componente imaginaria del triángulo circunscrito a la

circunferencia que pasa por las raíces de la ecuación: 3 2 20

2 2z i siendo los

lados del triángulo tangentes a la circunferencia en las raíces anteriores

SOLUCIÓN

Page 154: Capítulo 1: Números Complejos

1 12 6 2 6

2 2i